Download as docx, pdf, or txt
Download as docx, pdf, or txt
You are on page 1of 99

LEGAL MEDICINE

SEMESTRAL EXAM

HAS TREATED ANY PERSON OR


PERSONS OF SERIOUS PHYSICAL
1. Test used to determine whether
INJURIES. With malicious intent or gross
victim drowned in salt or fresh
negligence, shall suffer the penalty of
water is: Gettler
imprisonment for not less than one year nor
more than three (3) years and/or a fine of not
less than 1,000 nor more than 3,000 pesos, as
Rationale: Gettler’s test is used to a military tribunal may direct.
estimate the chloride content of the
blood from both sides of the heart.
If levels are less on the right side of 5. Consent for in vitro fertilization
the heart, victim is drowned by salt between married couple must be in a
water. If the are less on the left side, public document: before the in
victim is drowned by fresh water. vitro fertilization is carried out

Rationale: Written consent shall be


2. Special type od injury that may fabir documented before the procedure
homicide than murder: Patterned
wound
6. Reckless imprudence resulting in
Rationale: Patterned wound - Wound in the homicide: Four years and two
nature and shape of an object or instrument months
and which infers the object or instrument
causing it. Rationale:
Reckless imprudence resulting to
physical injuries (Article 365 in Revised Penal
3. In which of the following instances Code)
can you invoke your right against - careless, reckless, inexcusable, not justified
self-incrimination? Submission of - prison term: 4 yrs & 2 months
semen specimen - you can apply for probation

Rationale: Submission of semen specimen


can not be forced by the judge
7. Non official autopsy: elective

4. One year nor more than three (3) Rationale:


years and/or a fine of not less than Official autopsy- Medicolegal autopsy Non
1,000 nor more than 3,000 pesos pd: official autopsy- elective
Pd 169
Rationale: THE ATTENDING
PHYSICIAN OF ANY HOSPITAL WHO
LEGAL MEDICINE
SEMESTRAL EXAM
to determine the nature of death.
8. When called upon by the judicial
authorities, the following are duly
12. Commonly used method of deception
bound to assist in the administration - detection : LlE DETECTlON
METHOD
of justice on matters which are
Rationale: Methods of deception detection
medico-legal in character, EXCEPT: used by law enforcement agencies: Polygraph
or lie detector machine Polygraph or lie
Clinic secretary
detector machine

Rationale: It is the duty of every physician, 13. Impotency as a ground for annulment
when called upon by the judicial authorities, must appear to be present and
incurable: within five years of the
to assist in the administration of justice on
marriage
matters which are medicolegal in character.
Rationale: The action for annulment of
9. The complete relaxation and softening marriage based on this ground must be filed
of all the muscles found immediately by the injured party within five (5) years after
after death is termed: Rigor Mortis the celebration of the marriage

Rationale: Rigor mortis- Immediately


after death all of the muscles in the body 14. judicial death except: Mercy killing
relax.
Rationale: METHODS
10. Average time of death in drowning is: 1. Death by Electrocution
2 to 5 minutes 2. Death by Hanging
3. Death by Musketry
Rationale: Submersion for 1-1/2 minutes is 4. Death by Gas Chamber
considered fatal, if ordinary efforts for
respiration is made, however, a person may 15. Its importance approximate time of
survive even after 4 minutes of submersion. death: Rigor mortis
The average time required for death in
drowning is 2 to 5 minutes. Rationale: see rationale in number 11

16. A medical witness in the witness


11. Its Medico-Legal importance is to stand who vehemently refuses to
approximate the time of death: all answer questions propounded in court
may be declared guilty of a. DlRECT
Rationale: The following are used to CONTEMPT
determine the time of death: pallor mortis,
algor mortis, livor mortis, rigor mortis, and Rationale: Direct Contempt- vehemently
decomposition. Cadaveric spasm is used refuses to answer questions in court

Indirect Contempt- refuses or failed to


LEGAL MEDICINE
SEMESTRAL EXAM
appear without justifiable reason even - prison term: 4 yrs & 2 months
subpoenaed by the court - you can apply for probation

17. While doing an exploratory 19. Nonofficial autopsy is also known as:
laparotomy, Dr. P. Baya forgot to do
elective autopsy
a sponge and instrument count
because he was hurrying to catch a
date with his intern. As a result, he Rationale:
left a scalpel inside the abdominal Non-official/ Non-medicolegal-
cavity which caused the death of his Hospital-based or elective autopsy
patient. He is subsequently sued and
prosecuted. He is most probably
guilty of: Reckless imprudence 20. Washerwoman’s hands is indicative
resulting to homicide
that the victim: Has been
Rationale: immersed in water for some
Reckless imprudence resulting to
physical time
injury or homicide Action is
careless, imprudent, Rationale:
but unintentional Recklessness is
NOT excusable Washerwoman's hand and feet. The
and justifiable Maximum penalty is no skin of the hands and feet, is
more than 4 bleached, corrugated and sodden in
years and 2 months, since it is less appearance. This is not diagnostic of
than 6 years, drowning but proves only that the
you can apply for probation, but body has remained in water for some
you have to prove time without reference as to the cause
the probation administration that you of death.
are a modest
citizen 21. The ff are classified as closed wounds
except: perforating wound
18. Reckless Imprudence resulting in
homicide under Art 365 of the Rationale:
Revised Penal code is punishable with Perforating wounds are open wounds.
imprisonment of up to: Four years Examples of perforating wounds
and two months would be
stab wounds caused by
Rationale: penetration of a sharp-pointed
Reckless imprudence resulting to
instrument. Other types of open
physical injuries (Article 365 in
Revised Penal Code) wounds include abrasions,
- careless, reckless, inexcusable, not incisions, and lacerated wounds
justified
LEGAL MEDICINE
SEMESTRAL EXAM
22. What manner of death need not be
autopsied? death from Chronic
Rationale:
illness
The following are the situations
Rationale: wherein the wound of entrance bigger
For individuals who died from natural than the caliber:
causes, autopsies need not to be in contact or near fire, deformity of
performed. If one is performed, it is the bullet which entered, bullet
non-official/non medico legal might have entered the skin sidewise,
autopsy. and acute angular
approach of the bullet

23. Factors which make the wound of 26. In which of the following instances
entrance bigger than the caliber, can you invoke your right against
except: Contact self-incrimination: submission of
semen specimen
Rationale:
The following are the situations
Rationale:
wherein the wound of entrance bigger
than the caliber: Self-incrimination - individual right
in contact or near fire, deformity of not to implicate ones' self in a
the bullet which entered, bullet criminal offense
might have entered the skin sidewise,
and acute angular
approach of the bullet
27. Immediately after hearing a loud shot
coming from the bedroom, the
parents after forcibly opening the
24. Permission for cremation will not be
door, found their 25-year-old son
granted of which of the situation? the
dead with a gunshot wound at the
identity is not yet establish
right temple and with the butt of the
fatal gun held tightly on the palm of
Rationale:
the right hand. Such condition of the
Three conditions in which permission
hand is known as: cadaveric
for cremation is not granted includes
spasm
if the
deceased left a note, identity of the
person is not definite, and the exact Rationale:
cause of death cannot be ascertained Cadaveric Spasm
and the need for o Instantaneous rigidity of the
further inquiry or examination. muscles which occurs at the moment
of death due to extreme nervous
25. Instances when the size of the tension, exhaustion and injury to the
entrance wound is bigger than the nervous system
o Medicolegal use: findings of
caliber of the firearm used, EXCEPT:
weapons, hair and other substances
fragmentation of the bullet may indicate whether suicide,
homicide or murder
LEGAL MEDICINE
SEMESTRAL EXAM
28. To find out the truth is an essential cadaveric rigidity, or death stiffening,
requirement for the administration of or “death struggle of muscles” or
justice. Which of the following is the rigor mortis:- Three to six hours after
commonly used method of deception- death the muscles gradually stiffen. It
detection: lie detection method usually starts at the muscles of the
neck and lower jaw and spreads
Rationale: downwards to the chest arms and
DECEPTION DETECTION lower limb. Usually the whole body
1. Devices that record the psycho- becomes stiff after twelve hours.
physiological responses Polygraph/lie Chemically, there is an increase of
detector machine (25% error) lactic acid and phosphoric content of
2. Drugs that try to “inhibit the the muscle. The reaction becomes
inhibitor” acidic
3. Hypnotism …alteration of
consciousness and concentration in
which the subject manifests
heightened suggestibility
4. Observation 31. Subpoena duces tecum: requires a
5. Scientific Interrogation person to appear in court and
6. Confession … expressed produce documents or papers
acknowledgement by the accused in in his possession
a criminal case of the truth of his guilt
as to the crime charged Rationale:
Subpoena duces tecum - a process
which requires a person to produce at
29. What instance shall an autopsy be the trial some documents or papers
performed on a corpse? All of the which are under his control or
above possession that are pertinent to the
issues of his controversy, at a certain
date, time and place.
Rationale:
a. Upon written request of police
authorities 32. Wrestling: Mat Burns
b. Upon written request of the next of
kin Rationale:
c. Upon order of a competent court, Facial injuries and mat burns due to
mayor or provincial or city contact of the face to the
prosecutor. floor.

30. The whole body becomes rigid due 33. Loreta B. was a med student who was
to the contraction of the muscles engaged to Totoy M. When they
appearing three to six hours after reached clerkship, Totoy decided to
death: rigor dump Loreta for her friend M.
Aagaw. Enraged Loreta cut off the
Rationale: penis of Totoy. She is
Stage of postmortem rigidity, or
LEGAL MEDICINE
SEMESTRAL EXAM
guilty of: Mutilation a result of the injection, the patient
dies. Dr. K is prosecuted by the state.
Rationale: He is most probably guilty of:
Mutilation Murder
Act of looping or cutting off ANY
part or parts of the living body is the Rationale:
intentional cutting or chopping of an
organ.
No intention to kill. The following would fall under the
criteria of murder. The doctor played
an active participation in the death of
a person and with explicit intent to
end his life.
34. The following are classified as open
wounds EXCEPT:petechiae
36. Instantaneous rigidity of the muscles
Rationale: which occurs at the moment of death
due to extreme nervous tension,
Open wounds: when there is exhaustion and injury to the nervous
communication with the outside or system: Cadaveric spasm
break in the skin or mucous
membrane Rationale:
Cadaveric Spasm – instantaneous
1. Abrasion or scratch rigidity of the muscles that occurs at
the moment of death due to extreme
tension, exhaustion or injury to
2. Incised wound the nervous system or injury to the
chest. In cases of cadaveric spasm, a
3. Lacerated wound weapon may be held in the hand
before death and can only be
4. Punctured wound (stab wound) removed with difficulty.

37. To find out the truth is an essential


5. Wound produced by powder requirement for the administration of
explosion (gun shot wounds) justice. Which of the following is the
commonly used method of deception-
6. Mutilation detection: lie-detection method

7. Avulsion Rationale:
Methods of deception detection
which are currently being used or
applied by law enforcement
35. Dr. K decided that his patient had agencies may be classified as
suffered enough from the pain of
bronchogenic carcinoma. He decided
to end his suffering by giving him 100
mg of morphine. As
LEGAL MEDICINE
SEMESTRAL EXAM
follows: supply: Asphyxia
1. Devices which record the
psycho-physiological response: Rationale:
a. Use of a polygraph or a lie Asphyxia – absence/loss of O2
detector machine x In Greek means “absence pulse or
b. Use of the word association test hartbeat”
c. Use of the psychological stress x “suffocation”; death due to
evaluator anoxia/hypoxia
x Body deprived of O2 while in
excess of CO2 (i.e. hypoxia and
38. Mutilation: Cutting ear
hypercapnea)
Rationale:
Mutilation – is the
act or looping or 41. The following are classified as closed
cutting off any part wounds EXCEPT: perforating
or parts of the living wound
body
a. Intentionally Rationale:
depriving a person
totally or partially of
Close wounds: when there is no
some of the essential
breach of continuity of the skin or
organs for
mucous membrane
reproduction
b. Intentionally
depriving a person of 1. Superficial
any part of the body
other than the organs a. Petechiae
of
reproduction
b. Contusion or bruise

c. Hematoma or blood cyst

39. Special type defense wound 2. Deep


of injury that
may favor
homicide a. Deep
rather than
murder b. Dislocation
Rationale:
c. Simple fracture
Special type of injury that may favor homicide
rather than murder :defense wound
d. Rupture of organs

40. Death by interference with O2


LEGAL MEDICINE
SEMESTRAL EXAM
e. Concussion the eyes become staring and the
pupils are dilated. Examination of the
f. Internal hemorrhage visceral organs shows small petechial
hemorrhages, commonly known as
42. The different methods of judicial Tardieu Spots.
death are the following, EXCEPT:
Death by mercy killing or
euthanasia 45. A young man was stabbed in the
abdomen developed generalized
peritonitis and died. Peritonitis is the:
Rationale: proximal cause

Judicial deaths – Art. III Sec.1 Par. 19 Rationale:


Phil. Const. “cruel and unusual The stab wound is the primary cause
punishment shall not be inflicted. ; of death. The peritonitis is the
electrocution, hanging, musketry, gas secondary cayse of
chamber. death, which is defined as the injury
or disease that was survived for a
longer period of time.
43. The medical evidence known or
addressed to the senses of the court
46. As a witness in court, a physician may
not limited to the sense of vision, but refuse to answer a question
it is intended to that of hearing taste propounded to him if his answer: will
smell and touch - autoptic or real be self-incriminatory
evidence
Rationale:
Rationale:
Autoptic or Real Evidence RIGHT AGAINST
Made known or addressed to the SELF-INCRIMINATION
senses of the Court NOT limited to
that which is known through the sense • Sec. 17. Art. III, Philippine
of vision but is extended to what the Constitution states that, “No person
sense of hearing, taste, smell, and shall be compelled to witness against
touch is perceived himself.”The right is available not
only in criminal prosecutions but also
in all other government proceedings,
including civil actions and
44. Tardieu Spots: Convulsive phase administrative or legislative
investigations. It may be claimed not
Rationale: only by the accused but also by any
Convulsive Phase: witness to whom a question calling
This is due to the stimulation of the for an incriminating answer is
central nervous system by carbon addressed.
dioxide. The cyanosis becomes more
pronounced and
LEGAL MEDICINE
SEMESTRAL EXAM
47. This is the deliberate and painless
acceleration of death of a person
Rationale:
usually suffering from an incurable
and distressing illness: Stage of secondary flaccidity
Euthanasia - Muscle are no longer responding to
mechanical or electrical stimulus
Rationale:

Euthanasia also known as mercy


killing is the acceleration of death of a
person who is suffering from a
terminal illness.

48. The principle of Stare Decisis states


that: Only the Supreme Court
can revise, modify or affirm its
own decision

Rationale:

The principle of stare decisis


mandates lower courts to follow legal
precedents set forth by the Supreme
Court, which may at times reverse
itself.

49. While working as a warehouseman in


bocaue bulacan mang inyong was
charge set fire lower extremities
flexed and hands clenched :
pugilistic attitude

Rationale:
Pugilistic attitude refers to the classic
flexion of the upper extremities as the
body
cools.

50. The stage of death where in muscles


become unresponsive to electrical
stimuli: secondary flaccid
o The judge was not present when the incident took place,
therefore, he did not see for himself how the incident in
question occurred
INTRODUCTION, LICENSURE, PRACTICE OF MEDICINE
o If the judge rather is present during the incident in question,
Lecturer: Atty. Antonio Alejandro D. Rebosa, M.D., BSCrim, he/she will have to inhibit himself/herself  presence of
ACLM bias (the judge should always be objective)
 It often undermines the pursuit of truth, as the opposing parties
Medical Jurisprudence seek to win at all cost without the obligation to reveal the facts
 Licensure and regulatory laws which may be detrimental to the case
 Physician-patient-hospital relationship together with the other  The lawyer aims to win the fight not to help the court discover
paramedical personnel, their rights, duties and obligations facts or establish the truth
 Liabilities for non-compliance with the law
If you don’t follow the law, there will be Sources of Law
sanctions/punishments in the form of liabilities  Constitution
o Criminal: the patient wants you behind bars  Laws enacted by the legislative body
o Civil: they want your body Three branches of the government
o Administrative: they want your license o Legislative: composed of the House of Representatives and
For criminal and civil cases, this could be due to a violation of the Senate; they are the so-called “law-makers”
the law o Executive: composed of the Office of the President down to
For administrative cases, it is more for of a violation of the Code the different bureaus and departments; they execute and
of Ethics implement the law
Code of Ethics is not the same with Bioethics o Judiciary: composed of the Supreme Court; they interpret
o Bioethics focuses more on religious matters, dogmas, and apply the law
principles in respect to the issues of life  Decrees, orders, proclamation, letters, CA, BP, RA
Code of Ethics: 2 parts The revised Penal Code is taken from the Spanish
o Standard of care: the physician must be up-to-date with the “Código Penal”, which took effect on January 1, 1932
recent developments in the medical field  All other laws that came after the revised Penal
o Medical etiquette: since physicians belong to a noble Code are called “special laws”
profession, you must be upright, sober and diligent From 1935 to 1941, we were granted Commonwealth status by
 Deals with behavior and how you should conduct the United States  Commonwealth Acts
yourself
December 8, 1941: the Japanese, after bombing Pearl Harbor,
went to the Philippines to invade us
Purposes  Under the rule of the Japanese Imperial Army from
 To protect the public from charlatans 1942 to 1945
 To promote professionalism and foster professional  Liberated by the Americans and granted
interrelationship independence on July 4, 1946
 To develop awareness of the rights, duties and obligations of the  Laws that were passed by Congress were called
patient, physician and the hospital “Republic Acts”
 To control the increasing number of medical malpractice suits September 21, 1972: President Marcos proclaimed Martial
against physicians Law and was considered a dictator because he abolished the
 To explain the purpose and procedure of certain legislation Congress
 To study the need to amend, repeal our health care laws  He abrogated unto himself law-making powers 
inharmony with the recent scientific and social development Presidential Decrees
In 1978, we had the Batasang Pambansa, a unicameral
Adversarial Trial System legislative body
 The Philippine court is a court litigation where there is  The members call themselves “assemblymen”
competition of inconsistent version of facts and theories in law  “Batas Pambansa (BP)”
during trial  The most recognizable of which is BP 22 
Other countries make use of a “jury” system Bouncing Checks Law (estafa)
o Consists of 12 to 15 members of the jury February, 1986: President Corazon Aquino came into power
o The jury will decide whether the accused is guilty or not and the Marcoses were forced into exile
o The judge will only give the sentence as he/she decides  She, herself, was also considered a dictator because
which evidence is admissible in court she abolished the Batasang Pambansa
o Members of the jury are usually non-lawyers  Law-making powers were under her control
 Each party to the contest is given equal opportunity to  The laws that she signed were called Presidential
investigate the case, gather and present all proofs in support of Orders, Proclamations, Letters of Instructions
his/her allegation and give argument that his contention is and the likes
correct February, 1987: after Cory Aquino’s revolutionary
 The court is not expected to render a correct judgment government, the 1987 was ratified and in July of 1987, the
 Ultimate purpose is for a just solution Congress again was reconvened

[JC Checked by: Page 1


LEGAL DA

 July, 1987 up to the present time, all laws passed


by Congress were again called “Republic Acts” Eminent domain: refers to the inherent right of the state to
condemn private property to public use upon payment of
 Administrative acts, orders, Rules and Regulations
just compensation.
 Local customs
Police power: refers to the inherent right of the state to
In the Philippines, we have written, statutory laws as made
enact rules and regulations in order to protect the general
by Congress welfare of the people
In Europe, their laws are through customs  The state has the right to take away your license for
Common law = customs just and valid reasons
“Common law husband/wife”: a couple living in as husband
and wife, but with no blessing from the state Licensure and Regulatory Laws
 without the benefit of marriage  Administrative Bodies
As long as customs are not patently unlawful, illegal or  Board of Medical Education: primarily concerned with the
against public policy may ripen into law standardization and regulation of medical education
 Generally accepted principles of international law The Commission on Higher Education (CHED) now stands
as the board of medical education
 Professional Regulations Commissions (PRC)
Principle of Checks and Balances
Which of these branches is considered to be the most o To have general supervision and regulation of all
powerful? professions requiring examinations which includes the
o None, all 3 branches of government are co-equal practice of medicine
o No one is more powerful than the other because of the PD 223, June 22, 1973: Creating the Professional
principle of checks and balances Regulation Commission and prescribing its powers
The legislative branch makes laws, but the President in the and functions
executive branch can veto those laws with a Presidential Veto  Does not have jurisdiction over the practice of law
The legislative branch makes laws, but the judicial branch can Supreme Court gives the Bar exams and gives
declare those laws unconstitutional administrative complaints filed against lawyers
The executive branch, through the Federal agencies, has  Board of Medicine
responsibility for day-to-day enforcement and administration of o Its primary duties are to give examinations for the
Federal laws registration of physicians and supervision, control and
o These Federal departments and agencies have missions and regulation of the practice of medicine
responsibilities that vary widely, from environmental Acts as a jury: decides on administrative complaints filed
protection to protecting the Nation’s borders against doctors
The President in the executive branch can veto a law, but the Administrative complaints = violation on the provisions of
legislative branch can override that veto with enough votes the Code of Ethics
The legislative branch has the power to approve Presidential In criminal and civil cases, administrative cases filed at the
nominations, control the budget, and can impeach the President PRC is decided by 6 doctors (majority rule)
and remove him or her from office
The executive branch can declare Executive Orders, which are Professional Regulations Commissions
like proclamations that carry the force of law, but the judicial
Compositi
branch can declare those acts unconstitutional  One (1) Commissioner
The judicial branch interprets laws, but the President nominates  Two (2) Associate Commissioners
Supreme Court justices, court of appeals judges, and district
court judges who make the evaluations  Exercise general administrative, executive and policy- making
The judicial branch interprets laws, but the Senate in the Powers,
functions Functions
for the whole agencyand Responsibilities
legislative branch confirms the President’s nominations for
judicial positions, and Congress can impeach any of those judges Board of Medicine
and remove them from office
Compositi
Law and the Practice of Medicine  Six members appointed by the president from a list submitted by
 The State must maintain high standard of practice by setting up the Executive Council of the Philippine Medical Association
rules and regulations with regards to qualifications and (PMA)
procedures for the admission to the profession Membership from the PMA is not mandatory However, a
 These are legal safeguards to guarantee the safety of the patient certain privilege of being a member of the PMA is
and impose liability to the practitioner who through his act or PhilHealth accreditation
omission causes damage or injury to the health and welfare of Philippine Health Insurance (PhilHealth): a socialized
the patient medical insurance program, in which the State pays for a
 The right to regulate the practice of medicine is based on the portion of the hospital bill and the professional fees of the
police power of the state doctor
A State cannot function without the so-called inherent If the doctor is a member of the PMA, he/she can be
powers, namely: taxation, eminent domain and police accredited by PhilHealth
power Unlike in the practice of law, a lawyer must be a member
Taxation: refers to the power of the State to force its of the Integrated Bar of the Philippines (IBP) in good
inhabitants to pay money and punish those who refuse to standing
pay taxes with imprisonment or seizure of property If a lawyer is not a member of the IBP in good standing, if
he gets suspended by the IBP, he may also be suspended in
the practice of law

[JC Checked by: Page 2


LEGAL DA

In the PMA, if a doctor gets suspended, he/she is not o Of good moral character
suspended in the practice of the PRC because their license I o Have completed the first two years of the medical course
s with the PRC  Final or Complete Examination
o Citizen of the Philippines or of any country who has
Qualifications submitted competent and conclusive documentary evidence
 Natural-born citizen confirmed of the Philippines to practice medicine under the
 Duly-registered physician same rules and regulations governing citizens thereof
 In the practice of medicine for at least 10 years (RECIPROCITY RULE)
 Of good moral character and of recognized standing in the When board exams are cancelled, they are to be given on the
medical profession as certified by PMA next schedule of the board exams (given twice a year)
 Not a member of any faculty of any medical school (including o Reset = to be given in the succeeding month
any pecuniary interest) If an individual flunks the board exams for 3 times, he/she shall
Pecuniary interests include being an owner or stockholder be sent back to clerkship
of any medical as well o However, during the pandemic, only a refresher course is
The physician does not have to resign from his post but offered
rather, he/she must take the appropriate leave of absence  Holder of certificate of registration
o No issuance to any candidate who has been
 Convicted by a court of competent jurisdiction of any
Powers, Functions and Responsibilities crime involving moral turpitude
 To determine and prepare the contents of the licensure  Found guilty of immoral or dishonorable conduct
examinations after investigation by the Board of Medicine
 To promulgate such rules and regulations for the proper  Declared to be of unsound mind
conduct of the examinations, correction and registration
 To administer oath Mandatory Malpractice Insurance
 To study the conditions affecting the practice of medicine Why are we against this?
 To investigate violations, issue summons, subpoena and It will encourage more patients to sue doctors, to the point that
subpoena duces tecum even your own brother may sue you
The board cannot cite an individual for contempt and If we will have a mandatory malpractice insurance, medical
sentence you to a jail term practice will not be regulated by the PRC anymore, rather it will
Rather, the board would have to go to court and have the be now dependent on the insurance company
individual punished o No insurance policy/no coverage = no work
 To conduct hearings or investigations of administrative cases In the United States, once a complaint is filed against you, it will
filed before them be available for public viewing on social media platforms and
 To promulgate decisions on such administrative cases subject to the Internet
the review of the Commission Also, in the US, it’s okay to have a malpractice insurance
 To issue certificate of registration because a mere complaint from a patient would suffice for the
 To suspend, revoke or reissue certificate of registration for insurance company to settle  it will not go to court
causes provided by law or by the rules and regulations In the Philippines, you will get an insurance for malpractice,
promulgated however, you will also get a demand letter from the patient’s
When your license is revoked, you can still apply for a
lawyer  ask the insurance to settle for it  the insurance
reinstatement after a period of 2 years
company cannot pay at the moment, as there should be a filed
Must show proof to the PRC that change has been made by
complaint first before the insurance company covers for you
the individual, free of disgrace from the medical profession
o This encourages the patient to sue you
 To promulgate, with the approval of the PRC, rules and
o The physician in question must also admit that he is
regulations in harmony with the provisions of the Medical Act of
“negligent” for the insurance company to settle for the filed
1959 and necessary for the proper practice of medicine
complaint  only the civil liability will be settled
Admission to the Practice of Medicine  clinical aspect is yet to be settled  subject for the
judge’s approval (may actually send you to jail for
Prerequisites admitting negligence)
 Minimum age requirement
o At least 21 years of age Contingent fee: you are paid upon the outcome of the treatment
 Proper Educational Background  unethical in the field of medical practice  you are not
o Requirement for Admission in the College of Medicine supposed to wager with unforeseen circumstances
o Holder of a Bachelor’s degree o Perfectly allowable in the practice of law
o Not convicted of any crime involving moral turpitude Never take indigent patients for granted  always be cordial
You may still take the boards unless proven guilty,
however you may not take the oath with a pending Scope of Examinations
criminal case  Preliminary Examinations
 Examination Requirements o Anatomy and Histology
o Must have passed the corresponding Board o Physiology
Examination o Biochemistry
o Microbiology and Parasitology
 Preliminary Examination
o At least 19 years of age  Final Examinations

[JC Checked by: Page 3


LEGAL DA

o Pharmacology and Therapeutics


diagnose, treat, operate, or prescribe any remedy for human
o Pathology
disease, injury, deformity, psychical, mental or psychical
o Internal Medicine
o Obstetrics and Gynecology condition;
o Pediatrics and Nutrition c) who shall falsely use the title of M.D. after his name,
o Surgery and Ophthalmology shall be considered as engaged in the practice of
o Otorhinolaryngology medicine;
o Preventive Medicine and Public Health CHED memorandum  putting an MD after your
o Legal Medicine, Ethics and Medical Jurisprudence name without passing the board exams is considered
illegal practice of medicine
Practice of Medicine Position paper: “M.D.” is an academic
 What is the practice of medicine? achievement, whether you passed the board exams
 Pursuant to Sec. 10, Art. III of the Medical Act of 1959 as or not, you are entitled to use the M.D. after your
amended: name or be called a doctor
 It is a privilege or franchise granted by the State to any person to
Punishment for illegal practice of medicine  a
perform medical acts upon compliance with law, that is, the
Medical Act of 1959, as amended, which has been promulgated fine between P1,000-P10,000, a prison term
by the State in the exercise of police power to protect its between 1 to 5 years or both
citizenry from unqualified practitioners of medicine Unwritten rule: if a doctor offers his professional fee for
o The practice of medicine is a privilege, not a right free, you must give a gift worth at least 10% of his
o It is granted by the State by virtue of police power professional fee
 It is diagnosing and applying the usage of medicine and drugs The original program for Medicine is 5 years:
for curing, mitigating or relieving bodily disease or conditions o The first 3 years is didactic (classroom discussion)
o The 4th year is half classroom, half clinical
DECS vs San o The 5th year is purely clinical
San Diego, private respondent graduate of UE (Zoology), took o In the 5-year program, you pay for the clinical, but the
NMAT 3 times and flunked 3 times. advantage of that is the school will look for a hospital for
The fourth time he will take NMAT, he was rejected by DECS you
and DCEM for the rule that: o In 1970  Medicine became a 4-year program
o “A student shall be allowed only three (3) chances to take o Early 2000’s  they wanted for the reinstitution of the 5-
the NMAT. After three (3) successive failures, a student year program
shall not be allowed to take the NMAT for the fourth time”.  Dr Rebosa: “I went against it, because why fix
By agreement of the parties, herein defendant was allowed to something that isn’t broken?”
take the NMAT scheduled on April 16, 1989, subject to the  Against the fact that students will be paying for
outcome of his petition. their clinical year
In an amended petition filed with leave of court, San Diego  By DECISIONS OF COURTS are not considered to
squarely challenged the validity and constitutionality of the
constitute practice of medicine:
“NMAT Law” (MECS Order No. 12, Series of 1972),
o One who takes BP reading
containing the above-cited rule. The additional grounds raised
were due process and equal protection. o Application of medicated massage;
o Hospital;
Respondent Judge Teresita Dizon-Capulong ruled that the
MECS Order No. 12, Series of 1972 was valid and held that the o The hospital is not a natural person like us
petitioner had been deprived of his right to pursue a medical o Nurse anesthetist
education through an arbitrary exercise of the police power.
“SAN DIEGO BOBO KA, ‘DI KA PWEDE KA MAG-  By PROVISIONS OF LAW are not considered to constitute
Exemption
MEDICINE PAPATAYIN MO KAMI EHH” (Rebosa, practice of medicine (Sec 11, Art. III, Medical Act of 1959 as
2021) amended):
o Any medical student duly enrolled in an approved
Acts Constituting the Practice of Medicine medical college;
Pursuant to Sec. 10, Art. III of the Medical Act of 1959 as o Dentist;
amended: o Physiotherapist;
a) who shall for compensation, fee, reward in any form paid o Optometrist;
to him directly or through another, or even without the o Any person who renders any service gratuitously in
same, physically examine any person, and diagnose, treat, cases of emergency or in places where the services of a
operate or prescribe any remedy for human disease, injury, physician, nurse or midwife are not available;
deformity, physical, mental, psychical condition or any o Any person who administers or recommends any
ailment, real or imaginary, regardless of the nature of the household remedy as per classification of existing
remedy or treatment administered, prescribed or Pharmacy Laws;
recommended; o Clinical psychologist with the prescription and direct
b) who shall by means of signs, cards, advertisement, supervision of a physician;
written or printed matter, or through the radio, television
or any other means of communication, either offer or
undertake by any means or method to

[JC Checked by: Page 4


LEGAL DA

Additional
Practicing medicine is a privilege, not a right
When it comes to vehicular accidents, since it is unintentional,
the law encourages settlement (areglo)
Culpa aquiliana and culpa contractual are the two kinds of
civil negligence
Culpa contractual (breach of contract): fault or negligence
incident in the performance of an obligation which already
existed
 E.g., a contract established between a passenger and a
driver to be brought to a certain destination
Culpa aquiliana (quasi-delicts): an act or omission which
causes damages to another, there being fault or negligence
and there being no pre-existing contractual relationship
between parties
Medical professionals = ordinary diligence
Drivers, pilots, helmsman = extraordinary diligence
PRC administers the oath
o The board exams are administered twice a year
o Failure to take the oath after passing the board exams
entails that the individual cannot be granted of his/her
license and cannot practice medicine as well

““Fear can hold you prisoner. Hope can set you free.”

- Stephen King, Shawshank Redemption

[JC Checked by: Page 5


LEGAL MEDICINE
Branch of medicine that deals with the application of medical knowledge to the
purposes of law and in the administration of justice

MEDICAL JURISPRUDENCE
Deals with the aspect of law and legal concepts in relation with the practice of medicine

MEDICAL JURISPRUDENCE includes:


 Licensure and regulatory laws;
 Physician-patient-hospital relationship together with the other paramedical personnel,
their rights, duties and obligations;
 Liabilities for non-compliance with the law.

PURPOSES
 To protect the public from charlatans;
 To promote professionalism and foster professional interrelationship;
 To develop awareness of the rights, duties and obligations of the patient, physician, and
the hospital;
 To control the increasing number of medical malpractice suits against physicians;
 To explain the purpose and procedure of certain legislation;
 To study the need to amend, repeal our health care laws in harmony with the recent
scientific and social development.

ADVERSARIAL TRIAL SYSTEM


 Philippine courts is a court litigation where there is competition of inconsistent version
of facts and theories in law during trial;
 Each party to the contest is given equal opportunity to investigate the case, gather and
present all proofs in support of his allegation, and give argument that his contention is
correct ;
 Ultimate purpose is for a just solution.
 “. it often undermines the pursuit of truth as the opposing parties seek to win at all
cost without the obligation to reveal the facts which may be detrimental to their case.
The lawyer aims to win the fight not to help the court discover facts or establish the
truth.”

SOURCES OF LAW
 Constitution
 Laws enacted by the legislative body
 Decrees, Orders, Proclamation, Letters, CA, BP, RA
 Administrative acts, orders, Rules and Regulation
 Local customs
 Generally accepted principles of International law

RMK
LAW AND THE PRACTICE OF MEDICINE
 The State must maintain high standard of practice by setting up rules and regulations
with regards to
qualifications and procedure for the admission to the profession. These are legal safeguards to
guarantee the safety of the patient and impose liability to the practitioner who through his act
or omission causes damage or injury to the health and welfare of the patient.
 The right to regulate the practice of medicine is based on the police power of the state.

LICENSURE AND REGULATORY LAWS


 ADMINISTRATIVE BODIES
 BOARD OF MEDICAL EDUCATION
o Primarily concerned with the standardization and regulation of medical
education
 PROFESSIONAL REGULATIONS
o Its primary duties are to give examinations for the registration of physicians and
supervision, control and regulation of the practice of medicine

BOARD OF MEDICAL EDUCATION


 Composition:
 Chairman - Secretary of Education
 Members - Secretary of Health
 Director, Bureau of Private Schools
 Chairman, Board of Medicine
 Representative, PMA
 Council of Deans, APMC
 Dean, UP-College of Medicine
 Functions:
o To determine and prescribe the requirements for admission into a recognized
college of Medicine;
o To determine and prescribe requirements for the minimum physical facilities;
o To determine and prescribe the minimum number and qualifications of teaching
personnel;
o To determine and prescribe the minimum required curriculum;
o To authorize the implementation of experimental curriculum;
o To accept applications for admission to a medical school;
o To select, determine and approve hospitals for training;
o To promulgate, prescribe and enforce the necessary rules and regulations.

PROFESSIONAL REGULATIONS COMMISSION


 Composition:
o Commissioner
o Two Associate Commissioner
 Exercise of Power and Functions of the Commission: exercise general administrative,
executive and policy-making functions for the whole agency

RMK
BOARD OF MEDICINE
 Composition:
 Six members appointed by the president from a
 list submitted by the Executive Council of the PMA.
 Qualifications:
o Natural-born citizen;
o Duly-registered physician;
o In the practice of medicine for at least 10 years;
o Of good moral character and of recognized standing in the medical profession as
certified by PMA;
o Not a member of any faculty of any medical school (including any
pecuniary interest). Powers, Functions and Responsibilities:
 To determine and prepare the contents of the licensure examinations;
 To promulgate such rules and regulations for the proper conduct of the examinations,
correction and registration;
 To administer oath;
 To study the conditions affecting the practice of medicine;
 To investigate violations, issue summons, subpoena and subpoena duces tecum;
 To conduct hearings or investigations of administrative cases filed before them;
 To promulgate decisions on such administrative cases subject to the review of the
Commission;
 To issue certificate of registration;
 To suspend, revoke or reissue certificate of registration for causes provided by law or by
the rules and regulations promulgated;
 To promulgate, with the approval of PRC, rules and regulations in harmony with the
provisions of the Medical Act of 1959 and necessary for the proper practice of medicine.

ADMISSION TO THE PRACTICE OF MEDICINE Prerequisites:


1. Minimum age requirement: at least 21 years of age
2. Proper Educational Background
o Requirements for Admission in the College of Medicine
o Holder of a Bachelor’s degree;
o Not convicted of any crime involving moral turpitude;
o Certificate of Eligibility from the Board of Medical Education;
o Good moral character
3. Examination Requirements: must have passed the corresponding Board Examination
Preliminary Examination
 At least 19 years of age;
 Of good moral character;
 have completed the first two years of the medical course;
o Final or Complete Examination
o Citizen of the Philippines or of any country who has submitted competent and
conclusive documentary evidence confirmed by the DFA showing that his
country’s existing laws permit citizens of the Philippines to practice medicine

RMK
under the same rules and regulations governing citizens thereof (RECIPROCITY
RULE).
4. Holder of certificate of registration
 No issuance to any candidate who has been:
 Convicted by a court of competent jurisdiction of any crime involving moral turpitude;
 found guilty of immoral or dishonorable conduct after investigation by the Board of
Medicine;
 Declared to be of unsound mind.

Scope of Examination:
 Preliminary - Anatomy and Histology, Physiology, Biochemistry, Microbiology and
Parasitology
 Final - Pharmacology and Therapeutics, Pathology, Medicine, Obstetrics and
Gynecology, Pediatrics and Nutrition, Surgery and Ophthalmology, Otorhinolaryngology,
Preventive Medicine and Public Health, Legal Medicine, Ethics and Medical
Jurisprudence

PRACTICE OF MEDICINE
1. What is the “practice of medicine”?
2. It is a privilege or franchise granted by the State to any person to perform medical acts
upon
3. compliance with law, that is, the Medical Act of 1959 as amended which has been
promulgated by the State in the exercise of police power to protect its citizenry from
unqualified practitioners of medicine.
4. It is diagnosing and applying and the usage of medicine and drugs for curing, mitigating,
or relieving bodily disease or conditions.
5. 5. ACTS CONSTITUTING THE PRACTICE OF MEDICINE (pursuant to Sec.10, Art. III of the
Medical Act of 1959 as amended):
A. Who shall for compensation, fee, reward in any form paid to him directly or through
another, or even without the same, physically examine any person, and diagnose,
treat, operate or prescribe any remedy for human disease, injury, deformity,
physical, mental, psychical condition or any ailment, real or imaginary, regardless of
the nature of the remedy or treatment administered, prescribed or recommended;
B. Who shall by means of signs, cards, advertisement, written or printed matter, or
through the radio, television or any other means of communication, either offer or
undertake by any means or method to diagnose, treat, operate, or prescribe any
remedy for human disease, injury, deformity, physical, mental or psychical
condition;
C. Who shall falsely use the title of M.D. after his name, shall be considered as engaged
in the practice of medicine.

RMK
By DECISIONS OF COURTS are not considered to constitute practice of medicine:
1. One who takes bp reading;
2. Application of medicated massage;
3. Hospital;
4. Nurse anesthesist

Exemptions
By PROVISIONS OF LAW are not considered to constitute practice of medicine (Sec.11, Art. III,
Medical Act of 1959 as amended):
a) Any medical student duly enrolled in an approved medical college;
b) dentist;
c) physiotherapist;
d) optometrist;
e) Any person who renders any service gratuitously in cases of emergency or in places
where the services of a physician, nurse or midwife are not available;
f) Any person who administers or recommends any household remedy as per classification
of existing Pharmacy Laws;
g) Clinical psychologist with the prescription and direct supervision of a physician;
h) Prosthetist.

RMK
 Have registered with the PRC and paid their
professional fee;
 Pay the corresponding income tax
Midterm Topics
 Foreign physicians qualified to practice by Reciprocity Rule or
Atty. Antonio Alejandro D. Rebosa, M.D., BSCrim, ACLM by endorsement;
 Medical Students pursuant to Sec. 11(a) and Sec. 12 (d), Art.
Faith Healing
III, Medical Act of 1959 as amended;
 There is nothing in the Medical Act of 1959 exempting it from
 Limited practitioners of medicine
the definition of the acts which constitute practice of medicine
o Those that are governed by specific licensure laws
Faith healing is allowed as long as the healer does not
collect any professional fees  charity
Rationale why artificial persons cannot practice medicine
 Related to constitutional guarantee to religious freedom
o Freedom to believe and freedom to act in accordance with  Cannot be subjected to licensure examinations as required by
one’s belief law;
 Acted in pursuance of his religious belief and with the tenets of For example, MCU Hospital is an artificial person  juridical
the church he professes to, not deemed to be a practice of person  it can own, sell or be sued  however, it cannot take
medicine but a part of his religious freedom the licensure exam
 Practice of medicine m be employed and controlled by
Illegal Practice of Medicine unqualified physicians;
 Practice of medicine by any person not qualified and not duly-  Professional relationship between the patient and the physician
admitted to perform medical acts in compliance with law will be impaired;
Punishable with a fine of P1,000 to P15,000 and a prison term of  Deprivation of free choice of physicians
5 years, or both imprisonment and fine depending on the
discretion of the court WHO Definition of a Physician
 … is a person who after completing his secondary education,
Penalties follows a prescribed course of medicine at a recognized
 Pursuant to Sec. 28, Art. IV, Medical Act of 1959 as amended: university or medical school, at the successful completion of
o Any person found guilty of “illegal practice” shall be which, is legally licensed to practice medicine by the responsible
punished by a fine of not less than one thousand pesos or authorities and is capable of undertaking the prevention,
more than ten thousand pesos with subsidiary imprisonment diagnosis and treatment of human illness by the exercising
in case of insolvency or by imprisonment of not less than independent judgment and without supervision
one year no more than five years, or by both such fine and
imprisonment, in the discretion of the court Physician-Patient Relationship
 Contract: is the meeting of minds between two persons whereby
Qualified to Practice Medicine in the Philippines one binds himself with the respect to the other, to give
 Those who have complied with the prerequisites to the practice something or to render some service (Art. 1305, NCC)
of medicine in accordance with Sec. 8, Art. II, Medical Act of  Nature of the relationship
1959 as amended; o Consensual: based on mutual consent of both parties
 Those who can have limited practice without any certificate of o Fiduciary: based on mutual trust and confidence
registration in accordance with Sec. 12, Art. III, Medical Act of
1959 as amended:  Consent – manifested by the meeting of the offer and the
o Exclusive consultation in specific and definite cases; Requisites
acceptance upon theof a Contractual
thing and the causeRelationship
which are to constitute
o Attached to international bodies to perform certain definite the contract (Art. 1319, NCC)
work in the Philippines;  Object – the subject matter of the contract which is the medical
o Commissioned medical officers stationed in the Philippines service which the patient wants to be rendered to him by his
in their own territorial jurisdiction; physician
o Exchange professors in special branches of medicine;  Cause – is the consideration or the factor that instigated the
o Medical students who have completed the first four years of physician to render the medical service to the patient, which
medical course, graduates of medicine and registered nurses could be remuneratory or an of liberality
who may be given limited and special authorization by the
DOH;  Expressed: explicitly stated orally or in writing
 “Balikbayan” physicians pursuant to PD 541, Allowing Former  Implied:Forms of Physician-Patient
the existence can be inferredRelationship
from the acts of the
Filipino Professionals to Practice Their Respective Professions contracting parties.
in the Philippines o Inferred by law as a matter of reason and justice for their
o Proviso: acts or conduct
 Of good standing prior to their departure and in their It was not reduced into writing nor verbally agreed upon but
adopted country; it can be deduced that both parties that are bound by some
form of contract

[JC Checked by: Page 1


LEGAL DA

Example: an unconscious person brought to the ER


 He must inform the physician of what occurred in the course of
Some Instances where there is no Physician-Patient treatment;
Relationship by DECISIONS OF COURTS  He must cooperate and follow the instructions, orders and
suggestions of the physician
 Pre-employment PE for purposes of determining whether an  He must state whether he understands the contemplated course
applicant is suitable for employment; of action;
 PE for eligibility for insurance;  He must exercise the prudence to be expected of an ordinary
 Physician appointed by court to examine the accused; patient under the same circumstances
 In performing an autopsy;
There is no physician-patient relationship when the Stages of Physician-Patient Relationship
patient is dead even if he is a former patient  Commencement: it is the very time the physician is obliged to
The patient must be ALIVE to establish a physician- comply with the legal duties and obligations to his patient
patient relationship  Termination: it is the time when the duties and obligations by a
 Casual consultation in an unordinary place physician to his patient ceases. The following are some ways of
termination of the relationship:
Psychological Patterns of Physician-Patient Relationship o Recovery of the patient or when the physician considers
 Activity-Passivity Relation that his medical services will no longer be beneficial to the
o No interaction between the physician and patient because patient;
patient is unable to contribute activity. This is characteristic o Withdrawal of the physician provided a) with consent of
in an emergency case when the patient is unconscious the patient, and b) patient is given ample time and notice;
 Guidance-Cooperation Relation o Discharge of the physician by the patient;
o Patient is conscious and suffering from pain, anxiety and o Death of either party;
other distressing symptoms, he seeks help and willing to o Incapacity of the physician;
cooperate. The physician is in a position of trust o Fulfillment of the obligations stipulated in the contract
 Mutual Participation Relation o In emergency cases, when the physician pf choice of the
o It is in the nature of a negotiated agreement between equal patient is already available or when the condition
parties emergency ceases;
o Expiration of the period as stipulated;
o Mutual agreement for its termination
DUTIES and OBLIGATIONS Imposed on the PHYSICIAN
in the Physician-Patient Relationship Rights of Physicians
 He should possess the knowledge and skill of which an average  Inherent Rights
physician is concerned; o To choose patients
o General Practitioners v Specialist o To limit practice of medicine
 He should use such knowledge and skill with ordinary care and o To determine appropriate management procedures
diligence; o To avail of hospital services
o Locality rule: the standard of care is measured by the  Incidental Rights
degree of care in the locality
o Right way while responding to an emergency
o Similar locality rule: diligence is determined when the other
o Right of exemption form execution of instruments
physicians in the locality or similar locality could have
acted the same way o Library to hold certain public/private officers to perform
certain services to compensation right to membership in
o National standard of care: the diligence is determined on
medical societies
what is applicable on a national standard basis While such
standards exist, preferably, a global standard of care must  Constitutional Rights
now be adhered to o Rights Generally Enjoyed by Every Citizen
 He is obliged to exercise the best judgment; o Pursuant to the provisions of Art. III, Bill of Rights and the
Philippine Constitution of 1987
 He has the duty to observe utmost good faith
If you are arrested, you should be read your Miranda
Rights:
Nota bene
“You have the right to remain silent. Anything you
 Physician-patient relationship does not imply guaranty or any say can and will be used against you in a court of
promise that the treatment will be successful law. You have a right to an attorney. If you cannot
 It does not imply any promise or guaranty that the treatment will afford an attorney, one will be appointed for you.”
benefit the patient In all criminal prosecutions, the accused shall be
 It does not imply any promise or guaranty that the treatment will presumed innocent until the contrary is proved,
produce certain results and shall enjoy the right to be heard by himself
 It does not promise or guaranty that the treatment will not harm and counsel, to be informed of the nature and
the patient cause of the accusation against him, to have a
 It does not promise that the physician will not commit errors in speedy, impartial, and public trial, to meet the
an honest way witnesses face to face, and to have compulsory
We are entitled to honest errors but not to gross errors process to secure the attendance of witnesses
and the production of evidence in his behalf.
However, after arraignment, trial may proceed
DUTIES and OBLIGATIONS Imposed on the PATIENT
notwithstanding the absence of the accused
in the Physician-Patient Relationship
 He must give an honest medical history; provided that he has been duly notified and his
failure to appear is unjustifiable.
No involuntary servitude in any form shall exist
except as a punishment for a crime whereof the
party shall have been duly convicted

[JC Checked by: Page 2


LEGAL DA

Right to Choose Patients


Right to Avail of Hospital Services
 Any person who is given right to practice medicine is not  Right to determine the appropriate management procedure
obliged to practice medicine
 Doctrine of Superior Knowledge
 Cannot be compelled to accept professional employment
o The physician has superior knowledge and the patient just
 NB: The law does not give any qualification on the right of the
follows orders or instructions and usually places himself
physician to choose his patient, however, the Code of Medical
under the command and control of the physician
Ethics and RA 6615 provides otherwise in cases of emergency
 Right of way while responding to the call of emergency
o RA 6615: an act requiring government and private hospitals
and clinics to extend medical assistance in emergency cases  Rule 39, Sec. 12, Rules of Court
 Related Provisions Right of Exemption from Execution
 Right to hold certain public and private offices which can only
o Art. II, Sec. 2 Code of Ethics – “…free to choose whom he of Instruments and Library
be filled up by physicians
will serve… always respond to any request for his  Right to perform certain services
assistance in emergency…”
 Right to membership in medical societies
o Art. II, Sec. 3 Code of Ethics – “In cases of emergency,
o Any qualified medical practitioner has the right to become
…. a physician should administer at least first aid treatment
a member of the PMA through one of its component
and then refer to a more qualified and competent society
physician…”
o Membership in a medical society may be voluntary or
It is the physician’s inherent right to choose his involuntary
patient unless in the case of an emergency o “Membership in the PMA is not mandatory, you may still
Emergency: a situation whereby if there’s no practice and renew your license every 3 years without the
immediate medical attendance, it could be life- need to be a part of it”
threatening or could result to serious injuries and  Philippine Medical Care Act of 1969 (RA 6111 as amended)
permanent complications for the patient provides that membership to the PMA is a requirement before a
For example, a patient with acute glaucoma  physician can practice medicine under Medicare
even if it is not life-threatening  emergency  We don’t have Medicare anymore, as it is now replaced by
failure to instill ophthalmic drops or emergency the Philippine Medical Insurance (PhilHealth) which was
care  permanent blindness signed into law on Feb 14, 1995 by President Fidel Ramos
o Sec. 1 RA 6615, 8344, 10932 – “All government and PhilHealth: socialized insurance scheme  the state pays
private hospitals… are required to render immediate for the medical bills and professional fees of the doctor
emergency medical assistance…”
 RA 8344: an act penalizing the refusal of hospitals and Right to Compensation
medical clinics to administer appropriate initial
 Based on the physician-patient contractual relationship;
medical treatment and support in emergency or serious
 Existence of friendship does not imply gratuitous services
cases,
 RA 10932: an act strengthening the anti-hospital  Art IV, Sec. 2, Code of Ethics:
deposit law by increasing the penalties for the refusal o “…should willing render gratuitous service to a colleague,
of hospitals and medical clinics to administer to his wife and minor children or even parents provided, the
appropriate initial medical treatment and support in latter are aged and being supported by the colleague. He
emergency, or serious cases should, however be furnished with the necessary traveling
 *BP 702, otherwise known as "an act prohibiting expenses… this provision shall not apply to physicians who
the demand of deposits or advance payments for are no longer in the active practice…”
the confinement or treatment of patients in o Only the professional fee is free, however transportation,
hospitals and medical clinics in certain cases" medications, instruments and other pertinent paraphernalia
to be used should be compensated
 *BP 702  amended by RA 8344 
 Doctrine of Unjust Enrichment
amended by RA 10932
o No one must enrich himself at the expense of others
If a pregnant patient is denied admission because of
o “Service rendered, service paid”
financial considerations, and should the patient or
baby die, the presumption of guilt is reversed
 Simple Contractual Fee – specifically stating the value of such
 the doctor is guilty unless proven otherwise Kinds
medical service, either of Medical
orally Fees
or in writing
o Sec. 24, No. 12, Medical Act of 1959 – “…although the
 Retainer Fee – measured by the space of time rendered by the
ethical rule obliges a physician to attend to an emergency,
patient
his failure to respond to it may not make him liable if in
o E.g. As a company doctor, you are paid a specific amount
doing so, there is a risk to his life”
monthly or weekly, regardless if you have seen the
employees or not
Right to Limit Medical Practice
 Contingent Fee – depends upon the failure of the treatment
 Field of specialty
instituted
 Private clinic or hospital
In the field of medical practice, this is considered unethical
 Within a political/geographical boundary as opposed to the practice of law
 Certain days of the week/hours of the day
 Certain class of people
 With due regard to dictate of conscience retirement
 Imposed by the public, religion, professional ethics, medical
society, law, contract

[JC Checked by: Page 3


LEGAL DA

 Dichotomous fee (fee splitting) – the physician may require the


 Medical services rendered under a contract of employment
services of a person who may act as an agent to solicit patients,
unless expressly provided otherwise
and the agent will have a share in the medical fee
Dichotomous fee is always considered unethical
Rights of Patients
Medicine is not a business, it is a vocation
 Right to give consent to diagnostic and treatment procedures
o Art. III, Sec 5, Code of Ethics: “… solicitation of patients,
directly or indirectly, through solicitors or agents is The patient is the final arbiter of his/her own body Individuals
unethical have a right to self-determination, to make decisions about
their lives without interference from others (Principle of
 Straight Fee (package deal) – for the amount tendered by the
Autonomy)
patient to the physician, the latter shall be responsible for the
As long as the patient is conscious, coherent, of sound
payment of the hospital bill, lab fees, medicines and other
mind, and of legal age, he alone will have to give consent
incident expenses
Minors cannot give consent unless the procedure is deemed
o This kind of fee is unethical because the amount wagers
with the unforceable contingencies life-saving
For a consent to be valid, it must be given voluntarily
Unethical  compromising quality for profit without being forced or being subjected to coercion The
consent must have been well-informed, including the
Method of Collection of Payment for Medical Services possible risks, benefits, harm and mode of alternative
 Extra-judicial – billing or referral to a bill collection agency treatment (Principle of Informed Consent) The subject
 Judicial methods matter must be legal; if the service to be rendered is an
o Facts to be Proven in Court: abortion  ILLEGAL
o Physician employed is duly qualified and licensed;  Right to religious belief
o The physician has rendered professional service to the In the Philippines, we do not have an official religion The
patient; State cannot impose and recognize one religion as its
o The professional fee demanded is reasonable; official religion
o The person liable for the payment is the defendant Jehovah’s Witness  respect their right of refusal to the
 The obligation to pay devolves on the patient himself provided transfusion of blood
he is of legal age, of sound mind and has the capacity to enter  If you want to contest the decision, you may go to
into a contractual relation court
 If the patient dies or becomes legally incapacitated to pay, Muslim/Iglesia ni Cristo  take note of food preferences
medical fee shall be made from the following persons in order: accepted by their religion
o Spouse; Seventh Day Adventist  avoid secular work and business
o Descendants (must be of legal age) of the nearest degree; during the Sabbath day (Friday evening to Saturday
Illegitimate children cannot give consent evening)
o Ascendants (parents then grandparents) of the nearest  Medical relief and humanitarian work is accepted
degree;  Right of privacy
Paternal grandparents over maternal if there is Right to refrain from unwarranted publicity  the right not
disagreement to be identified
o Brothers and sisters  Right to disclosure of information
o Other blood relatives: uncle or aunt, cousins The patient has the right to know what is wrong with him
o Guardian; however if designated by the court, the guardian In the Philippines, the right to disclosure of information is
will be the first in line more relative rather than absolute
o Directors of establishments, orphanages, teachers For  If the patient has cancer, you must tell the patient
emergency cases, when nobody can give consent for the
(absolute)
patient, this is the judgment call of the attending physician
 Gradual disclosure of information, usually upon the
If there is disagreement, the last resort is the courts of law
family’s request, however you must still tell the
 Nota bene
patient of the diagnosis (relative) as he will be the
o Implied promise to pay the physician by the benefactor of
primary decision-maker regarding treatment
the medical services rendered in emergency cases
 Right to confidential information
Instances where the physician cannot recover professional fees Statutory privilege communication – you are not supposed
to divulge or disclose to third parties without the
 Agreement that the service is gratuitous; authorization of the patient, whatever it is that you have
 In government charity hospitals, health centers and other learned from the patient in the context of a physician-
similar heath units; patient relationship
 Rendered in private charitable institutions if expressly
However, this right is not absolute  subpoena  you are
gratuitous to the indigent patients;
duty-bound to answer the questions, even if against the
 Waiver on the part of the physician; objection of the subject/patient
 Breach of contract;  Right to choose his physician
 When the physician cannot charge the patient pursuant to the Except if the patient is a charity/service case
Code of Ethics;
 Patient can withdraw from the attending physician any time
 Those covered by Philhealth (no balance billing)
 The physician may also withdraw from the patient-physician
o When you sign up for Philhealth, you are signing that contract any time
you will not be asking for any extra professional fees
From the book of Dr. Solis: the requirements would be to
get the patient’s consent and to give the patient enough time
to look for a replacement

[JC Checked by: Page 4


LEGAL DA

Dr. Rebosa: no need to get the patient’s consent as long as


you give him enough time to look for a replacement Requisites of Valid Consent
 Right to treatment  Informed or enlightened consent
Emergency services must be rendered even if the patient  Voluntary
cannot afford treatment  Subject matter must be legal
Includes life-threatening events and cases wherein, if no
medical treatment is given, it may lead to serious  Expressed consent – written or oral
 Forms
Implied consent may of Consent
be deduced from the conduct of the patient
complications and injuries
“No deposit policy”  institute immediate medical Unconscious patient brought to the ER  the physician
procedures until the patient is stable. Once the patient is shouldn’t wait for the patient to regain consciousness
stable, in theory, you may now transfer the patient to a before taking give consent and take appropriate action
government hospital
 Right to refuse necessary treatments  General or blanket consent: signed upon entry to a hospital upon
admission Scope of the Consent
You have the right to live, but you do not have the
right to die  Limited or conditional consent
Euthanasia, in the Philippines, is not yet legal  Non-liability or exculpatory clause
“I shall not hold the medical staff and hospital liable
Right to Give Consent to Diagnostic Procedures for any negligence, incompetence, ignorance or
The patient has the right to avail himself/herself of any imprudence resulting to injury or death”
recommended diagnostic and treatment procedures. Signed by both the patient and relatives (witness)
Any person of legal age and of sound mind may make an
advance written directive for physicians to administer terminal  Awareness and assent
 Informed/Enlightened
Full disclosure Consent
of facts and willingness of the patient to
care when he/she suffers from the terminal phase of a terminal
illness provided that: submit
o He is informed of the medical consequences of his choice;  Quantum of Information necessary to form the basics of a valid
o He releases those involved in his care from any obligation consent
relative to the consequences of his decision; o Nature of his condition;
o His decision will not prejudice public health and safety. o Nature of proposed treatment or procedure;
The Patient has the right to refuse diagnostic and medical o Possible alternative methods;
treatment procedures, provided that the following conditions are o Risks involved;
satisfied; o Chances of success or failure
o The patient is of legal age and is mentally competent;  Consent must be given freely or voluntarily
o The patient is informed of the medical consequences of
his/her refusal;  Patient
o The patient releases those involved in his care from any Persons
If patient whoconsent
is a minor, can give Consent
must be obtained from the
obligation relative to the consequences of his/her decision; parents;
and In the absence of the parents, consent of the grandparents
o The patient's refusal will not jeopardize public health and must be obtained, paternal grandparents having preference;
safety In the absence of patents and grandparents, the eldest
brother or sister, provided, one is of age and not
Bases of Consent disqualified to give consent
 The physician-patient relationship is fiduciary in nature Other persons who may give consent having substitute
Fiduciary: built on trust and respect parental authority
 Patient’s right to self-determination  Subject matter is legal
 Contractual relationship The subject matter or procedure applied to the patient and
which the patient consented must not be that which the law
Purposes penalizes or against public policy
 Nota bene
 To protect the patient from unnecessary/unwarranted procedure
o Consent of a minor is not valid if the procedure will not
applied to him without knowledge
benefit him
 To protect the physician from any consequences for failure to o Expressed refusal of a minor to surgery shall not prevail
comply with legal requirements over the existing emergency
Before surgery, patient must first sign a proforma consent  Doctrine of parens patriae, the court may grant consent for the
form  will not result to the dismissal of case minor
 rather, will result to litigation Parens patriae – the State is our “substitute parent”

Instances when consent is not needed Right to Religious Belief


 In cases of emergency, there is an “implied consent” or the  Art. III, Sec. 5, Philippine Constitution
physician is given “privilege” because he is reasonably entitled o No law shall be made respecting an establishment of
to consent religion or prohibiting the free exercise thereof
 When the law made it compulsory for everyone to submit to the
procedure
o The State may compel you to take a certain vaccine, even
against your will, by way of police power (if the State
chooses to use police power)
o The Supreme Court decides if the State should compel you
to receive the vaccination

[JC Checked by: Page 5


LEGAL DA

 The free exercise and enjoyment of religious pression and the patient by the parents to have the child treated and
worship, without discrimination or preference, shall forever be parents have no right to base it on religious beliefs or any
allowed. other grounds
 No religious test shall be required for the exercise of civil or  When the law provides for treatment, the patient has no right to
political rights refuse treatment
 The social commitment of the physician is to sustain life and
Right of Privacy
relieve suffering. Where the performance of one’s duty conflicts
 Right of Disclosure of Information
with the other, the choice of the patient, or his family or legal
 The physician-patient relationship being fiduciary in nature, the representative if incompetent to act on his own behalf, should
physician is obliged to make full and frank disclosure to the prevail.
patient or any person who may act on his behalf all the pertinent
 In the absence of the patient’s choice or authorized proxy, the
facts relative to his illness
physician must act in the best interest of the patient
 Art. 1339, Civil Code – “…failure to disclose facts, where there
is duty to reveal them, as when the parties are bound by Liabilities of A Physician
confidential relations, constitutes fraud”  Administrative
o Right to practice is temporarily withdrawn from the
Right of Confidential Information physician;
 Statutory Privileged Communication o A valid exercise of the police power of the State;
o Pursuant to the Rules of Court, Rule 130, Sec. 24(c), a o Laws: Medical Act of 1959 as amended including the Code
person authorized to practice medicine, surgery and of Ethics and Rules and Regulations of the PRC;
obstetrics cannot in a civil case without the consent of the o Penalty: reprimand, suspension, to revocation of license
patient, be examined as to any information which he may Reprimand = a stern warning that a repetition of the same
have acquired in attending such patient in a professional offense would be dealt accordingly with severe sanctions
capacity, which information was necessary to enable him to
Not merely a violation of a law, but it goes against
act in that capacity and which would blacken the character
ethical practice
of the patient
Ethics can be divided into 2:
In law  statutory privileged communication  Standard of Care: a physician must keep abreast of
In ethics  ethic/professional confidential information medical innovations and knowledge to keep up with
 Ethical Privileged Communication the standard of care
o Pursuant to Art. II, Sec. 6, Code of Medical Ethics, the medical  Medical Etiquette: appropriate or ethical behavior;
practitioner should guard as a scared trust anything that is upright, sober and diligent at all times
confidential or private in nature that may discover or that may be By following the law, you are usually morally and
communicated to him in his professional relation with his ethically correct – but there are some instances that what
patients, even after death may be legal, may be immoral (and vice versa)
o He should never divulge this confidential information, or  Legal but immoral = contraception (goes against the
anything that may reflect upon the moral character of the person teachings of the Church)
involved, except when it is required in the interest of justice,  Moral but illegal = giving of alms/money
public health and public safety
o Medical malpractice is an administrative liability
Some instances where confidentiality is not applicable  Lawyers often make the mistake of filing
 When such disclosure is necessary to serve the best interest of malpractice under criminal or civil liabilities
justice; o Evidence required in an administrative case = substantial
 When the disclosure will serve public health and safety evidence only
 When the patient waives its confidentiality  Criminal
o An act or omission which constitute a crime by the
Right to Choose Physician physician;
 Right tot treatment o Laws: Revised Penal Code and other special laws;
 In emergency cases the patient has the right to treatment o Penalty: imprisonment and/or fine
 Sec. 1, 1st par, RA 6615 Commission of crimes or criminal offenses as stated in
the revised Penal Code (January 1992) and special
o Provides that “…all government and private hospitals or
criminal laws that provide for imprisonment and fines
clinics duly licensed to operate are required to render
If you are acquitted from your charges, you cannot be
immediate medical assistance to patients in the ER, who are
in danger of dying and or suffered serious physical injuries” tried again for the same facts  reconsideration of
Art. II, Sec. 3 Code of Medical Ethics motion  violative of the motion double jeopardy If you
are convicted, you have the right to appeal  Supreme
o “In cases of emergency wherein immediate action is
Court
necessary, a physician should administer at least 1 st aid
Life imprisonment = an appeal is made automatically
treatment and then refer the patient to a more qualified and
competent physician, if the case does not fall within his Evidence required in a criminal case = guilt beyond
particular line” reasonable doubt
“People of the Philippines v Rebosa” – against the State
Right to Refuse Treatment  Civil
 In the legal sense, every man of adult age and of sound mind has o Awarded against a physician to compensate for the injury he
suffered on account of the physician’s act or omission as a
the right to determine what must be done in his own body. A
breach of the contractual relationship of both parties;
man is the master of his own self and may expressly prohibit a
life-saving surgery or medical treatment
o Doctrine of parens patria – the State has the right to assume
guardianship when the child is neglected by

[JC Checked by: Page 6


LEGAL DA

o Laws: Civil Code of the Philippines and other related


that employer Teodoro Toribio, a member of the National
laws;
Workers Brotherhood, made a false claim that there was a
o Art. 100, RPC states that “every person criminally liable is
civilly liable” shortage of leather soles in Ang Tibay, making it necessary for
him to lay off workers
o Penalty: damages
NLU alleges that such claim was unsupported by the Bureau of
Payment of money or indemnification in the form of
Customs’ records and the accounts of native dealers of
damages
leather. Such was just a scheme adopted to discharge all the
Evidence required in a civil case = preponderance of
members of the NLUE from work. Hence, they say that Teodoro
evidence
was guilty of unfair labor practice for discriminating against
“Rebosa v Rebosa” – person against a person
NLU and unjustly favoring NWB Regarding the exhibits
For laws to be followed, there should be sanctions for
attached to this case, NLU says that they are inaccessible to the
disobeying because without punishment, why would one
respondents that even with the exercise of due diligence, they
follow the law?
could not be expected to have obtained them and offered as
 Sanctions  come in the form of liabilities evidence in the CIR. In addition, the attached documents and
The burden to prove that a certain doctor is guilty lies on exhibits are of such far-reaching importance and effect that their
the patient – we are presumed innocent unless proven admission would necessarily mean the modification and reversal
otherwise of the judgment rendered herein
 Doubt shall always be construed in favor of the accused NLU was entitled to a new trial
Dr. Rebosa’s analogy: “To prove a criminal case, you o The court observed that, except as to the alleged agreement
need a passing grade of 90 in your classes, while a civil between the Ang Tibay and the NWB, the record is barren
case would only need a passing grade of 70. Let’s say all and does not satisfy the thirst for a factual basis to predicate
evidence has been presented, yet all the presented facts a conclusion of law
only equate to a grade of 85, then the doctor is acquitted o Therefore, in the interest of justice, a new trial should
of criminal charges. commence giving the movant the opportunity to present
 However, since a grade of 85 is higher than 70, the new evidence
doctor can be filed for a civil case  the doctor is
acquitted, but pays for moral damages Grounds for Administrative Liabilities
“Passing grade” for administrative liabilities = 50 When  Sec. 24, Art. III, Medical Act of 1959 as amended
you are criminally liable, you are not just imprisoned, but  Personal Disqualifications
you are made to pay for moral damages as well 1. Immoral or dishonorable conduct;
If you are convicted of a crime involving moral turpitude, 2. Insanity;
you will not only go to jail, pay moral damages to the 3. Gross negligence, ignorance or incompetence
complainant, the court will also order the PRC to strip you resulting in an injury to or death of the patient;
off your license
 Violation of ethics  filed via PRC
 Moral turpitude = vileness, baseness or depravity of
character  However, if the case filed is reckless imprudence or
 Rape, murder, carnapping, drugs homicide  criminal
4. Addiction to alcoholic beverages or to any habit- forming
drug rendering him incompetent to practice medicine

 Administrative
Quantum of evidence Liabilities evidence, such
needed: substantial
relevant evidence as a reasonable mind might accept as adequate Criminal Acts
to support a conclusion 1. Conviction by a court of competent jurisdiction of any criminal
 Administrative due process offense involving moral turpitude;
1. Right to hearing; 2. Fraud in the acquisition of the certificate of registration;
2. Tribunal must consider the evidence presented; 3. Performance of or aiding in any criminal abortion
3. Decision must have something to support itself; 4. Knowingly issuing false medical certificates;
4. Evidence must be substantial; Most common committed offense; the punishment for this
5. Decision must be based on the evidence adducted at the act is a term of imprisonment for 6 months to 2 years for
hearing, or at least contained in the record and disclosed to every one (1) false certificate
the parties; Only registered doctors can commit this crime; if a lawyer
6. The Board or its judges must act on it or their independent commits the act, is now considered falsification or fraud
consideration of the facts and the law of the case, and not 5. Aiding or acting as a dummy of an unqualified or unregistered
simply to accept the views of a subordinate in arriving at a person to practice medicine
decision (Ang Tibay vs CIR) 6. Civilly liable to the patients tendered as during the time the
physician acted as a dummy
Ang Tibay v CIR If the crime was committed by a doctor without a license or
an intern, he/she will not be eligible to take the board exam
A motion for reconsideration was filed by the Solicitor- General
Cannot file an administrative case if the doctor is
on behalf of the respondents of CIR, a debt collection law firm
unlicensed  civil and criminal charges only
The National Labor Union on the other hand prays for the
remanding of the case to CIR for a new trial. Sample Case for Due Process
Ang Tibay filed an opposition for both the motion for
During face-to-face classes, while Dr. Rebosa is having a lecture, he
reconsideration of CIR and the motion for a new trial by the
suddenly shoots all of his students. The act was caught on tape via
National Labor Union (NLU)
CCTV. However, he is not sent directly to jail
Toribio Teodoro owns and operates Ang TIbay, a leather
company which supplies the Philippine Army. NLU avers

[JC Checked by: Page 7


LEGAL DA

as he is awarded due process, for he may have a reason/defense on


why he committed the act. Imelda Marcos Embezzlement Case
“Self-defense” or under the suspicion of persecution Mental In the 1990’s, Imelda Marcos, former First Lady of the
Philippines, was acquitted of charges of racketeering and fraud
illness/insanity  exempting circumstance in criminal law
in stealing more than $200 million from the Philippine treasury
(no criminal liability)
and investing most of it in jewels, art and four pieces of prime
However, since he admitted to the committed crime, he is
Manhattan real estate.
now GUILTY unless proven otherwise
It culminated in a three-month trial that tested a novel concept in
American jurisprudence: whether the wife of a foreign head of
Unprofessional Conduct state should face a criminal trial in a United States court.
1. False or extravagant or unethical advertisements wherein other The case also engulfed Adnan M. Khashoggi, a wealthy Saudi
things than his name, profession, limitation of practice, clinic businessman, a co-defendant who was acquitted on charges that
hours, office and home address, are mentioned; he helped Mrs. Marcos conceal her involvement in the
It is legal, but unethical investments.
A doctor cannot be used as an actor wherein medications Imelda Marcos was found not guilty of racketeering, conspiracy,
are advertised  if he advertises for a generic brand of obstruction of justice and mail fraud.
Vitamin C it’s okay, but if he advertises “Enervon C” it is The jurors said they were not convinced that Mrs. Marcos knew
unethical about any wrongdoing, and questioned why the case was tried in
2. Issuing any statement or spreading any news or rumor which is the United States.
derogatory to the character and reputation of another physician o On their very first ballot, in the first moments of
without justification; deliberations, they voted 10 to 2 to acquit her
3. Violation of any of the Code of Ethics as approved by the PMA
"If the glove doesn't fit, you must acquit"
Special Question: Judge v Jury - Johnnie Cochran
The OJ Simpson Trial
The People of the State of California v. Orenthal James
Simpson
O.J. Simpson trial, criminal trial of former college and
professional gridiron football star O.J. Simpson, who was
acquitted in 1995 of the murder of his ex-wife Nicole Brown
Simpson and her friend Ronald Goldman.
The defense team that represented OJ Simpson was called the
“Dream Team”. The lawyers that were part of this team were:
o Robert Shapiro
o Johnnie Cochran
o F. Lee Bailey
o Alan Dershowitz
One of their main arguments was the police mismanagement of
the case. The Defense focused on discrediting the Los Angeles
police department. They claimed that the police failed to
conduct a well-constructed, proper investigation.
Johnnie Cochran addresses the court stating that main police
involved within the case were racist. He argued that police
Detectives tried to frame Mr. Simpson of the crime by planted
crucial evidence like the bloody glove in his residence.
After many months of deliberation and numerous witnesses
within the criminal trial, 12 juries finally came to a verdict. Mr.
Simpson was found not guilty in both counts of murder. Directly
after Mr. Simpson criminal trial concluded, a civil lawsuit was
filed against OJ by the parents of Ronald Goldman.
The outcome of this trial was different from that of the criminal.
The lawyer for the Goldman family, Mr. Petrocelli brought
crucial evidence into this trial which was not involved within the
Criminal trial.
A lie detector test that Mr. Simpson completed after the crime
was examined, which showed how OJ had failed this test.
This test involved questions asking if he had involvement within
the crime.
The jury within this trial awarded the Goldman family and the
Simpson children 12.5 million dollars in compensation.

[JC Checked by: Page 8


(anesthesiologist) will be the one too be sent to prison and to lose his licens
The surgeon will pay; however, he will not go to jail and he will not lose hi

Presem Topics
Lecturer: Dr. Albert D. Rebosa, MD, LLB

Legal Principles and Doctrines


Applied in Medical Malpractice Rogelio Ramos et al., vs Court of Appeals
Legal doctrines enunciated by the Supreme Court are not laws in This was the case of a certain Erlinda Ramos who was scheduled
itself but rather principles or guidelines in a case decided for a cholecystectomy after being referred to the respondent, Dr.
o As part of jurisprudence, they become parts of the law of Hosaka
the land Since neither petitioner Erlinda nor her husband, petitioner
o Serves as a guide when cases of similar facts are to be Rogelio, knew of any anesthesiologist, Dr. Hosaka
decided for the stability of Supreme Court decisions
recommended to them the services of Dr. Gutierrez
(stare decicis)
Dr. Hosaka’s negligence can be found by not determining if his
o Only the Supreme Court can repeal, modify or change legal
doctrines based on latest rulings and decisions anesthesiologist (Dr. Gutierrez) observed proper anesthesia
General rule: only the doer of the act is criminally and protocols which led to the patient entering into a comatose state
administrability liable but someone else can civilly liable o Dr. Hosaka at that time, has also scheduled another
It is up to the parties concerned to invoke the applicability of operation in a different hospital at the same time as the
these doctrines scheduled cholecystectomy  he arrived 3 hours after the
No hard and fast rules – it is up to the judge on which of these scheduled operation
doctrines will apply Since the ill-fated operation, Erlinda remained in comatose
condition until she died on August 3, 1999
Persons Involved in Malpractice Cases Dr. Hosaka was remiss in his professional duties toward his
Person who gets injured – the patient patient, thus sharing equal responsibility for the events
Person responsible for the injuries sustained by the first person –
medical staff and everyone else included in the healthcare team Borrowed Servant Doctrine
Person who is made civilly liable – the hospital and the Servants of the hospital = residents, interns, clerks, nurses,
management pharmacists, medtechs and everybody else who works for the
hospital
Doctrine of Vicarious Liability
Also known as the Doctrine of Imputed Negligence The consultant is not a servant  not an employee of the
Art. 20 of the Civil Code – every person is liable for the hospital  an independent contractor
consequences of his or her own negligence when another person o Only given the privilege to admit patients under his
is injured or died as a result service and to utilize the facilities of the hospital
Physicians are legally liable for the errors of their employees The consultant as an independent contractor can borrow “servants”
when they are acting within the scope of their duties of the hospital
Vicarious liability – transmission or imputation of one’s liability o The borrower becomes the employer and negligent act will
to another one who exercises supervision and control be held liable against the borrower
o The fault of the resident physician trainee will redound to The hospital may escape its liabilities because, temporarily, the
be the fault of the trainer consultant borrower becomes the employer, not the hospital
Doctrine of Respondeat Superior
o “Let the master answer” For example, the attending physician ordered medications that
Employer-employee relationship: the employer is liable for should be given IM however the nurse gave the medications via
the employee IV and the patient dies, the nurse will go to jail and lose his
The responsibility of the employer ceases when he has license
proven that he observed all diligence to prevent damage o If you apply the Doctrine of Vicarious Liability  the
hospital will
Captain of the Ship Doctrine o If you apply the Borrowed Servant Doctrine  the
The surgeon or main attending physician is equated to the attending physician is made liable by the acts made of the
captain of the ship responsible for the safety of its crew This servants he borrowed from the hospital
doctrine refers to an operating room set-up
o The surgeon is accountable for everything that goes Doctrine of Ostensible Agent
wrong within the four corners of the OR
Also known as the Doctrine of Apparent Authority
The hospital is made liable by the acts of the consultants
o The hospital has authority over the consultants so it
should pay for their mistakes
o Consultants = “pseudo-employee”
For example, the anesthesiologist wrongfully gave carbon Under the Doctrine of Ostensible Agent, there are physicians in
dioxide, instead of oxygen, and the patient dies, the doer the hospital who are deemed ostensible

[JC Checked by: Page 1 of


LEGAL DA

agents and therefore, there liability may redound to the liability


of the hospital Some Instances or Applicability of Res Ipsa Loquitur
Persons who are considered ostensible agents: Leaving of foreign objects in the body of a patient after
o Radiologist: if the radiologist interpreted the chest x-ray as operation
benign, but when in fact there is a presence of Wrong laterality such as left enucleation was done instead of
right enucleation
bronchogenic carcinoma  patient thinking he was
Removal of the wrong part of the body
normal went home and dies after a few moments  E lectrical burn s from machines
L ost or thrown specimens
negligence on the part of the radiologist  the W
rong medications given
radiologist goes to jail and loses his license
 By technicality, since the hospital benefits from the Wrong route and dosage of medications
services of the radiologist, despite being a consultant, Wrong blood was transfused
the hospital will have to pay as well
o Pathologist: the pathologist for example interprets a frozen Doctrine of Contributory Negligence
section specimen as malignant  due to this a radical Contributory negligence has been defined as conduct on the part
mastectomy was ordered by the surgeon on both patient’s of the plaintiff, contributing as a legal cause to the harm he has
suffered, which falls below the standard of care When the
breasts  5 days later, the same specimen was read by 3 plaintiff’s own negligence was the immediate and proximate
other pathologists saying that it was benign  negligence cause of injury  he cannot recover from damages
on the part of the surgeon If his negligence is only contributory or due to the defendant’s
 the surgeon goes to jail and loses his license lack of due care  the plaintiff may recover from damages, but
 The pathologist and the hospital will pay for the the court shall mitigate the damages to be awarded
damages by the doctrine of Ostensible agent
 If you are to apply the captain of the ship doctrine, the A 70-year-old male patient about to undergo surgery the
surgeon pays for the damages following day. The surgeon makes his rounds the night before
o House Anesthesiologist the surgery and then asks the patient several questions. He asks
Employed Resident Physicians or “moonlighters” if he has comorbidities and the patient says he is perfectly fine.
Employed Emergency Medicine Consultants House The doctor proceeds with the surgery, however the patient dies
Specialist or Hospitalist intra-operatively. It turns out the patient is diabetic, insulin
dependent and is suffering from multiple comorbidities. The
Doctrine of Res Ipsa Loquitur relatives filed a P5 million case against the doctor.
“The thing speaks for itself” o Most likely the court will award P200,000, because the
The cause of injury under the management of the physician and patient himself contributed to his own demise by not
borrowed servants and in the ordinary course of things, the disclosing his complete medical history
injury caused would not have happened if only proper care was o However, the doctor is still negligent, because he believed
used the patient who is already 70 years old
o The accident was caused by the physician’s want of care o SOP = if the patient is over 40, he is referred to a
Applied when the circumstances of the attendant upon the cardiologist before surgery for risk assessment (CP
injury/death are of such character clearance)
Common knowledge doctrine  there is no more need for an Doctrine of Continuing Negligence
expert witness If the physician, after a prolonged treatment of a patient, which
In 99.5% of cases filed against doctors, the patients that the normally produces improvement of the illness, fails to
doctors have breached the standard of care that the specialty investigate non-response may be held liable if in the exercise of
requires the care and diligence he could have discovered the cause of
Anesthesiologist commits an error  must be enlightened by non-response
an expert witness  another anesthesiologist  most often
than not, the doctor wins the case Patient goes to the doctor, complaining of fever of 5 days
o When res ipsa loquitur is applied, and the deficiencies of duration. The doctor then prescribes Paracetamol. After 5 dies,
the doctor can be proven by common knowledge  the patient comes back. The doctor persisted to continue his
current regimen. After 5 days, the patient comes back and the
expert witness is needed
no Rule doctor switches to Acetaminophen. After another 5 days, the
Bad Result is not res ipsa loquitur
o Bad result affords no presumption of negligence patient dies due to fever of unknown origin and dengue
o However, if the physician fails to remove foreign object o Is the doctor negligent? Yes, he was continuously
from the patient (e.g. sponge or glove used negligent. On the 2nd visit, he should have already requested
intraoperatively), this will create inference of negligence for laboratory tests or antibiotics.
o Also, since we are a tropical country, the first thing which
comes to mind when there is fever should have been
Requisites of Res Ipsa Loquitur infection
The accident is of a kind, which ordinarily does not occur in the
absence of someone’s negligence Doctrine of Last Clear Chance
It is caused by an instrumentality within the exclusive control of The doctrine infers understood, obligation and lapse of sufficient
the defendant or physician time to effectually act upon impulse to save the life or prevent
The possibility of contributing conduct, which would make the injury of the patient
plaintiff or patient responsible is eliminated A physician who has the last clear chance of avoiding injury to
his patient but negligently fails to do so may be held liable

[JC Checked by: Page 2 of


LEGAL DA

Fact: It is unlawful for a pedestrian to be inside the


expressway/freeway. Additional Provisions
If you run over an illegal pedestrian in the expressway, are your The doctor was driving his car when he chanced upon an
criminally liable? Generally, no, but you are civilly liable unconscious patient who was hit by a vehicle. His conscience
tells him that as a doctor, he should help. He
and will pay t, but by doing so, his s
o TPL = third party liability insurance parks his car beside the stree car
bumped from behin d and he sustains injuries and damages
o You cannot register your car without settling the TPL t o his ca r.
When are you criminally liable? Who can he sue?
o Under the Doctrine of Contributory Negligence = the d o He can sue the driver who bumped his car who caused him
river
overhimself
speeding,
is expired license and registration papers, injuries.
dilapidated car, defective brakes/lights, driver with o He can also s ue the first driver, since if it wasn’t for th e
blindness/hearing deficits  the driver will pay accident he caused, the 2nd accident wouldn’t have b
o Under the Doctrine of Last Clear Chance = while driving, een
the driver chanced his way on a pedestrian trying to cross Doctrine of Corporate Negligence
the freeway. The driver intentionally runs over the Pursuant to the Supreme Court ruling in the case of PSI vs
pedestrian to “teach him a lesson”, instead of swerving Agana
o Physicians are deemed employees of the hospital
away from the patient  he h ad a last o The fault of will redound to be the fault of the hospital
c
hance of avoiding the patient, however, he still o Pro hac vice = facts and decisions applicable in this
p t
ursued the case only
Doctrine of Assumption of Risk o Non-doctrinal
When a patient freely consents to a treatment procedure, the This is only a precarious precedent as doctors are deemed to be
inherent risk attached thereto is deemed assumed by the patient employees of the hospital but hospitals do not have actual
provided the physician acted in accordance with sound medical control over physicians in their management and treatment for
standards their patients
The physician is not a guarantor of care nor does he warrant a
good result Sole Responsibility or Shared Responsibility
The act of one is the act of all, unless of one of the co-
o A n egative outcome does not mean it was the result o f
managing physicians withdraws from the patient before the
a n egligent t
incident
For example, Dr. Rebosa refers his patient to Dr. Batar for
COVID vaccines  we are all under the Doctrine of psychiatric clearance. She goes and sees his patient and writes in
Assumption of Risk since all of these vaccines are experimental the chart, “Dear Dr. Tony Rebosa, thank you for the referral. I
except for Pfizer in the United States suggest the following and give these medications”. The nurse
In the Philippines, all vaccines are just for emergency use since then calls Dr. Rebosa to tell him that Dr. Batar already saw the
it takes 10 years for a vaccine to be approved patient. The nurse then carries out Dr. Batar’s orders after
notifying Dr. Rebosa. However, after a few hours, the patient
Doctrine of Foreseeability dies. It turns out that when Dr. Batar was making her rounds, she
If foreseeable complications arise from competent and diligent was drunk.
treatment and care, and the physician did not do anything, he can o Who will go to jail?
be made liable  Dr. Rebosa, because there is n t
Accordingly, the physician cannot be held liable if the injury o physician-
r elationship between the patient and Dr. Batar
was based on unforeseen conditions or complications that may  She is just there to evaluate; she is not free to treat the
arise from treatment or care patient
 The final decision rests with Dr. Rebosa so he is
Patient is unconscious on the stretcher. The patient is being criminally liable
monitored by the intern and the stretcher is in stable conditions. o Who will be civilly liable?
However, the intern left the patient for a few seconds just to get  Dr. Rebosa and Dr. Batar
something 20 ft away. Unfortunately, the patient went into o Who will be administratively liable?
seizure and falls into the ground face-first, sustaining multiple  D r. , because she made her rounds under the
fractures and lacerations influence of alcohol  against the Code of Ethics
What doctrine is applicable here?  She v iolated medical etiquette – being morally
o Doctrine of Foreseeability right and sober at all times
 You should have had the ri ght foresight that
p atient will fall on the stretcher and sustain injuries Other Doctrines
if left untreated The State cannot be sued without its consent
o Res Ipsa Loquitor o Suing is one thing, collecting is another thing
 Patient had a s troke and sustained facial fracture s o If you are run over by a bulldozer owned by the State, the
inside the hospita l  How do you justify that? government is immune from suit, but the driver who is a
government employee, is not immune from suit
Rescue Doctrine or Good Samaritan Law o A full-time government doctor, likewise, is not immune
Under this doctrine, if a physician came to help and was not able from suit
to save the life of the patient, the physician cannot be held liable  You cannot sue Jose Reyes, PGH or East Ave, but the
for it doctors and nurses working in that hospital can be
This doctrine does not constitute an obligation to rescue, but the sued
duty to rescue where it exists may be held liable  Government doctors have more liabilities than those in
o Art. 275 of the Revised Penal Code: abandonment of the private sector
person in danger or abandonment of one’s own victim

[JC Checked by: Page 3 of


LEGAL DA

o Administrative: liabilities of a private doctor  PRC, o A hospital is usually owned by non-doctors


while a public doctor will be under PRC, Ombudsman  If the non-medical owners interfere with the care
and the Civil Service Commission
given by doctors  i llegal practice of medicine
 The Ombudsman and CSC has no
jurisdiction over the private sector
Medical Records
The medical record of a patient is the source of all information
regarding consultation, confinement, treatment and management
P ls: 2Jose
Kinds of or
Reyes Hospitals
East Ave
ublic in a hospital or clinic
Private hospitals: MCU-FDTMF Hospital The medical record is a property of the hospital and contains
o But MCU also has a charity division for service cases confidential information regarding the patient
M
If a service case patient dies, they can still sue MCU,
edical s are not owned by the patient  they are
however, MCU will invoke 4 doctrines for them to be immune only entitled to the medical abstract and the photocopies of their
from being sued (these doctrines are only theoretical and as Dr. diagnostic test results
Rebosa would say it, it will only be asked during the exam): The attending physician does not own the medical records

T rust fund When demanded by the patient or by someone who could


Instances When the Contents of
Dr. Rebosa donates P10 million for the medical treatment
of indigent cases in MCU. Apparently, the hospital just lost the Medical
f, which Record can be
must be made into writing and the
a ct in his
a case and is being ordered to pay P10 million pesos. MCU proper identification
Hospital, however refuses to pay, claiming they don’t have When the l aw compels such disclosur e
Through
money. as ubpoena issued by courts and other agencies
Can the patient ask for the P10 million donated by Dr. of the government
Rebosa to MCU-FDTMF hospital? o Theatie hospital is not obliged to give the records to the p
o No because it is not for payment of debts or satisfaction of o It is only obliged to submit their record if there is a valid
obligations, but rather, entrusted for indigent patients
ourt
P c OH and r or from agencies such as the NBI, PNP, D
ublic
It is the duty of the government to provide healthcare needs to
indigent members of the community Legal Medicine
However, if for example the government doesn’t have money,  A branch of medicine which deals with the application of
the service case patient cannot sue MCU-FDTMF under the medical knowledge to the purposes of law and in the
government’s authority administration of justice
 Legal medicine: a
pplication of medicine to legal cases
In charity
Implied Waivercases, the patient knew very well that he is  Forensic medicine: application of medical science to elucidate
legal problems
subjected to a different routine of care as compared to the 
services of the patients in the hospital’s private/pay division
Medical
practicejurisprudence:
of medicine knowledge of w in relation to the
o Under the care of interns, clerks, medical students and other  Scope: application of medical and paramedical sciences as
instructional purposes
demanded by law and administration of justice
o If the patient dies, they cannot sue the hospital knowing that
in the first place, there is an “implied waiver” of the
aforementioned services and under this kind of care  The ability to acquire facts, arrange them and draw a conclusion
Nature
from facts of the Study of
in the administration ofjustice
Legal Medicine

I ndependent Contractor  Medical jurist, medical examiner, medicolegal officer,


medicolegal expert:
The most unfair for consultants
o A physician who specializes primarily with medico- legal
For having the privilege to admit patients, they also have the duties
duty to train the interns and other medical students
In a charity case, the ones who checks the patient is the resident, o Imhotep – 2980 BC  earliest medico-legal expert
but there is a service case consultant – for example, Dr. Rebosa
o He is in charge of training the residents and he cannot
charge a single centavo Difference Ordinary Physician Medico-Legal
Officer
o If the p atient dies under the resident’s care , the
resident is a e Injury/disease Treatment Cause
dministratively point of view
o However, in the independent contractor theory, the
Examining a To diagnose To testify
attending physician will be civilly liable, because the
contract of giving care is not between the hospital and the patient
patient, but rather, between the patient and the attending Minor injuries Ignored Records all/qualify for
physician crime
The hospital only has 3 duties: Principle of Stare Decicis
o To provide the venue  When the court has once laid an interpretation of law as applied
o To provide personnel to certain facts, it will adhere to and apply to all future cases
o To provide equipment and to ensure that they are safe The where the facts are substantially the same
hospital cannot practice medicine
If it is considered jurisprudence since it has reached the Supreme
o It is not a natural person
Court, the lower courts cannot reverse or modify
o It is not a juridical person
o A hospital cannot be subjected to a licensure exam

[JC Checked by: Page 4 of


LEGAL DA

Basic Principles Governing Subpoena ad testificandum: a paper that requires a person to


Application and Effects of appear and give testimony at a certain time and place,
 “Ignorantia legis nominen excusat”: prevent use as defense in commanding him or her to put aside all pretense and excuses
violation Direct contempt: occurs under the court's own eye and
within its own hearing
Indirect contempt: occurs out of the presence of the
court, thereby requiring the court to rely on the
“Ignorance of the law excuses no one from compliance there with”
testimony of third parties for proof of the offense
 Experimental evidence – if allowed by the court to confirm or
 Law shall have no retro-active effect unless the contrary is corroborate
provided o If you present a theory, since it is not yet proven, you may ask
It is given retro-active effect if the law is curative in nature the court to consider experimental evidence
and it is favorable to the accused  Documentary evidence – writings/photographs
In 2006, the death penalty was abolished. By the time of its o Medical certificates
abolition, there were 3,000 convicts in death row just o Medical expert opinion
waiting for their turns to be executed o Deposition
Since this new law is favorable to them, all their death
sentences were commuted to life
Theoretically, if the death penalty is reinstated by the
administration, they will not be affected since the new
law will not be favorable to them  Physical evidence – articles and materials found in
 Rights may be waived, unless the waiver is contrary to law, connection with the investigation and which aids in establishing
public order, public policy, morals or good customs or the identity of the perpetrator
prejudiced to a third person with a right recognized by law  Criminalistics – ID, collection, preservation and mode of
 Customs which are contrary to law, public order or public policy presentation of physical evidence; application of sciences in
shall not be countenanced crime detection and investigation
A custom must be proved as a fact according to the rules  Corpus delicti evidence: objects/substances which may be a part
of evidence of the body of the crime
Common law: unwritten laws, but due to customs and  Associative evidence – may link suspect to the crime
traditions, it is rightened to be a law  Tracing evidence – evidence that may assist investigator in
Statutory law: written laws locating the suspect; e.g. blood prints on the floor
 Laws are repealed by subsequent ones, and their violation or  Sketching – rough drawing of the scene or object
non-observance shall not be excused by disuse, custom or o Rough: made at the crime scene
practice to the contrary o Finished: sketch prepared from the rough sketch for court
Even if a law was enacted 100 hundred years ago, but it has presentation
not been repealed or amended no matter how absurd that
law is, it will remain to be in effect  “dura lex sed lex” Kinds of Evidence Necessary for Conviction
(no matter how harsh the law, it is the law)  Direct – that which proves the fact in dispute without the aid
of any inference or presumption
Medical Evidence
Evidence that can stand on its own  DNA, blood, semen,
 Means sanctioned by the rules of court of ascertaining in a
saliva
judicial proceeding the truth respecting a matter of fact
 Circumstantial – the proof of facts from which taken either
For evidence to be admissible, it must be relevant, material and
singly or collectively, the existence of a particular fact in dispute
competent
may be inferred as a necessary or probable consequence
Relevant: evidence having any tendency to make the existence
of any fact that is material to the determination of the proceeding
more probable or less probable than it would be without the
evidence.
Material: evidence that is necessary to determine the outcome of Weight, Sufficiency
matter to theand
case Appreciation of Evidence
a disputed issue  Civil case preponderance of evidence (substantial
o Example: the guilt or innocence of a defendant Competent: evidence only)
evidence that has a sufficient level of reliability.  Criminal case – proof/guilt beyond reasonable doubt
o Introducing evidence that is not relevant, material or (violation of laws)
competent into the case at hand will confuse the jury and  Administrative case = substantial evidence (violation of the Civil
can sway their opinions based on information that has Code, Family Code and contracts)
nothing to do with the alleged crime.
Photography, videotapes, photocopy
Types of Medical Evidence Sketching Methods of preserving evidence
 Autoptic or real evidence – made known to the senses of the Description
court “Manikin” method
 Testimonial evidence – MD in the witness stand In the mind of the witness
Ordinary witness: gives his/her testimony about Special methods: embalming
circumstantial evidence
Expert witness: gives his/her testimony about certain Deception and Detection
topics that the average lay person wouldn't know   Knowledge of truth is important in the administration of
justice, lies solely in the ability to evaluate the statement given
example: A doctor can provide evidence about things like
by the suspect or witness
cause of death
Subpoena duces tecum: a paper that requires a person to
give testimony and bring certain papers with him that  Devices that record the psycho-physiological responses
Methods of deception and detection
[JC Checked by: Page 5 of
LEGAL DA

o Polygraph/lie detector machine – 25% error


You cannot be enforced to undergo a lie detector test
against your will  Right against Self Incrimination Characteristics that may not be easily
 Mental memory
The results of a lie detector test are not admissible
 Speech
because it is not scientific/conclusive unless the
 Gait
subject waives its inadmissibility in a document signed
by him in the presence of a lawyer  Mannerism
o Word association test  Hands and feet
o Psychological stress evaluator  Complexion
 Drugs that try to inhibit the inhibitor  Eye changes
o Truth serum – hyoscine hydrobromide  Facies
o Narcoanalysis  Left/right handedness
o Intoxication  Degree of nutrition
 Hypnotism – alteration of consciousness and concentration in
which the subject manifests heightened suggestibility Gait – manner of walking
 Observation  Ataxic: tabes dorsalis
 Scientific Interrogation  Cerebellar: staggering movement
 Confession – expressed acknowledgment by the accused in a  Cow’s: swaying movement, knock-knee
criminal case of the truth of his guilt as to the crime charged  Paretic: short steps, foot drag
 Spastic: stiff
The only time you could not invoke your right to self-  Festinating: involuntary movement
incrimination is during mechanical acts (fingerprinting) or in  Frog: hopping gait, infantile
acts which do not need the use of intellect  Waddling: duck
Types of Criminal Offenders Facies/Facial Expressions
 Based on behavioral attitude: active, aggressive impulsive
 Hippocratic: pinched nose; the face of impending death
offender
 Based on proficiency  Mongolian: almond eyes
o Ordinary – lowest form of criminal career, limited skill  Facies leonine: lion-like
 Myxedemic: pale, edematous face
o Professional – highly skilled, rarely detected  e.g.
pickpockets/shoplifters  Considered to be the most valuable method of identification and
 Based on psychological classification Fingerprinting
is universally used because:
 Emotional – heat of passion, anger, revenge o There are no two identical fingerprints
 Non-emotional – commit crimes for financial gain and are o Fingerprints are not changeable
usually recidivist
The odds of having the same fingerprint with another person 
Recidivist: those who commit the same act over and over
again 1/64,000,000,000
 Dactylography – is the art and study of recording fingerprints as
Techniques of Interrogation a means of identification
 Emotional appeal – the interrogator is trying to appeal to your  Dactyloscopy – is the art of identification by comparison of
emotions fingerprints; study and utilization of fingerprints
 Mutt and jeff – good cop-bad cop routine; one of them is very  Poroscopy – the study of the pores found on the papillary or
harsh while the other one would contrast the other friction ridges of the skin for purposes of identification
 Bluff on split pair technique – several suspects are interrogated, Kinds of Impression
then a certain suspect enters the suspects room to call a bluff
Example: The interrogator tells the interrogated suspect that  Real impression – impression on the finger bulbs with the
one of the suspects have already told the truth as a bluff to printing ink on the surface of paper
encourage the interrogated suspect to concede  Chance impression – fingerprints impressed by mere chance
 Stern approach – the suspect was made to contest by way of the without any intention to produce which may be
o Visible
use of harsh language  according to Doc Rebosa, this is use
o Plastic
of torture o Latent
 Lengthy, time-consuming narration – lazy interrogation Can fingerprints be effaced? Can fingerprints be forged?
Fingerprints cannot be effaced as long as the dermis of the bulbs
Identification of the finger is not completely destroyed
 Law of multiplicity of evidence in identification
 The greater the number of points of similarities of 2 persons Dental Identification
Possibility of 2 persons to have the same is remote Enamel is
compared, the greater is the ability for the conclusion to be
correct the hardest substance of the body  outlasts other tissues in
putrefaction
Example: in a plane crash with multiple passengers, you are to
identify a certain person. Body parts were scattered. The certain  Bibliotics – science of handwriting analysis
person you are looking for has a foot with 7 digits however the  Graphology – study ofHandwriting
handwriting for the purpose of
body part found had 8 digits, so it is ruled out determining the writer’s personality, character and aptitude

[JC Checked by: Page 6 of


LEGAL DA

 Disguised writing – deliberate attempt on the part of the writer


to alter his writing habit by endeavoring to invent a new writing Identification of Sex
Test to determine the sex:
style or imitating the writing of another person
o Social test
 Signature Forgery Examination – the most common activity of a
o Genital test
questioned document examiner
o Gonadal test
o Such documents may be found in checks, deeds of
conveyances, anonymous letters, suicide notes, receipts etc. o Chromosomal test  Barr cells in females
Question document examiner: a criminologist whose Evidences of sex:
specialty is forensic handwriting analysis o Presumptive evidence
 Classification of signature forgery  General features, hair in some parts
o Traced  Transvestism – sexual deviation by desire to assume
o Simulated the attire and be accepted as a member of the opposite
o Spurious sex
o Highly probable – vagina, large breast
o Conclusive evidence – ovary in females

Identification of Skeletal Remains


 Determination of sex of the skeleton: pelvis, skull, femur, Determination of Age
Legal importance
sternum, humerus Aid to identification
Try to identify first if the bony remains are of humans or Determination of criminal liability
not Determination of right of suffrage
If it is of human remains, identify if it is male or female Determination whether a person can exercise civil rights
 Difference of Male and Female Pelvis Determination of the capacity to marriage
Anatomic Landmark Male Female Requisite to certain crimes
Construction wall Heavier Lighter
Height Greater Lesser  Legal importance in the study of blood
Pubic arch Narrower and Wider and more o ForIdentification of Blood
disputed parentage and Blood Stains
(Paternity/Maternity)
less round round o Circumstantial or corroborative evidence against or in
Diameter of the true Less Greater favor of the perpetrator of the crime
pelvis o Determination of the cause of death
Curve of iliac crest Reaches higher Lower level o Determination of the direction of escape of the victim or
level the assailant
Greater sciatic Narrow Wide o Determination of the approximate time the crime was
notch committed
Body of pubis Narrow Wide o Determination of the exact place of the commission of
Iliopectineal line Sharp Rounded crime
Obturator foramen Egg-shaped Triangular o Determination of the presence of certain diseases
Sacrum Short and Long and wide
narrow Chemical Examination
 Saline extract of the blood stain plus will give a brownish
 Difference of Male and female cranium tinge due to the formation of alkaline hematin
Anatomic Landmark Male Female  Benzidene test
Shaft Less curved More curved  Guiacum test
Mastoid process Larger Smaller  Phenolphthalein test
Cranium placed Mastoid process Occipital and  Leukomalachite Green test
horizontally rests on maxillary bone
Styloid process Shorter Longer/slender Microchemical Tests
Forehead Higher, more Less high, more  Hemochromogen crystal or Takayama test
oblique vertical  Teichman blood crystal test or hemin crystal test
Superciliary ridges Less sharp, Sharper  Acetone – haemin of Wagenhaar test
more rounded
Zygomatic arches More prominent Less prominent Biologic Tests
Lower jaw Larger and Narrower and  Precipitin Test – determines whether blood/semen is of human
wider lighter origin or not
Face Larger in Smaller  Blood grouping
proportion to
cranium Legal Classification of Physical Injuries
Acronym to remember: MLSS  mastoid process is larger and
the styloid process is shorter in the male Determination of the Mutilation
Art. 262 RPC
duration of interment – all soft tissues in a grave disappear Intentionally depriving a person, totally or partially of some of
within one year the essential organs of reproduction
Basis of the estimate for duration of interment Intentionally depriving a person of any part of the human body
o Presence or absence of soft tissue adherent to the bones other than the organs of reproduction
o Firmness and weight, brittleness, dryness of the bones o Mutilation to be punishable must be intentional
o Degree of erosion of the surface of the bones o Vasectomy/tubal ligation = not mutilation; legitimate
o Changes in the clothing, coffin and painting method of contraception despite the fact that it is done

[JC Checked by: Page 7 of


LEGAL DA

intentionally and deprives a person of his power of


o Caused by sharp edged instrument or object (e.g. blade,
reproduction knife, scalpel etc.)
 Self-inflicted incised wounds
Mayhem  Stab wound
Unlawful and violent deprival of another of the use of a part of o Caused by object/instrument with sharp point and sharp
the body so as to render him less able in fighting, either to edge
defend himself or to annoy his adversary o Single/double bladed
 Hacking wound
Serious Physical Injuries o A large incised wound caused by an ax, bolo, samurai etc.
Art. 263 RPC and Art. 264  Contusion/Bruise
Any person who shall wound, beat or assault another (Art. 263) o Extravasation of blood into the surrounding tissues
and administering injurious substance without intent to kill (Art. because of damage to blood vessels
264) o Shape, color (blue, brown, green, yellow)
o Ecchymosis, petechiae
Penalties concerning these injuries  Hematoma (blood cyst/tumor/bukol)
Prison mayor: because of the physical injuries inflicted, the o Extravasation of blood in a newly formed cavity
injured person becomes insane, imbecile, important or blind o Contusion hematoma, preorbital hematoma (blackeye)
Prision correctional in its medium and maximum periods  Post-mortem Contusions
o Loss of speech, hearing or smell The doctor performing the autopsy should determine if its
o Loss of an eye, hand, foot, arm or leg; loss of the use or ante-mortem or post-mortem
incapacitated for the habitual work he used to do Ante-mortem: the patient sustained the contusions when he
Prision correctional in its minimum and medium periods was alive, probably due to assault or a beating
o Person injured shall be deformed  the accused will be convicted
o Loss of any other body part, incapacitated for more than Post-mortem: if the patient was already dead when he
90 days
sustained the contusions, the accused might be acquitted
Arresto mayor in its maximum period to prision correctional
What should the doctor do in the autopsy table? The doctor
o If the physical injuries shall have caused the illness or should incise the contusion
incapacity for labor for more than 30 days
 Presence of blood in the surrounding tissues
= ante-mortem
Less Physical Serious Injuries  No presence of blood/blood is still in the vessels
Art. 265 RPC = post-mortem
Injuries which cannot be considered as serious as the  Abrasions
aforementioned articles above o Contact with rough surface, friction, graze, scratch
o Linear, multi-linear, confluent
Do not take abrasions for granted as it tells a thousand
Slight Physical Injuries and Maltreatment
Art. 266 RPC stories  find the reason behind the presence of abrasions
Arresto menor: offender has inflicted physical injuries which
shall incapacitate the offended party for labor from 1 to 9 days Gunshot Wounds
or shall require medical attendance of the same period Arresto Death or physical injuries brought about by powdered propelled
menor or fine not exceeding P200 and censure when the substances
offender has cause physical injuries which do not prevent the o Firearm shot: the injury is caused by the missile propelled
offended party from engaging in his habitual work nor require by the explosion of gunpowder located in the cartridge shell
medical attendance and the rear of the missile
Arresto menor in its minimum period or a fine not exceeding o Detonation of high explosives (grenades): explosion inside
P50 when the offender shall ill treat another by deed without the metallic container will cause fragmentation of the
causing injury container
Firearm – an instrument used for the propulsion of a projectile
by the expansive force of gasses coming from the burning of
Physical Injuries Inflicted in a Tumultuous Array gunpowder
Art. 252 RPC
There is a tumultuous array o Includes muskets, shotguns, revolvers, pistols, other deadly
Participants suffered from serious physical injuries weapons which a bullet, ball, shell or other missile may be
The person who inflicted serious physical injuries cannot be discharged by means of gunpowder or other explosives
identified o Includes air rifles except of small calibers and limited range
All those who appear to have used violence upon the person of o The barrel of any firearm shall be considered as a complete
the offended party shall be penalized by arrest from 5 to 15 days firearm for all purposes thereof
Nature of Injuries A weapon in order to cause injury must have two principal
 Lacerated/Avulsed Wound component parts:
o Caused by contact with blunt object o The cartridge or ammunition: bullet primer, cartridge case,
o Jagged or irregular powder charge
o E.g. fist blows, vehicle bumpers, baseball bat, lead o Firearm instrument for the propulsion of a projectile force
pipes etc. of gases from a burning powder
 Punctured Wound  Point of entry vs Point of exit
o Caused by sharp-pointed instrument/object (e.g.
needle, icepick, nail, pin etc.)
 Incised (sliced/cut) wound

[JC Checked by: Page 8 of


LEGAL DA

Entrance Wound Exit Wound Personal history may reveal social, economic, business or marital
problems which cannot be resolved
Appears to be smaller than Always bigger than the
Gunpowder presence in the hand of the victim
the missile owing to the missile
elasticity of the tissue Entrance wound usually does not contain clothing
Edges are inverted Edges are everted Fingerprints of the victims on the butt
Suicide note at the vicinity
Usually oval or round Does not manifest any
No disturbance in the place of death
depending upon the angle of definite shape
approach of the bullet Russian roulette – unfortunate victim has no predetermined desire
of self-destruction
Contusion collar or contact ring Absent
is present due to invagination
Site of wound of entrance has no point of election Fire
of the skin and Evidences that
spinning of the missile is made when thethe Gunshot
victim Wounds
is at some distancemay beofHomicidal
Signs
struggle or defense wounds
Tattooing/smudging/singeing Absent
Disturbance in the surroundings
may be present when firing is
Wounding firearm usually not found in the scene of the crime
near
Testimony of witness
Underlying tissues are not Underlying tissues may be
protruding or seen protruding from the wound
 Always present after
Products of combustionfire May be absent, if missile is
lodged in the body Evidences that the Gunshot Wounds may be Accidental
 Tattooing/smudging/singeing: as seen in close contact wounds Usually one shot
Paraffin test may be positive Negative No special area of body involved
 Contusion collar: found in the point of entry
Shotgun wounds: a shoulder fired firearm having a barrel that is Consideration on the testimony of the assailant and
smooth-bored and is intended for the firing of a changed determination as to whether it is possible by knowing the
compound of one or more balls or pellets relative position of the victim
o In shotgun wounds, the bullets are huge (slugs) with Testimony of the witness
hundreds of pellets inside  once the pellets are released
Dermal nitrate tests (Paraffin test, Diphenylamine test, Lung’s
in the air, all hit each other  billiard ball effect Tests fortest)
test, Gonzales’ the Presence of Powder Residues
o Paraffin test: false positive/false negative
How would you know whether a gun recovered during a stray Walker’s test (C-acid test, H-acid test)
bullet incident was the same gun investigated?
o Get that gun Harrison and Gilroy test
o Test fire that gun – try to simulate the same conditions Tests
Neutron for the presence
Activation Analysis of Primer Components
o Get the bullet extracted from the body of the crime o More advanced test in modern times
scene o Extremely sensitive even with small quantities
o Examine both bullets using a double comparison match Flameless Atomic Absorption Spectroscopy
o If the metallic abrasions seen on both the bullet fired from Use of scanning electron microscope with a linked x-ray analyzer
the gun and the bullet recovered from the crime scene are
Thermal Injuries
the same  it can be concluded that it was fired from the
Burns – due to flame or fire
same gun
Scalding – due to boiling liquid
o Firearm identification ballistics
o Scalding liquid runs on the body surface  distributing
the heat
Penal provisions of laws relative to firearm o Easily cools off
Sec. 2692 RAC – unlawful manufacture, dealing in acquisition, o Temperature not as high except in oils and molten
disposition or possession of firearms or ammunitions thereof or metals
instruments used or intended to be used in the manufacture of o Dupuytren’s Classification
firearms or ammunition  1st degree: erythema
Sec 2690 RAC – selling of firearms to unlicensed purchaser Sec  2nd degree: vesicle formation
2691 RAC – failure of personal representative of deceased  3rd degree: destruction of the cuticle, part of true
licensee to surrender firearm skin, painful
Art 155 RPC – Alarms and Scandals Art  4th degree: whole skin is destroyed, ulceration, not
254 RPC – Discharge of firearms painful
 5th degree: deep fascia, muscles
Evidences that the Gunshot Wounds may be Suicidal  6th degree: charring of the limbs
Shots fired in a closed locked room, or open uninhabited place
Death near the place where victim was found Special Types of Injuries
Shot fired with the muzzle of the gun in contact with the part of  Defense wounds: instinctive reaction of self-preservation 
body involved hands/fractures
Location of entrance wound accessible to the part of the body  Patterned wounds: wound in the nature and shape of the
Shot usually solitary instrument  wheels, abrasions from ropes
Direction of fire is compatible with the trajectory of bullet  Suicidal wounds: wound produced with an intent to take
one’s life

[JC Checked by: Page 9 of


LEGAL DA

 Self-inflicted wounds: wound produced on oneself but no This stage usually lasts about three to six hours after death. In
intention to end his life warm places, the average duration is only one hour and fifty-
one minutes (Mackenzie cited by Modi, p. 122) Chemically, the
Death
reaction of the muscle is alkaline and the normal constituents of
RA 7170 – Organ Donation Act of 1991
the individual muscle proteins are the same as in life
Death – the irreversible cessation of circulatory and respiratory
The cells in the body are still alive, and anaerobic chemical
functions or the irreversible cessation of all functions of the
processes may continue in the tissue cells. (Reddy & Murty,
entire brain, including the brain stem
2014)
A person shall be medically and legally dead if either:
o There is an absence of natural respiratory and cardiac Stage of Postmortem Rigidity (Rigor Mortis)
functions and, attempts at resuscitation would not be
successful in restoring those functions. Post-mortem stiffening of the body muscles, as a result of the
 In this case, death shall be deemed to have occurred at shortening of the muscle fibers
the time these functions ceased; or At this stage, the individual cell death begins to take place,
o There is an irreversible cessation of all brain functions; and certain biochemical changes in the muscle
considering the absence of such functions further attempts The muscles begin to stiffen 4 hours after death
at resuscitation or continued supportive maintenance would By the end of 8-12 hours, the stiffening extends to the entire body
not be successful in restoring such natural functions. Rigor mortis affects both, voluntary and involuntary muscles of
 In this case, death shall be deemed to have occurred at the body
the time when these conditions first appeared. The stiffening of the muscles occurs sequentially.
The death of the person shall be determined in accordance with o The first to stiffen are the muscles of the eyelids.
the acceptable standards of medical practice and shall be o This is followed by the muscles in the neck region, lower
diagnosed separately by the attending physician and another jaw, facial muscles, chest muscles, upper limbs, abdominal
muscles, and lower limbs
consulting physician, both of whom must be appropriately
qualified and suitably experienced in the care of such patients. o The fingers and toes, stiffen last
The death shall be recorded in the patient's medical record In warm countries, rigor mortis sets in from 2-3 hours after death
o It is fully developed in the body after 12 hours
o It may last from 18 – 36 hours and its disappearance is
Three Kinds of Death concomitant with the onset of putrefaction
Somatic
Stage of Secondary Flaccidity and Onset of Decomposition
o state of the body in which there is complete, persistent
and continuous cessation of the vital functions of the Rigor mortis will continue to persist until the decomposition of
brain, heart and lungs which maintain life and death the muscle fibers takes place, due to the action of microbial
o Hardly impossible to determine the exact time of death activity
o Immediately after death, Decomposition is due to action of bacteria in various tissues of
 The face and lips become pale the body
 The muscles become flaccid This process begins at the point of death and continues from then
 The sphincters are relaxed on, until the body is skeletonized
 The lower jaw tends to drop The decomposition process can be broken down into a series of
 The eyelids remain open and pupils dilate stages (Lee Goff, 2009)
 Skin loses its elasticity o Autolysis: self-digestion of tissues
 Body fluids tend to gravitate to the most dependent Begins soon after death when the cell membranes
portions of the body become permeable and begin to breakdown
 Body heat gradually assumes the temperature of the Proteolytic, glycolytic and lipolytic ferments of
surroundings glandular tissues continue to act which lead to the
Molecular autodigestion of organs
o After cessation of the vital functions, there is still animal Facilitated by weak acid and higher temperature and
life among individual cells. delayed by the alkaline reaction of the tissues of the
o About 3-6 hours later, there is death of individual cells body and low temperature
known as molecular or cellular death o Putrefaction: final stage of death
o Exact occurrence cannot be definitely determined due to Putrefaction is the breaking down of the complex
several factors proteins into simpler components associated with the
State or suspended animation or apparent death evolution of foul-smelling gases and
o This condition is not really death but merely a transient loss accompanied by the change of color of the body This
of the vital functions of the body on account of disease, stage often follows the phase of rigor mortis By this
external stimulus or other forms of influence. time, there can be visible and sometimes quite
o It may arise especially hysteria, uremia, catalepsy and grotesque changes that can be seen on the body
electric shock
Medico-Legal Significance of Death
The civil personality of a natural person is extinguished by death
Muscular Changes Following Death The property of a person is transmitted to his heirs at the time of
Stage of Postmortem Irritability (Primary Flaccidity)
death
Immediately after death, there is complete relaxation and
The death of the partner is one of the causes of dissolution of
softening of all the muscles of the body
partnership agreement
The extremities may be flexed, the lower jaw falls, the
eyeball loses its tension, and there may be incontinence of
urination and defecation

[JC Checked by: Page 10 of


LEGAL DA

The death of either the principal or the agent is a mode of


o Determining the cause of death
extinguishment of agency
o Providing correlation of clinical diagnosis and clinical
The criminal liability of a person is extinguished by death; The symptoms
civil case for claims which does not survive is dismissed upon
o Determining the effective- ness of therapy
the death of the defendant.
o Studying the natural course of disease process; and
o Educating students and physicians
Forensic Entomology The Civil Code states who is the rightful person to give such
Clostridium perfringens/Clostridium welchii: most important consent – Art. 294 and 305
microorganism in determining type of death by way of forensic o The consent shall be obtained from:
entomology  The spouse
The microorganism that plays an important role in  The descendants of the nearest degree;
decomposition is Clostridium welchii  The ascendants, also of the nearest degree
o It grows in the parenchymatous organs and is responsible  The brothers and sisters
for the disintegration of cytoplasm, destruction of nuclei  As stated in Art. 294, Civil Code
and generation of gasses in the cells o In case of descendants of the same degree, or of brothers
Other destructive agents during decomposition include flies and and sisters, the oldest shall be preferred. In case of
maggots, reptiles, rodents, other mammals, fishes and crabs, and ascendants, the paternal shall have a better right (Art. 305,
molds Civil Code)

Medico-Legal or Official Autopsy


Postmortem Lividity/Stasis
Post-mortem lividity: usually develops 3-6 hours after death An examination performed on a dead body for:
Appears as small petechia-like red spots which later coalesce o Determining the cause, manner and time of death,
with each other to involve bigger areas in the most dependent recovering, identifying, and preserving evidentiary material
portions of the body o Providing interpretation and correlation of facts and
Hypostatic Lividity circumstances related to death
o The blood merely gravitates into the most dependent o Providing a factual, objective medical report for law
portions of the body but still inside the blood vessels and enforcement, prosecution, and defense agencies
still fluid in form o Separating death due to disease from death due to external
o Any change of position of the body leads to the formation cause for protection of the innocent
of the lividity in another place In cases which require a medico-legal autopsy, the dead body
o This occurs during the early stage of its formation belongs to the state for the protection of public interest until
Diffusion Lividity such time as a complete and thorough investigation into the
o This appears during the later stage of its formation when circumstances surrounding the death and the cause thereof has
the blood has coagulated inside the blood vessels or has been completed
diffused into the tissues of the body The physician tasked to perform such autopsy is considered to be
o Any change of position will not change the location of the the authoritative agent and representative of the state who has
lividity the "property right" of the dead body
Importance of Cadaveric Lividity
o It is one of the signs of death Sec. 95 (b), P.D. 856, Code of Sanitation:
o It may determine the position of the body after death or Persons
o Health Authorized to Perform Autopsies
officers
whether it was changes after its appearance in the body o Medical officer of law enforcement agencies
o The color of the lividity may indicate the cause of death o Members of the medical staff of accredited hospitals
 E.g. Asphyxia – dark CO poisoning  bright pink
o It gives us an idea as to the time of death Sec. 95 (b), P.D. 856, Code of Sanitation:
When shall an required
o Whenever Autopsybybe Performed
special laws; on a Dead Body?
Postmortem Examinations and Autopsies o Upon order of a competent court, a mayor and a provincial
An autopsy is a comprehensive study of a dead body, performed or city fiscal;
by a trained physician employing recognized dissection o Upon written request of police authorities;
procedure and techniques. o Whenever the Solicitor General, Provincial or city fiscal as
It includes removal of tissues for further examination authorized by existing laws, shall deem it necessary to
Autopsy v Postmortem Examinations disinter and take possession of the remains for examination
o Autopsy: indicates that, in addition to an external to determine the cause of death
examination, the body is opened and an internal o Whenever the nearest kin shall request in writing the
examination is conducted authorities concerned in order to ascertain the cause of
o Post-mortem examination: refers to an external death
examination of a dead body without incision being made,
although blood and other body fluids may be collected for Sexual Crimes: Against Chastity
examination Acts of Lasciviousness
Kinds of Autopsies
Hospital or Non-official Autopsy Acts conduct which is wanton, lewd, voluptuous or lewd
Medico-legal or Official Autopsy emotion

Hospital or Non-official Autopsy Art. 336. Acts of Lasciviousness


An autopsy done on a human body with the consent of the Any person who shall commit any act of lasciviousness upon
deceased person's relatives for the purposes of: other persons of either sex, under any of the

[JC Checked by: Page 11 of


LEGAL DA

circumstances mentioned in the preceding article, shall be


punished by prision correccional Adultery
Elements: Art. 333. Who are guilty of adultery
o The offender commits any acts of lasciviousness; Adultery is committed by any married woman who shall have
o The lascivious act is done under any of the following sexual intercourse with a man not her husband and by the man
circumstances: who has carnal knowledge of her knowing her to be married,
 By using force or intimidation. even if the marriage be subsequently declared void. Adultery
 By depriving her of her reasons or otherwise shall be punished by prision correccional in its medium and
unconscious; maximum periods.
 When the woman is under 12 years of age; If the person guilty of adultery committed this offense while
o The offended party must be a person of either sex being abandoned without justification by the offended spouse,
the penalty next lower in degree than that provided in the next
Art. 339. Acts of Lasciviousness with the Consent of the preceding paragraph shall be imposed.
Offended Party Elements:
The penalty of arresto mayor shall be imposed to punish any o The woman is married;
other acts of lasciviousness committed by the same persons and o She has had sexual intercourse with a man not her husband;
the same circumstances as those provided in and
Articles 337 and 338 o The man with whom she had sexual intercourse knows her
Elements: to be married even if the marriage has subsequently been
declared void.
o The offender commits acts of lasciviousness;
o The offended woman must be over 12 but under 18 years of
Art. 334. Concubinage
age, except when the victim is the sister or descendant of Concubinage
the offender Any husband who shall keep a mistress in the conjugal dwelling,
o The offender commits the act by abuse of authority, or shall have sexual intercourse, under scandalous
confidence, relationship or deceit circumstances, with a woman who is not his wife, or shall
o The victim must be a woman, virgin, single, or widow of cohabit with her in any other place, shall be punished by prision
good reputation, except when she is the sister or descendant correccional in its minimum and medium periods.
of the offender where virginity is not required Ways of committing the crime
o Keeping a mistress in the conjugal dwelling
Medical Evidences in the Crime of Acts of Lasciviousness o Having sexual intercourse, under scandalous
Like in the crimes of rape and seduction, medico-legal circumstances, with a woman not his wife
o Cohabiting with her in any other place
investigation is involved in proving the lascivious act itself and
the other elements to constitute the crime
Rape is now a public crime and not a private crime or crime against
Physical injuries may be suffered by the victim on the part of the Marital Rape
chastity as of 1997
body where the lascivious act was committed
The crime of rape has been amended in 1997 Old
law:
Abduction o Carnal knowledge or benign penetration
Abduction is the carrying away of a woman by an abductor with o Only a man can commit the crime of rape, and only a
lewd design. woman can be a victim of rape
Lewd design is the intent of the abductor to have sexual As of 1997:
intercourse with the woman abducted. o Insertion of foreign objects in the genitalia is now
considered rape
Art. 342. Forcible Abduction o Sodomy is considered rape
The abduction of any woman against her will and with lewd o A man can be a victim of rap e and a woman can be
designs shall be punished by reclusion temporal. The same
penalty shall be imposed in every case, if the female abducted be
under twelve years of age.
Elements charged with rape
o The victim abducted is a woman; o Forced oral sex on a person is also considered rape and
o The abduction is against her will; sexual assault
o The abduction is with lewd design
Medico-Legal Aspects of Sexual Crimes
Art. 343. Consented abduction
Virginity
The abduction of a virgin over twelve years and under Virginity is a condition of a female who has not experienced
eighteen years of age, carried out with her consent and with sexual intercourse and whose genital organs have not been
lewd designs, shall be punished by the penalty of prision altered by carnal connection.
correccional in its minimum and medium periods Elements A woman is a "virtuous female" if her body is pure and if she
o The offended party must be a virgin has never had any sexual intercourse with another, though her
o The offended party must be over 12 but under 18 mind and heart is impure (Thomas v. State, 19 Ga. App. 104, 91
years old S.E. 247, 250).
o The carrying away of the offended party is with her The presumption of a woman's virginity arises whenever it is
consent shown that she is single and continuous until overthrown by
o The taking away must be with lewd design proof to be contrary (U.S. V. Alvarez, 1 Phil. 242).
A woman is presumed to be a virgin when unmarried and of
good reputation

[JC Checked by: Page 12 of


LEGAL DA

Kinds of Virginity
 or obliteration of the vaginal rugosities. There will be
Moral Virginit y – the state of no t knowing the nature of laxity of its wall so that insertion of a medium size
sexual life and not having experienced sexual relation tube during the medical examination can be done with
o Moral virginity applies to children below the age of puberty slight resistance.
and whose sex organs and secondary sex characters are not o Hymen
yet developed  The fact that hymen is intact does not prove absence
P hysical Virginity – a condition whereby a woman is conscious of previous sexual intercourse
of the nature of the sexual life but has not experienced sexual  The presence of laceration does not mean defloration
intercourse
o The term applies to women who have reached sexual Passage of clotted blood during menstruation
maturity but have not experienced sexual intercourse Ulceration due to disease
Non-sexual Causes of Hymenal Laceration
o True Physical Virginity: a condition wherein the hymen is
intact with the edges distinct and regular and the
o pening small to barely admit the tip of the smalles t Jumping or running
finger of the examiner Falling on hard and
 E.g. if the thighs are separated sharp object
o F alse Physical Virginity: condition wherein the hymen Medical instrumentation
is unruptured but the orifice is wide and elastic to admit Local medication
two or more fingers of the examiner with a lesser degree of Self-scratching due to
resistance. irritation
 The hymen may be laxed and distensible and may Masturbation
have previous sexual relation. Insertion of foreign bodies
 In this particular instance the physician may not be Previous operation
able to make a convincing conclusion that the subject
is virgin.
Demi-virginity – refers to a condition of a woman who permits Classification of Hymenal s
any form of sexual liberties as long as they abstain from Laceration
rupturing the hymen by sexual act Incomplete laceration – rupture or laceration of the hymen is
o The woman may be embraced, kissed, may allow her considered incomplete when it does not involve the whole width
breasts to be fondled, her private organ to be held and other or height of the hymen
lascivious acts o Superficial – the laceration does not go beyond one half of
o The woman allows sexual intercourse but only "inter- the whole width of the hymen
femora" or even "inter-labia" but not to the extent of o Deep – the laceration involves more than one-half of the
rupturing the hymen width of the hymen but not reaching the base
"Virgo Intacta" – refers to a truly virgin woman Complete laceration – the hymenal laceration involves the
o No structural changes in her organ to infer previous sexual whole width but not beyond the base of the hymen Compound or
intercourse and that she is a virtuous woman complicated laceration – the laceration involves the hymen and
o Inasmuch as there are no conclusive evidences to prove the also the surrounding tissues
existence of such condition, liberal authorities extend the o It may involve the perineum, vaginal canal, urethra or
connotation of the term to include women who have had rectum
previous sexual act or even habitually but had not given Notches – indentation of the hymen simulating lacerations.
birth o They may be symmetrical and may extend to the vaginal
wall
o The mucous membrane over the notch is intact
Defloration o Notches may be mistaken for laceration
Defloration is the laceration or rupture of the hymen as a result
of sexual intercourse Exempting v Mitigating Circumstances
Parts of the female genitalia that must be examined to Mental Illness may be an exempting or mitigating circumstance
determine defloration: to criminal liability
o Condition of the Vulva
 Normally the labia majora and minora are in close o Art. 12, Revised Penal Code
contact with one another covering almost completely As an Exempting
o Circumstances which exemptCircumstance
from criminal liability
the external genitalia o The following are exempt from criminal liability:
 After defloration, the labia may gape exposing the  An imbecile or an insane person, unless the latter has
introitus vulvae acted during a lucid interval.
o Fourchette  When the imbecile or an insane person has committed
 The normal V-shape of the fourchette is lost on an act which the law defines as a felony (delito), the
account of the previous stretching during insertion of court shall order his confinement in one of the
the male organ hospitals or asylums established for persons afflicted,
 Retraction of the fourchette is not a good sign of which he shall not be permitted to leave without first
defloration obtaining the permission of the same court
 Ballet dancing, separation of the thighs, tree climbing,
cycling, horse riding, insertion of foreign body, etc. Imbecile distinguished from insane (Pascual, 2021)
may cause retraction of the fourchette without
previous sexual act. nsa
o Vaginal canal Imbecile is exempted in all cases while i e is not
 After repeated sexual acts, there is diminution of the exempted if it can be shown that he acted during a lucid interval
sharpness

[JC Checked by: Page 13 of


LEGAL DA

o During lucid interval (brief period of regaining sanity), the


A person suffering from delusion is not always insane
insane acts with intelligence.
If he can correct his wrong belief by later experiences, by logic
o Imbecile: one who, while advanced in age, has a mental
or information from other sources, then such delusion is not a
development comparable to that of children between 2-7
years old. proof of insanity
 One who is deprived completely of reason or
discernment and freedom of will at the time of Types of Delusion
committing the crime Delusion of Grandeur
To constitute insanity, there must be complete deprivation of o "Delirium of grandeur”
intelligence or that there be total deprivation of the freedom of o Megalomania or "folie de grandeur" – erroneous belief that
will he is in possession of great power, wealth, wisdom,
o Insanity is an exempting circumstance physical strength, etc.
o Complete deprivation of intelligence while commit an act; o It is not always a sign of insanity.
acts without the least discernment; or total deprivation of o A person may think he is a king and dresses and acts as
freedom of will such
Delusion of Persecution – a false belief that one is being
Case Where Defense of Insanity were Upheld by Court persecuted
An accused who committed homicide and has been known to be o A person may feel that he is being poisoned and prepares
for his coming end.
suffering from dementia praecox with delusions that he was
Delusion of Reference – one thinks that he is always the subject-
being molested sexually, or that his property was being taken,
matter of conversation, news, speech or action although it is not
was considered insane and exempted from criminal liability
a fact
(People v. Bonoan, 64 Phil. 87)
Delusion of Self-accusation – a false belief to have committed a
crime or hurt the feeling of others
As a Mitigating Circumstance Delusion of Infidelity – a false belief derived from pathological
Art. 13, Revised Penal Code — Mitigating circumstances jealousy that one's lover is unfaithful although she is chaste, and
o The following are mitigating circumstances: tries to assault her
 That the offender is deaf and dumb, blind or otherwise Nihilistic Delusion – a false belief that there is no world, that
suffering from physical defect which thus restricts his one does not exist, and that his body is dead
means of action, defense, or communication with his o This condition may occur in involutional melancholia
fellow beings. Delusion of Poverty – a false belief that one is financially
 Such illness of the offender as would diminish the ruined and that he has no money, is starving, sick or even dead
exercise of the will-power of the offender without Delusion of Control -- a false feeling that one is being controlled
however depriving him of consciousness of his acts. by other persons
 Any other circumstance of a similar nature and Hypochondriacal Delusion – a false feeling that one is suffering
analogous to those above mentioned. from an incurable disease, some parts of his body are not
One who was suffering from acute neurosis which made him ill- functioning, or that he is not physically capacitated to do a thing
tempered and easily angered was entitled to the mitigating on account of the disease.
circumstance because illness diminished his exercise of will Delusion of Depression – a patient experiences feelings of
power uneasiness, worthlessness and futility
o People v. Carpenter C.A. G.R. 4168 Apr. 22, 1940) Delusion of Negation – a feeling that some parts of the body are
Complete absence of willpower  exempting missing

Forensic Psychiatry Excessive, irrational and uncontrollable fear of a perfectly


natural situation or objectPhobias
Intelligence Quotient
Several test types are prepared corresponding to every age in
months and these are answered by the person examined The age
of the person examined is determined in terms of months
The number of months corresponding to the test type answered
divided by the age of the person in months is the intelligence
quotient (I.Q.)
Classification of IQ
IQ Classification
Above 140 “Near” genius or genius
120-140 Very superior intelligence
110-120 Superior intelligence
90-110 Normal or average intelligence
80-90 Dullness, rarely classified as feeble-
minded
70-80 Borderline deficiency, sometimes
classified as dullness, often as feeble-
minded
Below 70 Definitely feeble-minded

Delusions
A false or erroneous belief in something which is not a fact
Fear of Specific Objects
Ornithophobia – birds Harpaophobia – robbers
Hematophobia – blood Hierophobia – sacred things
Bibliophobia – books Belonophobia – sharp objects

[JC Checked by: Page 14 of


LEGAL DA
Antophobia – flowers Heliophobia – sun
Androphobia – men Dendrophobia – trees
Fear of Specific Situations
Tocophobia – childbirth Clinophobia – going to bed
Gephyrophobia – crossing Gamophobia – marriage
a bridge Agoraphobia – open space
Pengophobia – daylight Maieusiophobia – pregnancy
Dipsophobia – drinking Coitophobia – sexual
Acrophobia -- height intercourse
Fear of Place
Ecclasiophobia – churches Ochlophobia – crowds
Kenophobia – empty room Thalassophbia – sea
Claustrophobia – enclosed Ecophobia – home
spaces surroundings
Scholionophobia – school Potamophobia -- rivers
Siderodromophobia –
railroads
Fear of Illness or Death

[JC Checked by: Page 15 of


LEGAL DA

Thanatophobia – death Mysophpbia – infection Consequently, the public has a varied reaction as to whether it
Pathophobia – disease Apeirophobia – infirmity must be legalized or prohibited as an athletic sport
Spermophobia – germs Baciliphobia – microbes
Cardiophobia – heart Ophidiophobia – snakes Reasons Why Boxing Should Not Be Prohibited:
disease Cypridophobia – venereal It takes wayward youths who are victims of the educational
Nosemaphobia -- illness disease system off the streets
It teaches them self-discipline and controls and reinforces the
Asphyxia adage that nothing of value is acquired without hard work and
sacrifice
Anoxic Death Self-confidence can only be promoted through an individual
This is associated with the failure of the arterial blood to
become normally saturated with oxygen sport where the athlete must rely in his own talent and believes
It may be due to: in his own ability
o Breathing in an atmosphere without or with insufficient o Only through conflict can hidden resources surface
oxygen, as in high altitude
o Obstruction of the air passage due to pressure from Reasons Why Boxing Should be Prohibited:
outside, as in traumatic crush asphyxia There is too much risk of death or injury to the participants.
o Paralysis of the respiratory center due to poisoning, injury Unlike other sports, the intention of the combatants is to produce
or anesthesia, etc. injury as a principal way to win the contest.
o Mechanical interference with the passage of air into or o Young men should be discouraged from a pugilistic career
down the respiratory tract due to:
 Closure of the external respiratory orifice, like in Potential Injuries Suffered by Combatants in Boxing
smothering and overlaying The most common site of lacerated wound is the region of the
 Obstruction of the air passage, as in drowning, eyebrow and the cheek
choking with foreign body impact, etc. o Bleeding comes from the small blood vessels which are
 Respiratory abnormalities, like pneumonia, crushed between the superficial bone and the force applied
asthma, emphysema and pulmonary edema o Bleeding from the eyebrow may drip downwards to the eye
o Shutting of blood from the right side of the heart to the left and causes irritation and blurring of vision
without passage through the lungs as in congenital o If bleeding cannot be controlled on account of the severity
anomalies like potent foramen ovale of the injury, the referee may stop the bout
o Protagonist may be knocked down or accidentally fall on
the canvass causing laceration of the scalp
Anemic Anoxic Death
This is due to a decreased capacity of the blood to carry Serous effusion on the loose tissue around the eyeball and in the
oxygen eyelid may cause puffiness and closing of the eye
This condition may be due to: o Fracture of the orbital plate of the frontal bone may cause
o Severe hemorrhage swelling and discoloration in or around the eye
o Poisoning, like carbon monoxide  spectacle hematoma
o Low hemoglobin level in the blood Trauma on the pinna of the ear may produce hematoma with
subsequent necrosis of the auricular cartilage
o After healing and scarring process, the pinna appears to be
Stagnant Anoxic Death
This is brought about by the failure of circulation thick and irregular  cauliflower ears
The failure of circulation may be due to: Fracture of the nasal septum, mandible and maxillary bone may
o Heart failure develop as a consequence of a hard hook or a straight blow
o Shock o Fracture of the skull is quite unusual to develop because of
o Arterial and venous obstruction, incident to embolism, the difficulty to do so with a gloved fist.
vascular spasm, varicose veins, or the use of tourniquet Retinal detachment  partial or complete loss of vision Muscle
cramps, sprain and dislocation
Histotoxic Anoxic Death A kidney punch may cause peri-renal hemorrhage or laceration of
This is due to the failure of the cellular oxidative process, al- the kidney  loss of function and uremia
though the oxygen is delivered to the tissues, it cannot be A blow on the face may cause laceration of the lip and
utilized properly buccal mucosa with loosening or detachment of the teeth
Cyanide and alcohol are common agents responsible for Intracranial injuries
histotoxic anoxic death o Cerebral concussion (transitory period of
unconsciousness)
Sports Injuries
 Results from a blow on the head that may last for a
Boxing few seconds or longer
Boxing as a sport is sometimes described as an "organized o Subdural hemorrhage
brutality", "slaughter" and "carnage"  Most common injury which may be localized or
Unlike other sports, the primary objective of the combatants is to extensive
knock out or win by decision by delivering a stunning or  The most common site of hemorrhage is the middle
weakening punches cranial fossa
Generally, in most sports, the infliction of physical injuries is  A blow on the jaw is transmitted to the temporo-
purely accidental but in boxing, it is the direct and primary mandibular joint and then to the middle cranial fossa
objective of the combatants
o In this respect, boxing is considered to be one of the most
brutal among the athletic sports

[JC Checked by: Page 16 of


LEGAL DA

 Repeated trauma on the skull cause tearing of the dural


A distortion of sense of time, distance, vision and hearing.
emissary veins
o A minute seems like an hour
o Pontine hemorrhage
o Eyes tend to focus on one object to the exclusion of others
 Boxer’s hemorrhage
 May be the result of severe beating. o Certain sounds become striking in character and music
takes on a new dimension
Whetted appetite – food and drink taste especially good.
Drugs
A tendency to be confused about the past, present and future
Amphetamine Impaired short-term memory Tendency
Also called "meth" "speed", "crystal", "crank", "white cross to be easily distracted.
tablets The suggestibility and release of inhibition.
Increased sense of sociability and hilarity.
Dangers of Amphetamine Misuse These effects are at peak, shortly after smoking and fade away after
Overactivity leading to social consequence (car accident) or a few hours, leaving a desire to sleep
aggressive behavior; stealing and murder may have been
associated with excessive amphetamine taking Objective Effects of Marijuana
Production of a psychotic illness of the schizophrenic type. Moderate increase in resting pulse rate
Shock and collapse following amphetamine usage and excessive Reddening of the eyes due to dilatation of the conjunctival blood
physical exertion vessels
May lead to habituation Difficulty of speech and of remembering the logical trend of
Risk of suicide during the withdrawal phase. what was being said
Neurological and EEG examinations show slight increase in cortical
functions
Lysergic Acid Diethylamide (LSD)
First synthesized by Dr. Albert Hoffman and Dr. Arthur Stell Tremor and muscular incoordination In
while working in a Swiss pharmaceutical firm high dosage it may cause:
Synthesized from the ergot alkaloids (Claviceps purpurea) and o Frank hallucination, delusion and paranoid feeling
diethylamide portion of ergotamine ergonevine, the active o Confused and disorganized thinking
ingredient of oxytocic and vasoconstrictor drugs o Toxic psychosis

Symptoms of LSD use Other undesirable effects


Physiological: dilatation of the pupils, over-activity of Bronchitis and asthma
reflexes, Nausea and vomiting for novice smokers
increase of muscle tension, lack of coordination, visual Panic reaction occurs when the individual becomes frightened
disturbance, laughter. about the effects of the drug and starts to doubt that the changes
Somatic: dizziness, weakness, tremor, nausea, are irreversible.
drowsiness, paresthesia and blurred vision Amotivational Syndrome – progressive change from
Perceptual: alteration of shapes and color; music appreciation conforming, achievement-oriented behavior to a state of relaxed
with abnormal intensity; focusing difficulty; sharpening of the drifting
hearing sense, recurrent voice accompanied by brilliant Acute toxic psychosis
hallucinatory color sensation (synesthesia or seeing sound,
hearing color, etc.) References
Psychic: mood alteration, tension, distortion of time sense, Legal Medicine by Pedro P. Solis, MD, LLB, D.Crim Philippine
difficulty in thought expression, depersonalization, dream- like Medical urisprudence, Platinum Series by Albert
feeling and visual hallucination. D. Rebosa MD, LLB
o Delusion of omnipotence is common such that a user thinks Modern Legal Medicine Psychiatry and Forensic Science by Curran,
he can fly from a high building McGarry and Petty
o A number of deaths occur in this manner Forensic Pathology, A Handbook for Pathologists, Fisher and
Petty, July 1977, p. 1)
R.A. 7170. (2021). Retrieved 11 December 2021, from
Marijuana https://www.lawphil.net/statutes/repacts/ra1992/ra_7170_1 992.html
Mexican term for "pleasurable feeling"
Mixed preparation of the flowering tops, leaves, seeds and stem Ganesh, K. (2021). Post-Mortem Changes & Time Since
of the hemp plant, Cannabis sativa Death. Retrieved 11 December 2021, from
The potency of the mixture depends on the resin content and this https://legaldesire.com/post-mortem-changes-time-since- death/
is determined mainly by the plant strain and also by the factors Stages of Lividity or Livor Mortis. (2021). Retrieved 11
involved in cultivation, harvesting and preparation of the crop December 2021, from
Marijuana is not addictive https://www.coursehero.com/file/p5dlddi/Stages-of-Lividity- or-
o Physical dependence and dose tolerance do not develop Livor-Mortis-1-Hypostactic-Lividity-It-is-the-lividity/
with its use and withdrawal symptoms are not seen when
usage is discontinued “Doctors are just the same as lawyers; the only difference us
o Psychic dependence may occur among marijuana users that lawyers merely rob you, whereas doctors rob and kill you
Marijuana is a non-lethal drug to human subject too”
-- Anton Chekhov, Ivanov
Subjective Effects of Marijuana
A feeling of lightness of the extremities, followed by "rushes” of
warmth and well-being  sense of relaxation and mild euphoria

[JC Checked by: Page 17 of


ACADs L

MEDICAL JURIS PRUDENCE o Primarily concerned with the standardization and regulation of
Lectured by: Dr. ALBERT D. REBOSA, M.D., LL.B. medical education
🠚 PROFESSIONAL
🠚 REGULATIONS COMMISSIONS
PHYSICIAN
o To have general supervision and regulation of all professions
“ls a person who after completing his secondary education follows a requiring examinations which includes the practice of medicine
prescribed course of medicine at a recognized university or medical school, at 🠚 BOARD OF MEDICINE
the successful completion of which, is legally licensed to practice medicine by o Its primary duties are to give examinations for the registration of
the responsible authorities and is capable of undertaking the prevention, physicians and supervision, control and regulation of the practice of
diagnosis, and treatment of human illness by the exercising independent medicine
judgment and without supervision.” -WHO 🠚 BOARD OF MEDICAL EDUCATION
o Composition:
LEGAL MEDICINE  Chairman - Secretary of Education
 Members - Secretary of Health
🠚 Branch of medicine that deals with the application of medical knowledge to  Director, Bureau of Private Schools
the purposes of law and in the administration of justice  Chairman, Board of Medicine
 Representative, PMA
MEDICAL JURISPRUDENCE  Council of Deans, APMC
 Dean, UP-College of Medicine
🠚 Deals with the aspect of law and legal concepts in relation with the practice o Functions:
of medicine  To determine and prescribe the requirements for admission into
🠚 Includes: a recognized college of Medicine
o Licensure and regulatory laws;  To determine and prescribe requirements for the minimum
o Physician-patient-hospital relationship together with the other physical facilities
paramedical personnel, their rights, duties and obligations  To determine and prescribe the minimum number and
o Liabilities for non-compliance with the law qualifications of teaching personnel
 To determine and prescribe the minimum required curriculum
PURPOSES  To authorize the implementation of experimental curriculum
🠚 To protect the public from charlatans  To accept applications for admission to a medical school
🠚 To promote professionalism and foster professional interrelationship  To select, determine and approve hospitals for training
🠚 To develop awareness of the rights, duties and obligations of the patient,  To promulgate, prescribe and enforce the necessary rules and
physician, and the hospital regulations
🠚 To control the increasing number of medical malpractice suits against
physicians LICENSURE AND REGULATORY LAWS
🠚 To explain the purpose and procedure of certain legislation 🠚 Composition:
🠚 To study the need to amend, repeal our health care laws in harmony with o Commissioner
the recent scientific and social development o Two Associate Commissioner
🠚 Exercise of Power and Functions of the Commission: exercise general
ADVERSARIAL TRIAL SYSTEM administrative, executive and policy-making functions for the whole agency

🠚 Philippine courts is a court litigation where there is competition of


inconsistent version of facts and theories in law during trial
🠚 Each party to the contest is given equal opportunity to investigate the case,
gather and present all proofs in support of his allegation, and give
argument that his contention is correct
🠚 Ultimate purpose is for a just solution
🠚 It often undermines the pursuit of truth as the opposing parties seek to win
at all cost without the obligation to reveal the facts which may be
detrimental to their case. The lawyer aims to win the fight not to help the
court discover facts or establish the truth.
SOURCES OF LAW

🠚 Constitution
🠚 Laws enacted by the legislative body
🠚 Decrees, Orders, Proclamation, Letters, CA, BP, RA
🠚 Administrative acts, orders, Rules and Regulation
🠚 Local customs
🠚 Generally accepted principles of International law
LAW AND THE PRACTICE OF MEDICINE

🠚 The State must maintain high standard of practice by setting up rules and
regulations with regards to qualifications and procedure for the admission
to the profession. These are legal safeguards to guarantee the safety of
the patient and impose liability to the practitioner who through his act or
omission causes damage or injury to the health and welfare of the patient
🠚 The right to regulate the practice of medicine is based on the police power
of the state
LICENSURE AND REGULATORY LAWS

🠚 ADMINISTRATIVE BODIES
🠚 BOARD OF MEDICAL EDUCATION

by: P.G.Montanio & Page 1 of


ACADs L

BOARD OF MEDICINE
🠚 Composition:
o Six members appointed by the president from a list submitted by
the Executive Council of the PMA
🠚 Qualifications:
o Natural-born citizen
o Duly-registered physician
o In the practice of medicine for at least 10 years
o Of good moral character and of recognized standing in the medical
profession as certified by PMA
o Not a member of any faculty of any medical school (including
any pecuniary interest)

POWERS, FUNCTIONS AND RESPONSIBILITIES


🠚 To determine and prepare the contents of the licensure examinations
🠚 To promulgate rules and regulations for the proper conduct of the
examinations, correction and registration
🠚 To administer oath
🠚 To study the conditions affecting the practice of medicine;
🠚 To investigate violations, issue summons, subpoena and subpoena
duces tecum
🠚 To conduct hearings or investigations of administrative cases filed before
them
🠚 To promulgate decisions on such administrative cases subject to the
review of the Commission
🠚 To issue certificate of registration
🠚 To suspend, revoke or reissue certificate of registration for causes
provided by law or by the rules and regulations promulgated
🠚 To promulgate, with the approval of PRC, rules and regulations in
harmony with the provisions of the Medical Act of 1959 and necessary for
the proper practice of medicine

by: P.G.Montanio & Page 2 of


ACADs L
🠚 Nurse
ADMISSION TO THE PRACTICE OF MEDICINE anesthetists

PREREQUISITES BY PROVISION OF LAW (SEC.11, ART.III; MEDICAL ACT OF 1959 AS


AMENDED)
1. Minimum age requirement: at least 21 years of age
2. Proper Educational Background 🠚 Any medical student duly enrolled in an approved medical college
o Requirements for Admission in the College of Medicine 🠚 dentist
o Holder of a Bachelor’s degree; 🠚 physiotherapist
🠚 optometrist
o Not convicted of any crime involving moral turpitude
🠚 Any person who renders any service gratuitously in cases of emergency or
o Certificate of Eligibility from the Board of Medical Education in places where the services of a physician, nurse or midwife are not
o Good moral character available
3. Examination Requirements: must have passed the corresponding Board 🠚 Any person who administers or recommends any household remedy as per
Examination classification of existing Pharmacy Laws
o Preliminary Examination 🠚 Prosthetist
 At least 19 years of age
 Of good moral character FAITH HEALING
 have completed the first two years of the medical course
o Final or Complete Examination 🠚 There is nothing in the Medical Act of 1959 exempting it from the definition
o Citizen of the Philippines or of any country who has submitted of the acts which constitute practice of medicine
competent and conclusive 🠚 Acted in pursuance of his religious belief and with the tenets of his church
 documentary evidence confirmed by the DFA showing that his he professes, not deemed to be a practice of medicine but part of his
country’s existing laws permit citizens of the Philippines to religious freedom (freedom to believe and freedom to act in accordance
practice medicine under the same rules and regulations with one’s belief)
governing citizens thereof (RECIPROCITY RULE)
4. Holder of certificate of registration ILLEGAL PRACTICE OF MEDICINE (SEC.28, ART.IV; MEDICAL ACT OF 1959 AS AMENDED)
o No issuance to any candidate who has been:
 Convicted by a court of competent jurisdiction of any crime 🠚 Practice of medicine by any person not qualified and not duly-admitted to
involving moral turpitude perform medical acts in compliance with law.
 found guilty of immoral or dishonorable conduct after
investigation by the Board of Medicine PENALTIES
 Declared to be of unsound mind 🠚 Any person found guilty of “illegal practice” shall be punished by
o A fine of not <1,000 pesos or >10,000 pesos
SCOPE OF EXAMINATION o Subsidiary imprisonment in case of insolvency or by imprisonment of
🠚 Preliminary - Anatomy and Histology, Physiology, Biochemistry, not <1yr or >5yrs
Microbiology and Parasitology o Or both such fine and imprisonment, in the discretion of the court
🠚 Final - Pharmacology and Therapeutics, Pathology, Medicine, Obstetrics
and Gynecology, Pediatrics and Nutrition, Surgery and Ophthalmology, QUALIFIED TO PRACTICE MEDICINE IN THE PHILIPPINES
Otorhinolaryngology, Preventive Medicine and Public Health, Legal
Medicine, Ethics and Medical Jurisprudence 🠚 Those who have complied with the prerequisites to the practice of
medicine in accordance with Sec. 8, Art. III, Medical Act of 1959 as
amended
PRACTICE OF MEDICINE
🠚 Those who can have limited practice without any certificate of registration
🠚 What is the “practice of medicine”? in accordance with Sec.12, Art. III, Medical Act of 1959 as amended:
o It is a privilege or franchise granted by the State to any person to o Exclusive consultation in specific and definite cases
perform medical acts upon o Attached to international bodies to perform certain definite work in
 Compliance with law, that is, the Medical Act of 1959 as the Philippines
amended which has been promulgated by the State in the o Commissioned medical officers stationed in the Philippines in their
exercise of police power to protect its citizenry from unqualified own territorial jurisdiction
practitioners of medicine o Exchange professors in special branches of medicine
🠚 It is diagnosing and applying and the usage of medicine and drugs for o Foreign medical students who have completed the first four years of
curing, mitigating, or relieving bodily disease or conditions medical course, graduates of medicine and registered nurses who
may be given limited and special authorization by the DOH
ACTS CONSTITUTING THE PRACTICE OF MEDICINE 🠚 “Balikbayan” Physicians pursuant to PD 541, Allowing Former Filipino
Professionals to Practice Their Respective Professions in the Philippines
🠚 Pursuant to Sec.10, Art. III of the Medical Act of 1959 as amended: o Provisions:
A) paid to him directly or through another, or even without the same,  Of good standing prior to their departure and in their adopted
physically examine any person, and diagnose, treat, operate or country
prescribe any remedy for human disease, injury, deformity, physical,  Have registered with PRC and paid their professional fee
mental, psychical condition or any ailment, real or imaginary,  Pay the corresponding income tax
regardless of the nature of the remedy or treatment administered, 🠚 Foreign physicians qualified to practice by Reciprocity Rule or by
prescribed or recommended endorsement
B) who shall by means of signs, cards, advertisement, written or printed 🠚 Medical Students pursuant to Sec.11 (a) and Sec.12 (d), Art.III, Medical Act
matter, or through the radio, television or any other means of of 1959 as amended
communication, either offer or undertake by any means or method to 🠚 Limited practitioners of medicine: Those that are governed by specific
diagnose, treat, operate, or prescribe any remedy for human disease, licensure laws
injury, deformity, physical, mental or psychical condition
C) Who shall falsely use the title of M.D. after his name, shall be RATIONALE WHY ARTIFICIAL PERSONE CANNOT PRACTICE MEDICINE
considered as engaged in the practice of medicine
🠚 Cannot be subjected to licensure examinations as required by law
NOT CONSIDERED TO CONSTITUTE A PRACTICE OF MEDICINE 🠚 Practice of medicine may be employed and controlled by unqualified
physicians
BY DESCISION OF COURT 🠚 Impairing professional relationship between the patient and the physician
🠚 One who takes BP reading 🠚 Deprivation of free choice of physicians
🠚 Application of medicated massage
🠚 Hospital

by: P.G.Montanio & Page 3 of


ACADs L

CONSTITUTION DUTIES AND OBLIGATIONS OF PHYSICIANS IN A PHYSICIAN-PATIENT


“ls that written instrument enacted by direct actions of the people by which RELATIONSHIP
the fundamental powers of the government are established, defined, and by 🠚 He should possess the knowledge and skill of which an average physician is
concerned; General practitioner vs Specialist
which these powers are distributed among the several departments for their
🠚 He should use such knowledge and skill with ordinary care and diligence
safe and useful exercise for the benefit of the body politic.” −Justice Malholm
o “locality rule” − the standard of care is measured by the degree of
care in the locality
PURPOSES (11Am. Jur. 606)
o “similar locality rule” − diligence is determined when the other
🠚 To prescribe the permanent framework of a system of government physicians in the locality or similar locality could have acted the same
🠚 To assign respective powers and duties to the several departments way
🠚 To establish certain first principles on which the government is founded o “national standard of care” - the diligence is determined on what is
applicable on a national standard basis
ESSENTIAL PARTS 🠚 He is obliged to exercise the best judgment
🠚 Constitution of Liberty 🠚 He has the duty to observe utmost good faith
🠚 Constitution of Government
🠚 Constitution of Sovereignty PHYSICIAN-PATIENT REALTIONSHIP “DOES NOT” IMPLY GUARANTY OR
PROMISE THAT:
🠚 The treatment will be successful
AMENDEMENTS 🠚 The treatment will benefit the patient
🠚 Are isolated or piecemeal changes in the constitution while REVISION is the 🠚 The treatment will produce certain result
revamp or the rewriting of the entire instrument 🠚 The treatment will not harm the patient
🠚 The physician will not commit errors in an honest way
PHYSICIAN-PATIENT RELATIONSHIP
🠚 Contract - is the meeting of minds between two persons whereby one DUTIES AND OBLIGATIONS OF PHYSICIANS IN THE COURSE OF A
binds himself with respect to the other, to give something or to render PHYSICIAN- PATIENT RELATIONSHIP
some service (Art.1305, NCC) 🠚 He must give an honest medical history
🠚 Nature of the relationship: 🠚 He must inform the physician of what occurred in the course of the
o Consensual - based on mutual consent both parties treatment
o Fiduciary - based on mutual trust and confidence 🠚 He must cooperate and follow the instructions, orders and suggestions of
the physician
REQUISITES OF A CONTRATUAL RELATIONSHIP 🠚 He must state whether he understands the contemplated course of action
🠚 He must exercise the prudence to be expected of an ordinary patient under
🠚 Consent − manifested by the meeting of the offer and the acceptance upon the same circumstances
the thing and the cause which are to constitute the contract (Art.1319,
NCC) STAGES OF A PHYSICIAN-PATIENT RELATIONSHIP
🠚 Object − the subject matter of the contract which is the medical service
which the patient wants to be rendered to him by his physician COMMENCEMENT
🠚 Cause − is the consideration or the factor that instigated the physician to 🠚 It is the very time the physician is obliged to comply with the legal duties
render the medical service to the patient, which could be remuneratory or and obligations to his patient
an act of liberality
TERMINATION
FORMS OF PHYSICIAN-PATIENT RELATIONSHIP 🠚 It is the time when the duties and obligations by a physician to his patient
🠚 Expressed − explicitly stated orally or in writing ceases
🠚 Implied − the existence can be inferred from the acts of the contracting 🠚 The following are some ways of termination of the relationship
parties o Recovery of the patient or when the physician considers that his
medical services will no longer be beneficial to the patient
INSTANCES WHERE THERE IS “NO” PHYSICIAN-PATIENT RELATIONSHIP BY o Withdrawal of the physician provided:
DECISION OF COURTS  with consent of the patient, and
🠚 Pre-employment PE for purposes of determining whether an applicant is  patient is given ample time and notice
suitable for employment o Discharge of the physician by the patient
🠚 PE for eligibility for insurance o Death of either party
🠚 Physician appointed by court to examine the accused o Incapacity of the physician
🠚 In performing an autopsy o Fulfillment of the obligations stipulated in the contract
🠚 Casual consultation in an unordinary place o In emergency cases, when the physician of choice of the patient is
already available or when the condition of emergency ceases
PSYCHOLOGICAL PATTERNS OF PHYSICIAN-PATIENT RELATIONSHIP Expiration of the period as stipulated
o Mutual agreement for its termination
ACTIVITY-PASSIVITY RELATION
🠚 No interaction between physician and patient because the patient is unable RIGHTS OF PHYSICIANS
to contribute activity
🠚 This is characteristic in emergency cases when the patient is unconscious 🠚 INHERENT RIGHTS
o To choose patients
GUIDANCE-COOPERATION RELATION o To limit practice of medicine
🠚 Patient is conscious and suffering from pain, anxiety and other distressing o To determine appropriate management procedures
symptoms, he seeks help and willing to cooperate o To avail of hospital services
🠚 The physician is in a position of trust 🠚 INCIDENTAL RIGHTS
o Right of way while responding to emergency
o Right of exemption from execution of instruments
MUTUAL PARTICIPATION
🠚 It is in the nature of a negotiated agreement between equal parties o Right of library to hold certain public/private offices to perform
certain services
o Right to compensation
o Right to membership in medical societies
🠚 RIGHTS GENERALLY ENJOYED BY EVERY CITIZEN

by: P.G.Montanio & J.Wang Page 3 of 16


ACADs L

INHERENT RIGHTS 🠚 Related provisions:


 Art IV, Sec. 2, Code of Ethics
RIGHT TO CHOOSE PATIENTS
“…should willingly render gratuitous service to a colleague, to his
🠚 Any person who is given right to practice medicine is not obliged to wife and minor children or even parents provided the latter are
practice medicine aged and being supported by the colleague. He should however,
🠚 Cannot be compelled to accept professional employment be furnished the necessary traveling expenses…this provision shall
not apply to physicians who are no longer in the active practice…”
Note: The law does not give any qualification the right of the physician to
o Doctrine of Unjust Enrichment
choose his patient, however, the Code of Medical Ethics and RA 6615 provides  No one must enrich himself at the expense of others
otherwise in cases of emergency  “service rendered service paid”
🠚 Related provisions: 🠚 Simple Contractual Fee − specifically stating the value of such medical
 Art II, Sec.2 Code of Ethics KINDS OFeither
service, MEDICAL FEES
orally or in writing
“…free to choose whom he will serve...always respond to any 🠚 Retainer Fee − measured by the space of time rendered by patient
request for his assistance in emergency….” 🠚 Contingent Fee − depends upon the failure of the treatment instituted
 Art. II, Sec.3 Code of Ethics 🠚 Dichotomous Fee (Fee splitting) − the physician may require the services
“ln cases of emergency ...a physician should administer at least of a person who may act as agent to solicit patients, and the agent will
first aid treatment and then refer to a more qualified and share in the medical fee.
competent physician …”
 Sec.1 RA 6615 UNETHICAL FEES (ART.III, SEC 5, CODE OF ETHICS)
“All government and private hospitals…are required to render
immediate emergency medical assistance…” “...solicitation of patients, directly or indirectly, through solicitors or agents, is
 Sec.24 No.12, Medical Act of 1959 unethical.”
”….although the ethical rule obliges a physician to attend to an 🠚 Straight Fee:
emergency, his failure to respond to it may not make him liable if o For the amount tendered by the patient to the physician, the latter
in so doing, there is a risk to his life” shall be responsible for the payment of hospital bill, lab fees,
“Refusal of a physician to attend to a patient in danger of death is medicines, and other incidental expenses
not a sufficient ground for revocation or suspension of his o This kind of fee is unethical because the amount wagers with the
registration if there is a risk to the physician’s life” unforceable contingencies

RIGHT TO LIMIT HIS MEDICAL PRACTICE


🠚 field of specialty
🠚 private clinic or hospital
🠚 within a political/geographical boundary
🠚 certain days of the week/hours of the day
🠚 certain class of people
🠚 with due regard to dictate of conscience
🠚 retirement
🠚 imposed by the public, religion, professional ethics, medical society, law,
contract

RIGHT TO DETERMINE THE APPROPRIATE MANAGEMENT OR PROCEDURE


🠚 Doctrine of Superior Knowledge:
o The physician has superior knowledge and the patient just follows
orders or instructions and usually places himself in the command and
control of the physician

RIGHT TO AVAIL OF HOSPITAL SERVICES

INCIDENTAL RIGHTS
RIGHT OF WAY WHILE RESPONDING TO THE CALL OF EMERGENCY

RIGHT OF EXEMPTION FROM EXECUTION OF INSTRUMENTS AND LIBRARY


🠚 Rule 39, Sec.12, Rules of Court

RIGHT TO HOLD CERTAIN PUBLIC AND PRIVATE OFFICES


🠚 Which can only be filled-up by physicians

RIGHT TO PERFORM CERTAIN CERVICES WITH DU COMENSATION

RIGHT TO MEMBERSHIP ON MEDICAL SOCIETIES


🠚 Any qualified medical practitioner has the right to become a member of the
PMA through one of its component society
🠚 Membership in a medical society may be voluntary or involuntary

Note: Philippine Medical Care Act of 1969(RA 6111 as amended) provides that
membership to the PMA is a requirement before a physician can practice
medicine under the Medicare.

RIGHT TO COMPENSATION
🠚 Based on the physician-patient contractual relationship
🠚 Existence of friendship does not imply gratuitous services

by: P.G.Montanio & Page 4 of


ACADs L

METHODS OF COLLECTION OF PAYMENTS FOR MEDICAL SERVICES


🠚 Extra judicial − billing or referral to a bill collection agency
🠚 Judicial methods

JUDICIAL METHOD: FATCS TO BE PROVEN IN COURT


🠚 Physician employed is duly qualified and licensed
🠚 The professional fee demanded is reasonable
🠚 The person The physician has rendered professional service to the patient
🠚 liable for the payment is the defendant

PATIENT’S OBLIGATION TO PAY


🠚 The obligation to pay devolves on the patient himself provided he is of legal
age, of sound mind and has the capacity to enter into a contractual relation
🠚 If the patient dies or becomes legally incapacitated to pay, medical fee
shall be made from the following persons in order:
1. Spouse
2. Descendants, of the nearest degree
3. Ascendants, of the nearest degree
4. Brothers and sisters
🠚 Implied promise to pay the physician by the benefactor of the medical
services rendered in emergency cases.

INTANCES WHERE THE PHYSICIAN “CANNOT” RECOVER PROFESSIONAL FEES


🠚 Agreement that the service is gratuitous
🠚 In government charity hospitals, health centers and other similar health
units
🠚 Rendered in private charitable institutions if expressly gratuitous to the
indigent patients
🠚 Waiver on the part of the physician
🠚 Breach of contract
🠚 When the physician cannot charge the patient pursuant to the Code of
Ethics
🠚 Those covered by Phil health
🠚 Medical services rendered under a contract of employment unless
expressly provided otherwise

🠚 pursuant to the provisions of Art.III, Bill of Rights, Philippine Constitution RIGHT GENERALLY ENJOYED BY EVERY CITIZEN
1987

by: P.G.Montanio & Page 5 of


ACADs L

RIGHTS OF PATIENTS 🠚 Other person who may give consent having substitute parental authority

🠚 Right to give consent to diagnostic and treatment procedures Note:


🠚 Right to religious belief 🠚 Consent of minor is not valid if the procedure will not benefit him
🠚 Right of privacy 🠚 Expressed refusal of a minor to surgery shall not prevail over the existing
🠚 Right to disclosure of information emergency
🠚 Right to confidential information 🠚 Doctrine of parens patriae, the court may grant consent for the minor
🠚 Right to choose his physician
🠚 Right of treatment SUBJECT MATTER “MUST” BE LEGAL
🠚 Right to refuse necessary treatments 🠚 The subject matter or procedure applied to the patient and which the
patient consented must not be that which the law penalizes or against
RIGHT TO GIVE CONSENT TO DIAGNOSTIC PROCEDURES public policy
🠚 Patient is the final arbiter of what must be done with his body
🠚 Obligations of the Physician to Inform the Patient: RIGHT TO RELIGIOUS BELIEF
o Diagnosis  Art. III, Sec. 5, Philippine Constitution
o General nature of the contemplated procedure “No law shall be made respecting an establishment of religion or
 Risk involved prohibiting the free exercise thereof. The free exercise and enjoyment of
 Prospect of success religious profession and worship, without discrimination or preference,
 Potential danger if not applied shall forever be allowed. No religious test shall be required for the
 Alternative methods of treatment exercise of civil or political rights.”

BASES OF CONSENT
🠚 The physician-patient relationship is fiduciary in nature
🠚 Patient’s right to self-determination
🠚 Contractual relationship

PURPOSES OF CONSENT
🠚 To protect the patient from unnecessary/unwarranted procedure applied
to him without knowledge
🠚 To protect the physician from any consequences for failure to comply with
legal requirements

INSTANCES WHEN CONSENT IS “NOT” NECESSARY


🠚 ln cases of emergency, there is an “implied consent” or the physician is
“privilege because he is reasonably entitled to assume consent
🠚 When the law made it compulsory for everyone to submit to the procedure

REQUISITES OF A VALID CONSENT


🠚 Informed or enlightened consent
🠚 Voluntary
🠚 Subject matter must be legal

FORMS OF CONSENTS
🠚 Expressed consent − written or oral
🠚 Implied consent may be deduced from the conduct of the patient

SCOPE OF CONSENTS
🠚 General or Blanket consent
🠚 Limited or conditional consent
🠚 Non-liability or exculpatory clause

INFORMED OR ENLIGHTEN CONSENT


🠚 Awareness and assent

RIGHT TO TREATMENT

by: P.G.Montanio & Page 6 of


ACADs L

RIGHT TO PRIVACY

RIGHT TO DISCLOSURE OF INFORMATION


🠚 The physician-patient relationship being fiduciary in nature, the physician is
obliged to make full and frank disclosure to the patient or any person who
may act on his behalf all he pertinent facts relative to his illness

RIGHT TO CONFIDENTIAL INFORMATION


🠚 Statutory Privileged Communication
Pursuant to the Rules of Court, Rule 130, Sec. 24(c), a person authorized
to practice medicine, surgery or obstetrics cannot in a civil case, without
the consent of the patient, be examined as to any information which he
may have acquired in attending such patient in a professional capacity,
which information was necessary to enable him to act in that capacity,
and which would blacken the character of the patient.

🠚 Ethical/Professional Confidential Information


Pursuant to Art.II, Sec.6, Code of Medical Ethics, the medical practitioner
should guard as a sacred trust anything that is confidential or private in
nature that he may discover or that may be communicated to him in his
professional relation with his patients, even after death. He should
never divulge this confidential information, or anything that may reflect
upon the moral character of the person involved, except when it is
required in the interest of justice, public health or public safety.

INSTANCES WHERE CONFIDENTIALITY IS NOT APPLICABLE


🠚 When such disclosure is necessary to serve the best interest of justice
🠚 When the disclosure will serve public health and safety
🠚 When the patient waives its confidentiality

RIGHT TO CHOOSE HIS PHYSICIANS


🠚 full disclosure of facts and willingness of the patient to submit

QUANTUM OF INFORMATION NECESSARY TO FORM THE BASIS OD A VALID


CONSENT 🠚 In emergency cases the patient has the right to treatment
 Sec. 1, 1st par, RA 6615
🠚 Nature of his condition;
“…all government and private hospital or clinics duly licensed to operate
🠚 Natured of proposed treatment or procedure;
are required to render immediate medical assistance and to provide
🠚 Possible alternative methods;
facilities and medicine within its capabilities to patients in emergency
🠚 Risk involved;
cases who are in danger of dying and or suffered serious physical
🠚 Chances of success or failure
injuries…”
 Art II, Sec.3 of the Code of Medical Ethics
Note: Consent must be given freely or voluntarily
“ln cases of emergency, wherein immediate action is necessary, a
physician should administer at least 1 st aid treatment and then refer the
PERNSONS WHO CAN GIVE CONSENT
patient to a more qualified and competent physician if the case does not
🠚 Patient fall within his particular line.”
🠚 If patient is minor, consent must be obtained from the parents/guardian
🠚 In the absence of the parents, consent of the grandparents must be RIGHT TO REFUSE TREATMENT
obtained, paternal grandparents having preference 🠚 In the legal sense, every man of adult age and of sound mind has the right
🠚 In the absence of parents and grandparents, eldest brother or sister, to determine what must be done in his own body
provided one is of age and not disqualified by law to give consent

by: P.G.Montanio & Page 7 of


ACADs L
🠚 A man is the master of his own self and may expressly prohibit a life-saving
 Violation of any of the Code of Ethics as approved by the PMA
surgery or medical treatment
🠚 Doctrine of parents patria, the State has the right to assume guardianship
CRIMINAL LIABILITIES
when the child is neglected by the parents to have the child treated, and
parents have no right to base it on religious beliefs or any other grounds. 🠚 A criminal act is an outraged to the sovereignty of the State so it must be
🠚 When the law provides for treatment, the patient has no right to refuse instituted in the name of the sovereign people as party-plaintiff (People of
treatment the Philippines vs. X)
🠚 The social commitment of the physician is to sustain life and relieve 🠚 Quantum of evidence is proof beyond reasonable doubt. This does not
suffering. Where the performance of one’s duty conflicts with the other, mean absolute certainty as excluding possibility of error but only mean
the choice of the patient, or his family or legal representative if moral certainty, or that degree which produces conviction in an
incompetent to act on his own behalf, should prevail. In the absence of the unprejudiced mind (Rule 133, Sec. 2, Rules of Court)
patient’s choice or authorized proxy, the physician must act in the best 🠚 Presumption of Innocence and Equipoise Rule
interest of the patient 🠚 Conviction of a physician, aside from imprisonment and/or fine, his
registration may be cancelled or revoked if:
🠚 Administrative”  The law imposes revocation of the license
LIABILITIES OF PHYSICIANS  The crime wherein the physician was found guilty involved moral
o Right to practice is temporarily withdrawn from the physician
turpitude
INCIDENTAL TO THE PRACTICE OF MEDICINE
o A valid exercise of the police power of the State
o Laws: Medical Act of 1959 as amended including the Code of Ethics  Imprudence and Negligence- Art.365, Revised Penal Code
and Rules and Regulations of the PRC “Any person who, by reckless imprudence, shall commit any act which,
o Penalty: reprimand, suspension, to revocation of license had it been intentional, would constitute a grave felony, shall suffer the
🠚 Criminal penalty of arresto mayor in its maximum period to prison correctional in
o An act or omission which constitute a crime by the physician its medium period, if it would have constituted a less grave felony, the
o Laws: Revised Penal Code and other special laws penalty of arresto mayor in its minimum periods shall be imposed; if it
o Penalty: imprisonment and/or fine would have constituted a light felony, the penalty of arresto menor shall
🠚 Civil be imposed…”
o Awarded against a physician to compensate for the injury he suffered 🠚 Imprudence − deficiency of action or failing to take the necessary
on account of the physician’s act or omission as a breach of the precaution once they are foreseen.
contractual relationship of both parties 🠚 Reckless imprudence − voluntary, without malice, doing or failing to do an
o Laws. Civil Code of the Philippines and other related laws act which results from material damage by reason of inexcusable lack of
o Art. 100, RPC states that “Every person criminally liable is civilly precaution on the part of the person performing the act, taking into
liable.” Penalty: damages consideration his employment or occupation, degree of intelligence,
physical condition and other circumstances regarding persons, time and
ADMINISTRATIVE LIABILITIES place.
🠚 Simple imprudence − consist in neither the lack of precaution displayed in
🠚 Quantum of evidence needed: substantial evidence, such relevant evidence
those cases in which the damage impending to be caused is not immediate
as a reasonable mind might accept as adequate to support a conclusion
nor the danger clearly manifest.
🠚 Administrative due process:
🠚 Negligence- indicates a deficiency of perception or when the wrongful act
 Right to hearing
maybe avoided by paying proper attention and using due diligence in
 Tribunal must consider the evidence presented
foreseeing them.
 Decision must have something to support itself
 Evidence must be substantial
 Decision must be based on the evidence adduced at the hearing, or at
least contained in the record and disclosed to the parties
 The Board or its judges must act on its or their independent
consideration of the facts and the law of the case, and not simply
accept the views of a subordinate in arriving at a decision. (Ang Tibay
vs. CIR)

GROUNDS FOR ADMINISTRATIVE LIABILITIES


 Sec.24, Art.III, Medical Act of 1959 as amended
🠚 Personal Disqualifications:
 Immoral or dishonorable conduct;
 Insanity;
 Gross negligence, ignorance or incompetence resulting in an injury to
or death of the patient;
 Addiction to alcoholic beverages or to any habit-forming drug
rendering him incompetent to practice medicine.
🠚 Criminal Acts:
 Conviction by a court of competent jurisdiction of any criminal
offense involving moral turpitude
 Fraud in the acquisition of the certificate of registration
 Performance of or aiding in any criminal abortion
 Knowingly issuing false medical certificate
 Aiding or acting as dummy of an unqualified or unregistered person
to practice medicine
🠚 Unprofessional Conduct:
 False or extravagant or unethical advertisements wherein other
things than his name, profession, limitation of practice, clinic hours,
office and home address, are mentioned
 Issuing any statement or spreading any news or rumor which is
derogatory to the character and reputation of another physician
without justification

by: P.G.Montanio & Page 8 of


ACADs L

🠚 A civil suit filed against physician and/or hospitals is premised on CIVIL LIABILITIES
recovery of damages for their wrongful act or of employees.
🠚 Cause of action for damages is based on:
 Breach of Contract
 physician-patient relationship
 specific stipulations in the contract
 In an action for breach of contract, the negligence of the
doctor is not an issue, for if the doctor makes contract to effect
a cure and fails to do so, he is liable for breach of contract even
though he uses the highest possible professional skill.
 Tort (Quasi-delict)
 Legal wrongdoing independent of a contract
 Primary basis is negligence or fault of the physician as the one
directly responsible for the injury sustained by the patient
 Ordinarily, any malpractice action is based on torts or quasi-
delict in as much as negligence is usually a ground for injury
 Art. 2176 of the Civil Code provides that:
“Whoever by act or omission causes damage to another, there
being fault or negligence, is obliged to pay for the damage done.
Such fault or negligence, if there is no pre-existing contractual
relation between the parties, is called quasi-delict…”

🠚 Failure of a physician to properly perform the duty which devolves


MEDICAL MALPRACTICE
upon him in his professional relation to his patient which results to injury.
🠚 It may be defined as bad or unskillful practice of medicine resulting to
injury of the patient or failure on the part of the physician to exercise the
degree of care, skill and diligence, as to treatment in a manner contrary to
accepted standards of medicine resulting to injury to the patient.
🠚 Elements:
 The physician has a duty to the patient

by: P.G.Montanio & Page 9 of


LEGAL
MEDICINE

1.A body was found in an abandoned lot in North The physician should know better--this is the main
Fairview. At the time, the whole body was found to be thought under the doctrine of foreseeability. make
stiffened. The time of death is at least sure to be familiar with all the doctrines.
a. 24 hours 3. A mass of precepts that determine and regulates the
b. 3 hours relation of assistance, authority and obedience between
c. 12 hours members of a family and those which exist among
d. 18 hours members of a society for the protection of private
Rationale: interest:

a. Criminal law

b. Remedial law

c. Civil law

d. Special law

Rationale:

Cadaveric Rigidity or RIGOR MORTIS - The


whole body becomes rigid due to muscular
contraction, which develops 3 – 6 hours after
death and may last for 24 – 36 hours.

Biochemical changes in the body produce


stiffening, the rigor mortis that usually appears
4.A medical witness in the witness stand who disrespects
within 2 to 6 hours after death. Rigor mortis
the lawyer by challenging him to a fistfight while on the
begins in the muscles of the jaw and neck and
witness stand may be declared guilty of
proceeds downwards in the body to the trunk and
extremities and complete within 6 to 12 hours. a. conduct unbecoming a medical witness
2. A delirious and deranged patient was left alone in his b. criminal act by omission
bed as a result of which he stood up and jumped from
the window and later died. The liability of the doctor is c. direct contempt
under the doctrine of: d. indirect contempt
a. res ipsa loquitor Rationale:
b. rescue doctrine
Sec. 1, Rule 71 o Direct Contempt Punished
c. captain-of-the-ship
Summarily – A person guilty of misbehavior in the
d. Foreseeability
presence of or so near a court as to obstruct or
interrupt the proceedings before the same,
Rationale: including disrespect toward the court, offensive

1
AY 2020
LEGAL
MEDICINE

personalities toward others, refusal to be sworn or Rationale:


to answer as witness, or to subscribe an affidavit or
deposition when lawfully required to do so, may be Medical expert witnesses are physicians, nurses,
summarily adjudged in contempt by such court.” surgeons or other licensed practitioners whose
skills and experience qualify them to testify on a
particular medical area. In personal injury and
5. #A physician was injured while on a rescue mission medical malpractice lawsuits, attorneys often utilize
of victim of an accident. Supposed Mr. Dy is medical expert witnesses during both the discovery
responsible for the accident, he is liable to the and trial stages.
physician under the doctrine of: 7.A written record of evidence given orally and
transcribed in writing in the form of question by the
a. captain-of-the-ship interrogator, answered by the subject and signed by him
b. res ipsa liquitur later is termed:

c. both a. documentary evidence

d. neither b. oral testimony

Rationale: c. deposition
Doctrine of Res Ipsa Loquitor (common knowledge d. dying declaration
doctrine) is the nature of wrongful act is suggestive
Rationale:
of negligence.
Deposition involves the taking of sworn, out-of-
The captain of the ship doctrine, which has been
court oral testimony of a witness that may be
interpreted to mean that the surgeon's mere
reduced to a written transcript for later use in court
presence in the OR subjects him or her to legal
or for discovery purposes.
liability for everyone's negligence in that room (akin
to the responsibility of the captain of a ship who is
held responsible for everything that happens on
that ship) 8.A young man was stabbed in the abdomen
developed generalized peritonitis and died.
Peritonitis is the:

6.A physician who testifies in court on matters a. immediate cause of death


pertaining to his findings on his patient in the
course of their physician-patient relationship is b. underlying cause of death
considered c. proximate cause of death
a.an officer of the court d. manner of death
b.an expert medical witness

c.an ordinary witness d.amicus 9. An infant is defined in infanticide as

curiae a. A baby up to 2 years old


b. A newborn up to 28 days old

2
AY 2020
LEGAL
MEDICINE

c. A child less than 3 days old


d. A child less than 2 months old e.
10. As a general rule the following, EXCEPT ONE,
are true:
13. Doctrine of imputed negligence is synonymous
a. A negligent driver will be the person who with:
goes to jail and not his boss
a. vicarious liability
b. Doer of the act is the one responsible
b. borrowed servant doctrine
c. One is responsible for his own negligent act
c. rescue doctrine
d. My boss goes to jail for my negligent act.
d. none of the choices

Rationale:
11. As a witness in court, a physician may refuse to
DOCTRINE OF VICARIOUS LIABILITY -Doctrine
answer a question propounded to him if his answer
of Imputed Negligence/Command Responsibility. -
a. will be self-incriminatory Vicarious liability means the responsibility of a
person, who is not negligent, for the wrongful
b. will antagonize the judge conduct or negligence of another.
c. will not be relevant to the case at issue

d. will blacken the reputation of a friend 14. Dr. Cataman is a surgeon at Delaware Medical
Center. He wanted to do appendectomy in another
hospital. He called Dr. Bravo, a resident physician at
12. Cause of death could still not be determined the Delaware Medical Center to assist him. The
despite an autopsy done negligence of Dr. Bravo shall be imputed to Dr.
Cataman by reason of:
a. negative post-mortem
a. Independent contractor
b. negligent autopsy
b. Borrowed Servant Doctrine
c. negative autopsy
c. Vicarious liability
d. all of the choices
d. Fellow servant doctrine
Rationale:
Rationale:
Negative autopsy -is a post-mortem examination in
BORROWED SERVANT DOCTRINE • Ordinarily,
which a comprehensive analysis does not provide a
resident physicians, nurses and other personnel of
cause of death.
the hospital are employees or servants of the
Negligent autopsy – Cause of death is still unknown hospital; • In some instances, they are under the
because of so many errors committed during the temporary supervision and control of another other
autopsy than their employer while performing their duties; •
By fiction of law, they are deemed borrowed from

3
AY 2020
LEGAL
MEDICINE

the hospital by someone and for any wrongful act - Committed in presence of Chief Executive
committed by them during the period, their or Palace grounds
temporary employer must be held liable for the
discharge of their acts and duties; • In the - Scoffing at the corpse of the victim
determination whether one is a borrowed servant, it
is necessary that he is not only subjected to the
control of another with regard to the work done and 16. In the doctrine of vicarious liability, which of the
the manner of performing it but also that the work following is true?
to be done is for the benefit of the temporary
employer a. My employer is civilly responsible for
my negligent act

b. One is responsible for his own negligent act


15. Dr. K decided that his patient had suffered enough
from the pain of bronchogenic carcinoma. He decided c. A negligent driver will be the person who goes to jail
to end his suffering by giving him 100 mg of morphine. and not his boss
As a result of the injection, the patient dies. Dr. K is
prosecuted by the state. He is most probably guilty of d. Author of the act is the one responsible

Rationale:
a. Homicide
Vicarious liability is a legal doctrine that assigns liability
b. Giving assistance to suicide for an injury to a person who did not cause the injury but
c. Parricide who has a particular legal relationship to the person who
did act negligently.
d. Murder

Rationale:
Murder is the unlawful killing of another human 17. In which of the following instances can you
without justification or valid excuse invoke your right against self-incrimination?

Murder Aggravating circumstances: a. Finger printing


b. Submission of semen specimen
-Treachery c. Finger printing and taking of Photographs
d. Taking of photographs
– victim did not have the chance to defend self
Rationale:
- Defense wounds Submission of semen specimen can not be
forced by the judge
- Evident pre-meditation

- Use of superior force & night time to ensure


18. Increase of lactic acid, phosphoric acid and the
success
reaction become acidic in the muscles after death is
- Committed during conflagration or epidemic found in

a. postmortem rigidity

b. cadaveric rigidity
4
AY 2020
LEGAL
MEDICINE

c. both A and B 22. Kinds of death:


d. none of the choices a. somatic death

b. apparent death
19. It is found in the most dependent portion of the body
c. cellular death
involving the superficial layer of the skin, color is dull
red purplish, uniform nor elevated which appear after d. all of the choices
death;

a. postmortem lividity 22. Nilo Loko went home from the office early due to
b. postmortem suggillation a headache. When he walked into the bedroom, he
c. livor mortis found his wife having sex with his driver Totoy
d. all of the choices M. He got his gun and shot both his wife and Totoy. He is
e. none of the choices liable for:
Rationale:
a. Homicide
POST MORTEM LIVIDITY- It occurs in the most
extensive areas of the most dependent portions of b. Infanticide
the body after death Importance: to determine the c. Death Inflicted Under Exceptional
position of the body when the portion died Circumstances

d. murder
20. Its Medico-Legal importance is to approximate the
Rationale:
time of death:
Death or physical injuries inflicted under
a. rigor mortis
exceptional circumstances. — Any legally married
b. muscular contraction person who having surprised his spouse in the act
of committing sexual intercourse with another
c. cadaveric spasm person, shall kill any of them or both of them in the
d. all of the choices act or immediately thereafter, or shall inflict upon
them any serious physical injury, shall suffer the
Rationale: penalty of destierro.
Rigor Mortis -Utilized to approximate the time of
death -Generalized muscular contractions occur from
3 – 36 hours

Cadaveric spasm -Occurs immediately after


death -Useful to ascertain the circumstances of death

5
AY 2020
LEGAL
MEDICINE

23. Official autopsy is also known as a. requires a person to appear in court and produce a
person under his care and custody
a. Medicolegal autopsy
b. requires a person to appear in court
b. elective autopsy
c. requires a person to appear in court and
c. hospital-based autopsy
bring documents or object.
d. none of the choices
d. requires a person to appear and testify in court

24. Radiologists in a hospital are liable for damages


under the doctrine of: Rationale:

A subpoena duces tecum is an Order that


a. ostensible agent requires a witness to bring documents, books or
b. contributory negligence other items under his, her or their control, that he
c. res ipsa loquitor she or they is bound by law to produce into
d. none of the choices evidence.
Rationale:

Doctrine of ostensible agent states that 27. That kind of evidence necessary for conviction
pathologists, radiologists and anesthesiologists which proves the fact in dispute without the aid of any
are employees and at the same time inference or presumption and which correspond to the
independent contractors of the hospital. precise or actual point at issue:

a. circumstantial evidence
b. direct evidence
25. Reasons for the inadmissibility to the court of the
c. preponderance of evidence
result of Lie Detector examination are the following,
d. testimonial evidence
except:

a. there is no way to assure that a qualified examiner


administered the test (part of reasons) 28. The complete relaxation and softening of all the
muscles found immediately after death is termed:
b. polygraph techniques are still in
the experimental stage a. Cadaveric Rigidity

c. the test has not gained a degree of development b. Period of muscular flaccidity
beyond experimental stage this for me
c. Rigor Mortis
d. the test cannot be relied upon because
d. Stage of Primary flaccidity
the margin of error is at least 40%
Rationale:

At the moment of death, all of the muscles in the


body relax, a state called primary flaccidity.
26. Subpoena duces tecum

6
AY 2020
LEGAL
MEDICINE

29. The different changes that takes place in the Rationale:


muscles of the body after death:
Persons to make Medico-Legal Investigation:
a. stage of primary flaccidity
The inquest officer may request any of the following
b. stage of postmortem rigidity persons to perform the necessary autopsy or other
forms of investigation for the proper
c. stage of secondary flaccidity
administration of justice:
d. all of the choices
1. District health officers
e. none of the choices
2. Local health officers

30. The doctrine of “command responsibility” is 3. Members of the municipal board of health
applicable to which of the following? who are not a physicial or a “Cirujano
Ministrante” whenever a registered physician is
a. last clear chance
not
B. captain-of-the-ship
available
c. contributory negligence
4. Medical examiners of the City of Manila and
d. assumption of risk his assistants
Rationale: 5. Medico-legal officers of NBI
The captain of the ship doctrine, which has been 6. Medical staffs of hospitals, centers, and other
interpreted to mean that the surgeon's mere government physicians
presence in the OR subjects him or her to legal
liability for everyone's negligence in that room
(akin to the responsibility of the captain of a ship
who is held responsible for everything that 32. The medical witness who refuses to answer
happens on that ship) questions propounded to him may be cited for

a. perjury
31. The following are authorized to perform b. direct contempt
autopsies except
c. dishonorable conduct
a. City Health Officers
d. indirect contempt
b. Medicolegal Officers of the PNP Crime Rationale:
Laboratory
Direct Contempt Punished Summarily – A
c. Medical Staff of the NBI person guilty of misbehavior in the presence of or
d. None of the choices so near a court as to obstruct or interrupt the
proceedings before the same, including disrespect
toward the court, offensive personalities toward

7
AY 2020
LEGAL
MEDICINE
others, refusal to be sworn or to answer
as

8
AY 2020
LEGAL
MEDICINE

witness, or to subscribe an affidavit or deposition


when lawfully required to do so, may be summarily
adjudged in contempt by such court. 35. The surgeon with respect to the nurse under him
during a surgery is considered as:
33. The most prominent sign of death is
a. borrowed servant
a. cessation of heart action and circulation
b. captain-of-the-ship
b. insensibility of body and loss of power to move
c. the doctor who speaks for himself
c. cessation of respiration
d. ostensible agent
d. progressive fall of the body temperature

Rationale:
36. The term “res ipsa loquitor” literally means:
The progressive fall of the body temperature is one
of the most prominent signs of death due to the a. the patient speaks for himself
cessation of the metabolic processes inside the
body . b. the thing speaks for itself

c. the doctor speaks for himself

34. The state of the body in which there is complete and d. none of the choices
permanent cessation of the vital functions of the brain,
Rationale:
heart and lungs.
DOCTRINE OF RES IPSA LOQUITOR “The thing
a. Somatic death
speaks for itself”; nature of the wrongful act or
b. Molecular Death injury is suggestive of negligence.

c. Both of the choices

d. None of the choices 37. This is the deliberate and painless acceleration of
Rationale: death of a person usually suffering from an incurable
and distressing illness.
Somatic death / Clinical death - dead for less than
3 hours -Dead, but cells are still alive a. Homicide
b. Judicial Death
Molecular / Cellular death - dead for 3-6 hours - c. Suicide
Death also of the cells d. Euthanasia
Somatic death or clinical death – state of the body
in which there is complete, persistent and
continuous cessation of the vital functions of the 38. This is the instantaneous rigidity of the muscles
brain, heart and lungs which maintain life and which occurs at the moment of death due to extreme
health nervous tension, exhaustion and injury to the nervous
system.

a. Putrefaction
9
AY 2020
LEGAL
MEDICINE

b. Secondary flaccidity c. rescue doctrine


c. Cadaveric Spasm d. none of the choices
d. Rigor Mortis RATIONALE:
Rationale:
Borrowed servant rule is a legal doctrine in which
Cadaveric spasm, also known as an employer is held liable for the actions of a
postmortem spasm, instantaneous rigor, cataleptic temporary employee.
rigidity, or instantaneous rigidity, is a rare form of
muscular stiffening that occurs at the moment of
death and persists into the period of rigor mortis. 42. When the endotracheal tube is discovered to be
inserted into the esophagus as shown by X-ray, the
physician is liable under the doctrine of:

39. To find out the truth is an essential requirement for a. rescue doctrine
the administration of justice. Which of the following is
the commonly used method of deception-detection: b.res ipsa loquitor

c. foreseeability d.none
a. lie detection method
b. hypnotism of the choices
c. use of drugs
d. confession Rationale:

Res ipsa liquitor- the principle that the mere


occurrence of some types of accident is sufficient to
40. What instance shall an autopsy be performed on a imply negligence.
corpse?

a. Upon written request of police authorities


43. When the evidence presented corresponds to the
b. Upon written request of the next of kin precise point at issue and proves the fact in dispute
without the aid of any inference, it is called
c. Upon order of a competent court, mayor or
provincial or city prosecutor. a. circumstantial
d. All of the choices b. direct

c. tracing
41. When resident physicians and nurses are under the d. hearsay
temporary supervision of another physician during a
private surgery, the liability of the surgeon for the
negligence resident physician is covered by:

a. doctrine of foreseeability 44. When the patient is himself negligent causing


b. borrowed servant doctrine more damage to his existing injury, the doctrine
applicable is:

1
AY 2020
LEGAL
MEDICINE

a. captain-of-the-ship 47. Which of the following doctrines DOES NOT


b.res ipsa loquitor impose vicarious liability on the hospital?

c. contributory negligence a. Ostensible agent

d. assumption of risk b. Fellow servant doctrine

Rationale: c. Command Responsibility


DOCTRINE OF CONTRIBUTORY NEGLIGENCE - d. Corporate Negligence
Doctrine of Common Fault - It has been defined as
conduct on the part of the plaintiff or injured party,
contributing as a legal cause to the harm he has 48. Which of the following imposes purely
suffered, which falls below the standard which he is administrative liability on a government doctor?
required to conform to his own protection
a. Emergency Law (R.A. 8344)
b. Pharmacy Law
c. Code of Ethics of Public Servants
45. When the patient is usually inactive and virtually
d. Anti-Graft and Corrupt Practices Act (R.A.
places himself in the command or Control of the
3019)
physician, it is because of the doctrine of:

a. foreseeability
49. While doing an exploratory laparotomy, Dr. P.
b. assumption of risk Baya forgot to do a sponge and instrument count
because he was hurrying to catch a date with his intern.
c. superior knowledge As a result, he left a scalpel inside the
d. contributory negligence abdominal cavity which caused the death of his
patient. He is subsequently sued and
Rationale: prosecuted. He is most probably guilty of
Doctrine of Superior Knowledge - the physician has
superior knowledge and the patient just follows a. Murder
orders or instructions and usually places himself in b. Negligence
the command and control of the physician c. Homicide
d. Reckless imprudence resulting to homicide
Rationale:

46. When the surgeon is responsible for every corner of Reckless imprudence resulting to physical
the operating room, this doctrine is called: injury or homicide Action is careless, imprudent,
but unintentional Recklessness is NOT excusable
a. rescue doctrine and justifiable Maximum penalty is no more than 4
b. “res ipsa loquitor” years and 2 months, since it is less than 6 years,
c. both you can apply for probation, but you have to prove
d. Neither the probation administration that you are a modest
citizen

1
AY 2020
LEGAL
MEDICINE

50. While working as a warehouseman in Bocaue,


Bulacan, Mang Inyong was in charge of securing a
warehouse containing the non-explosive components of
firecrackers. While doing his rounds, he found a group
of robbers hauling away the contents of his warehouse.
In order to avoid capture, the group set fire to the
warehouse. After the blaze was controlled, Mang
Inyong was found dead in the remains of the warehouse
with his upper and lower extremities flexed and the
hands clenched. This is known as:

a. Rigor Mortis
b. Pugilistic attitude
c. Cadaveric spasm
d. Mummification

Rationale:

Pugilistic attitude is found

A. drowning

B. burn

C. stabbing

D. GSW

The limbs are flexed and the fingers partially


clenched simulating a “pugilistic” position of a boxer
as a result of heat stiffening.

1
AY 2020
Legal Medicine
Prelim Exam

1.A government physician may not practice his c. Corporation


profession within office hours. This is a limitation
imposed d. Company

by: Rationale:- A corporation is an artificial being created by


operation of law, having the right of succession and the
A. Religion powers, attributes and properties expressly authorized by law
or incident to its existence.
B. Self-imposed limitation A corporation, being a creature of law, "owes its life to the
state, its birth being purely dependent on its will," it is "a
C. Law creature without any existence until it has received the
imprimatur of the state acting according to law." A corporation
D. Medical Society will have no rights and privileges of a higher priority than that
of its creator and cannot legitimately refuse to yield obedience
to acts of its state organs.
A corporation has four (4) attributes:
2. A physician should never examine or treat patient of (1) It is an artificial being;
another without the letters knowledge and consent (2) Created by operation of law;
except when (3) With right of succession;
(4) Has the powers, attributes, and properties as
A. The patient expressly asks him to do so in the expressly authorized by law or incident to its
absence of attending physician? existence.

B. He is a relative of the patient


4. Before admission to the practice of medicine in
C. He will not charge the patient a professional fee the Philippines, an applicant must possess the
following requirements EXCEPT:
D. In case of emergency, the
attending physician is not readily A. Atleast 18yrs of age
available
B. Proper educational background
Rationale:- Art. IV, Sec. 15
C. Passed examination requirements
• “A physician should never examine or treat a hospitalized
patient of another without the latter’s knowledge and consent D. Holder of certificate of registration
except in cases of emergency.”
Rationale:- . Minimum age requirement: at least 21
years of age

3. An artificial being created by operation of law,


having the right of succession and the power, attributes
and propertiea expressly authorized by law incident to 5. CASE: Dr. B during hysterectomy procedure
its existence unintentionally cut patient A’s ureter. Patient A is
demanding 3.5M from dr. B for her mental anguish as
a. Juridical person a result of the incident. Patient is referring to what kind
of liability?
b. Artificial person
1
AY 2020 -
Legal Medicine
Prelim Exam

a. Administrative liability  Laws: Medical Act of 1959 as amended including the


Code of Ethics and Rules and Regulations of the
b. Civil liability PRC;

c. Criminal liability  Penalty: reprimand, suspension, to revocation of


license.
d. NOTA

Rationale:- A civil suit filed against physician and/or


hospitals is premised on recovery of damages for their 7. But since patient A cannot afford the professional fees
wrongful act or of employees. of the lawyer, she asked her husband to rather go to the
Cause of action for damages is based on: Professional Regulation Commission and directly
submit her complaint against Dr. B. The husband of
1. Breach of Contract patient is filing a complaint referring to what kind of
2. Tort(Quasi-delict )
liability

• Legal wrongdoing independent of a contract A. Administrative Liability

• Primary basis is negligence or fault of the physician as the B. Civil Liability


one directly responsible for the injury sustained by the
patient C. Criminal Liability

• Ordinarily, any malpractice action is based on torts or D. NOTA


quasi-delict in as much as negligence is usually a ground for
injury.

8.While in pain, patient A is reading a book entitled


Revised Penal Code of the Philippines. She is thinking
6. Patient A likewise went to a lawyer and asked that of invoking its provisions against Dr B. She must be
Dr. B’s license be revoked. Patient A is referring to thinking of planning to file a case against Dr B for
what kind of liability of a doctor?
A. Administrative Liability
A. Administrative liability
B. Civil Liability
B. Civil liability
C. Criminal Liability
C. Criminal liability
D. NOTA
D. NOTA
Rationale:- Art. 275, Revised Penal Code
Rationale:- ADMINISTRATIVE LIABILITY
 Abandonment of persons in danger
 Right to practice is temporarily withdrawn from the
and abandonment of one’s own victim
physician;

 A valid exercise of the police power of the State;  The penalty of arresto mayor shall be imposed upon:

- Anyone who shall fail to render assistance to any


2
AY 2020 -
Legal Medicine
Prelim Exam

person whom he shall find in an uninhabited place

3
AY 2020 -
Legal Medicine
Prelim Exam

wounded or in danger or dying, when he can render  Right to practice is temporarily withdrawn from the
such assistance without detriment to himself, unless physician;
such omission shall constitute a more serious offense
 A valid exercise of the police power of the State;
- Anyone who shall fail to help or render
 Laws: Medical Act of 1959 as amended including the
assistance to another which he has accidentally
Code of Ethics and Rules and Regulations of the
wounded or injured PRC;

 Penalty: reprimand, suspension, to revocation of


license.
9. One night at the hospital, Dr. B visited patient A in
her room when she still confined. Dr. B offered
sufficient compensation for patient A’s injuries. In this
11 . FATHER OF MEDICINE:
Situation, Dr. B is admitting his:
Ans. Hippocrates
A. Act of Administrative
Rationale:-
liability B.Civil liability

C. Criminal liability
12. Failure by a physician to perform his duty to
D. NOTA
his patient may be due to:
Rationale:- CIVIL LIABILITY
A. violation of a positive law
 Awarded against a physician to compensate for the
b. Negligence
injury he suffered on account of the physician’s act or
omission as a breach of the contractual relationship of both
c. Ignorance- 2 OTPS
parties;

 Laws. Civil Code of the Philippines and other related d. any of the above- REASON MENTIONED
laws; IN RATIONALE

 Art. 100, RPC states that “ Every person criminally RATIONALE:- Medical negligence happens when a
liable is civilly liable.”Penalty: damages doctor causes harm to a patient unknowingly, either
through simple ignorance or failing to take action
where it is needed.
10. In that same visit, Dr. B, said to patient A that he
did not violate any provision of the Medical 1959. Dr.
B. must be talking about what kind of liability? 13. If a married patient died and in the presence of the
legitimate Spouse, descendants, ascendant,brother and
Ans. Administrative liability
sister, to whom a claim for a professional fee be
Rationale:- ADMINISTRATIVE made?

LIABILITY a. Spouse

4
AY 2020 -
Legal Medicine
Prelim Exam

b. Descendants which For this reason do not require regulation


c. Ascendants d. NOTA
d. brother/sister RATIONALE:- The three-flunk rule is intended to
insulate the medical schools and ultimately the medical
RATIONALE:- The order is spouse, descendants,
profession from the intrusion of those not qualified to be
ascendants, siblings.
doctors. While every person is entitled to aspire to be a
doctor, he does not have a constitutional right to be a
doctor
14.If filipino physicians are allowed to practice in
spain, spanish doctors may also practice in the
philippines by virtue of the principle of:
16.In the right of physician to limit their practice, one
a. Exchange program of the following is not true:

b. Diplomacy a. Right to limit practice to certain specialty

c. International right b. Right to limit practice to certain place

d. Reciprocity c. Right to limit practice to certain hours

RATIONALE:- a person whose country's existing laws d. Right to limit practice to a certain nationality
permit citizens of the Philippines to practice medicine
RATIONALE:- RIGHT TO LIMIT HIS
under the same rules and regulations can be allowed by
MEDICAL PRACTICE
virtue of
 field of specialty
reciprocity
 private clinic or hospital

 within a political/geographical boundary


15. In DECS vs San Diego, the supreme court
explained the three-flunked rule. The following are  certain days of the week/hours of the day
statement true in relation to this case:  certain class of people

a. The three-flunk rule is intended to insulate  with due regard to dictate of conscience
the medical schools and ultimately the
medical profession from the intrusion of those  retirement
not qualified to be doctors
 imposed by the public, religion, professional ethics,
medical society, law, contract
b. The right to quality education

c. The medical profession directly affects the very


lives of the people, unlike other careers

5
AY 2020 -
Legal Medicine
Prelim Exam

17. Intern tata mad advertised himself to offer services Rationale:- MEDICAL JURISPRUDENCE
as a physician by means of cards and letterheads. The
following statements are true: deals with the aspect of law and legal concepts
in relation with the practice of medicine
a. He is guilty of illeagal pratice of medicine
MEDICAL JURISPRUDENCE includes:
b. He is guilty of criminal negligence
 Licensure and regulatory laws;
c. He is administratively liable
 Physician-patient-hospital relationship
d. AOTA together with the other paramedical
personnel, their rights, duties and obligations;
Rationale:- GROUNDS FOR ADMINISTRATIVE
LIABILITIES Sec.24, Art.III, Medical Act of 1959 as  Liabilities for non-compliance with the law.
amended

 Personal Disqualifications:
19. Laws passed between July 4, 1946 to September
1. Immoral or dishonorable conduct; 21, 1972 bear the prefix: - Republic Act a. C.A
( common wealth act)
2. Insanity;

3. Gross negligence, ignorance or incompetence b. P.D> yes( presidential decree


resulting in an injury to or death of the patient;
c. L.O. ( letter of instruction
4. Addiction to alcoholic beverages or to any habitforming
drug rendering him incompetent to practice medicine. d. R.A. ( republic act)

This act comes under BUSINESS AND Rationale:- Google and trans
PROFESSIONS CODE - BPC (code number 2052)

P.S ILLEGAL PRACTICE OF MEDICINE:- Practice of


medicine by any person not qualified and not duly- admitted 20. laws which govern the rules by which litigation
to perform medical acts in compliance with law. are conducted:

A. Substantive law

18. It is concerned with the aspect of law and legal B. Private law
concepts which has to do with the practice of
C. Adjective law
medicine:
D. Natural law
A. Medical ethics
Rationale:-
B. Legal medicine
Procedural law, also called adjective law, the law
C. Forensic medicine
governing the machinery of the courts and the
D. MEDICAL JURISPRUDENCE methods by which both the state and the individual

6
AY 2020 -
Legal Medicine
Prelim Exam

(the latter

7
AY 2020 -
Legal Medicine
Prelim Exam

including groups, whether incorporated or not)


enforce their rights in the several courts.
23.Pursuant to the Medical Act of 1959, a person shall
be considered as engaged in the practice of medicine.
21. Maldita’s daughter was suffering from a.Pre-employment physical examination
impacted tooth. She consulted Maldito who told
her to give paracetamol, a household remedy. b.Physically examine any person, and diagnose,
Which describe appropriately Maldito’s action? treat. operate or prescribe any remedy for
human disease, injury, deformity, physical,
a. Illegal practice of medicine mental and psychical condition

b. Illegal practice of pharmacy c.Who shall use the title of M.D. after his

c. Illegal practice of dentistry name d.All of the above

d.Not considered an act constituting practice of Rationale:- ACTS CONSTITUTING THE


medicine PRACTICE OF MEDICINE (pursuant to Sec.10, Art.
III of the Medical Act of 1959 as amended):
Rationale:- household remedies are not included in
practice of medicine, any person who examines and A) who shall for compensation, fee, reward in any form
prescribes or treats a patient is without supervision of a paid to him directly or through another, or even without
registed physician is practicing medicine the same, physically examine any person, and
diagnose, treat, operate or prescribe any
remedy for human disease, injury, deformity,
physical, mental, psychical condition or any
ailment, real or imaginary, regardless of the nature of
22.Manny is the son of a gastroenterologist, (Peter). the remedy or treatment administered, prescribed or
Upon graduation and before passing recommended;

the board exams, he put his name under a B) who shall by means of signs, cards, advertisement,
written or printed matter, or through the radio,
sign bearing his father’s name and started seeing patients.
television or any other means of communication,
A patient was mismanaged and suffered injury. Peter
was sued. What is the fault committed? either offer or undertake by any means or method to
diagnose, treat, operate, or prescribe any remedy for
a.Illegal practice of medicine against Manny
human disease, injury, deformity, physical, mental
b.Malpractice vs. Manny
or psychical condition;
c.Malpractice vs. Manny’s father
C) who shall falsely use the title of M.D. after his
d.None of the above name, shall be considered as engaged in the practice of
medicine.
Rationale:- Peter is sue for malpractice since his
name is used but Manny can be sued for illegal practice.

8
AY 2020 -
Legal Medicine
Prelim Exam

24. Qualification of candidates for the preliminary  A valid exercise of the police power of the State;
examination in the medical board examination are
the following except:  Laws: Medical Act of 1959 as amended including the
Code of Ethics and Rules and Regulations of the
A. He must be of good moral character PRC;

B. He must be aleast 21yrs of age  Penalty: reprimand, suspension, to revocation of


license.
C. He must have completed the first 2yrs of one
medical course.
26.The Board of Medicine is composed
D.NOTA
of a.one chairman and four members
Rationale:- Examination Requirements: must have
passed the corresponding Board Examination b.one chairman and two members

 Preliminary Examination c.one chairman and five

- At least 19 years of age; members

- Of good moral character; D. One chairman and six members.

- have completed the first two years of Rationale:- BOARD OF MEDICAL EDUCATION

the medical course Members -

P.S:- age requirement 21 is for practicing medicine and 1. Secretary of Health


not preliminary exam.
2. Director, Bureau of Private Schools

3. Chairman, Board of Medicine


25.reprimand is a sanction imposed against a
4. Representative, PMA
doctor when he is guilty of what liability?
5. Council of Deans,APMC
a. Administrative liability
6. Dean, UP-College of Medicine
b. Civil liability

c. Criminal liability
27. The department charged with confirming a foreign
d. No liability medical graduates documentary evidence when
applying for the medical board exam is:
Rationale:- ADMINISTRATIVE LIABILITY

 Right to practice is temporarily withdrawn from the


a.Department of Justice
physician;
b.Bureau of Immigration and Deportation

9
AY 2020 -
Legal Medicine
Prelim Exam

c.NBI d.Department of Foreign

affairs

1
AY 2020 -
Legal Medicine
Prelim Exam

Rationale:- Section 9. Candidates for Board


Examinations.-
29. The following EXCEPT one are rights of a
Candidates for Board examinations shall have physician, which one is the exception?
the following qualifications:
a.right to choose patient
He shall be a citizen of the Philippines or a citizen of
any foreign country who has submitted competent and b.right to limit practice of medicine
conclusive documentary evidence, confirmed by the
c.right to withdraw anytime as attending
Department of Foreign Affairs,9 showing that his
physician
country’s existing laws permit citizens of the Philippines
to practice medicine under the same rules and d.right to determine appropriate management procedure
regulations governing citizens thereof.
Rationale:- RIGHTS OF PHYSICIAN INHERENT

RIGHTS

 to choose patients
28. The following are qualified to practice medicine
 to limit practice of medicine
in the Philippines:
 to determine appropriate management procedures
a.Those who have complied with the pre-requisites to
the practice of medicine in accordance with Sec. 8 of the  to avail of hospital services
Medical Act of 1959

b.Balikbayan Physicians pursuant to PD 541


30.The following is/are right/s of the patient,
c.Both
except: a.right to religious belief
d.Neither
b.right to privacy
Rationale:- Qualified to practice medicine in the
c.right not to be detained
philippines
d.right to be sent home at all times
1. Those who have complied with the prerequisites to
the practice of medicine in accordance with Sec. 8, Art. Rationale:- RIGHTS OF PATIENTS
III, Medical Act of 1959 as amended;
1. Right to give consent to diagnostic and
2. Those who can have limited practice without any
treatment procedures
certificate of registration in accordance with Sec.12,
Art. III, Medical Act of 1959 as amended: 2. Right to religious belief

3. “Balikbayan” Physicians pursuant to PD 541, Allowing 3. Right of privacy


Former Filipino Professionals to Practice Their Respective
Professions in the Philippines Proviso 4. Right to disclosure of information

1
AY 2020 -
Legal Medicine
Prelim Exam

5. Right to confidential information d.retainers fee

6. Right to choose his physician

7. Right of treatment

8. Right to refuse necessary treatments

A patient cannot be detained in a hospital for nonpayment of


the hospital bill. The law provides a remedy for them to
pursue by filling the necessary suit in court for the recovery of
such fee or bill.

31. The Government (State) has the power to regulate


practice of professions in the country, including the
profession of medical practice. The Government
therefore has passed regulatory laws for this purpose
and violation of these regulatory laws will result to:

a.Administrative Liability

b.Civil Liability

c.Criminal Liability

d.None of the

Above

Rationale:- “A valid exercise of the police power of


the State”- Medical Act of 1959

32. The kind of professional fee which is


unethical because the amount wagers with
unforeseeable contingencies is known as:

a.contingent fee

B. Dichotomous fees

c.straight fee

1
AY 2020 -
Legal Medicine
Prelim Exam

Rationale:- Contingent fee- Fee depends Catholicism


on success or failure of treatment instituted; no
fee will be received for efforts until a result is b.Christianity
attained- unethical (does not account for c.Paganism
unforeseen circumstances)
d.None

Straight fee- Amount of medical fee is


dependent on what will be the remaining
balance when all of the other expenses have
been paid- unethical(quality compromised
in the name of profit)

Dichotomous fee (commission or fee splitting)

- Requires services of person/s who may act


as an agent (chasers, runners, touts or tipsters)
to solicit patients.
Sharing a fee with another physician,
laboratory or drug company, not based on
services performed.- unethical.

Contractual fee- Contract stipulates the


nature of procedure; specifically states the
value of such medical service either orally or
in writing. Ethical

Retainer fee- Measured by the space of


time and not by the quality or quantity of
medical services rendered. Ethical (based on
length of time of service)

33. The official religion of the


republic of the Philippines:

a.Roman

1
AY 2020 -
Legal Medicine
Prelim Exam

degree of care in the locality

34. The penalty of illegal practice of medicine is:

a. imprisonment of 1-5 years, fine of P1,000–P10,000

b.imprisonment of 1-10 years, fine of P1,000–P5,000

c.imprisonment of 1-15 years, fine of P1,000–P5,000

d.imprisonment of 1-10 years, fine of P5,000–P10,000

Rationale:- Pursuant to Sec.28, Art. IV, Medical Act of


1959 as amended – Any person found guilty of “illegal
practice” shall be punished by a fine of not less than
1,000 pesos or more than 10,000 pesos with subsidiary
imprisonment in case of insolvency or by imprisonment
of not less than 1 year no more than 5 years, or by both
such fine and imprisonment, in the discretion of the court.

35 .The physician has duties to his patient, and they are


the following, EXCEPT:

a.duty to possess average knowledge and skill

b.duty to possess highly specialized knowledge

c.duty to exercise ordinary

diligence d.duty to exercise good

faith

Rationale:- DUTIES and OBLIGATIONS Imposed on


the Physician in the Physician-Patient Relationship

1. He should posses the knowledge and skill of which


an average physician is concerned; General
practitioner vs Specialist

2. He should use such knowledge and skill with


ordinary care and diligence;

–“locality rule” – the standard of care is measured by the

1
AY 2020 -
Legal Medicine
Prelim Exam

–“similar locality rule” – diligence is


determined when the other physicians in the
locality or similar locality could

have acted the same way

–“national standard of care” - the diligence is


determined on what is applicable on a national
standard basis

3. He is obliged to exercise the best judgment;

4. He has the duty to observe utmost good faith.

36.The privilege or franchise granted by the state


to any person to perform medical acts upon
compliance with law:

a.exercise of medicine

b.practice of

medicine c.medical

informatics d.medical

malpractice

Rationale:- PRACTICE OF MEDICINE

1. What is the “practice of medicine”?

2. It is a privilege or franchise granted by the State


to any person to perform medical acts upon
compliance with law, that is, the Medical Act of
1959 as amended which has been promulgated by
the State in the exercise of police power to protect
its citizenry from unqualified practitioners of
medicine.

4. It is diagnosing and applying and the usage


of medicine and drugs for curing, mitigating, or
relieving bodily disease or conditions.

1
AY 2020 -
Legal Medicine
Prelim Exam

37. The right to regulate the practice of medicine by Medicine, Obstetrics and Gynecology, Pediatrics and
the State is based on:

a.Police power

b.Power of eminent

domain c.Medical Act of

1959 d.Constitution

Rationale:-

compliance with law, that is, the Medical Act of 1959 as


amended which has been promulgated by the State in the
exercise of police power to protect its citizenry
from unqualified practitioners of medicine

38. The scope of the preliminary examination in


the Philippine Medical Board Examination is:

a.Anatomy, Physiology, Pharmacology,

Microbiology b.Anatomy and Histology,

Physiology, Biochemistry, Microbiology and

Parasitology

c.Anatomy and Histology, Pathology, Physiology,


Pharmacology

d.Anatomy and Histology, Medicine, Preventive


Medicine, Biochemistry

Rationale:-

Scope of Examination:

Preliminary - Anatomy and Histology,


Physiology, Biochemistry, Microbiology and
Parasitology

Final - Pharmacology and Therapeutics, Pathology,


1
AY 2020 -
Legal Medicine
Prelim Exam

Nutrition, Surgery and Ophthalmology, b. injury to his patient because of defective


Otorhinolaryngology, Preventive Medicine hospital equipment
and Public Health, Legal Medicine, Ethics
and Medical Jurisprudence.

39. Tito Paeng is a faith healer who beats up


his clients to exorcise the evil spirits causing
the disease. One follower who was injured
sued her. Which of the following statement is
correct?

a.Freedom to act out ones belief is a


constitutionally protected right

b.Gloria is not guilty of inflicting

physical injuries c.Gloria is not

practicing the tenets of her religion

d.All statements are incorrect

Rationale:- Faith Healing

 There is nothing in the Medical Act of 1959


exempting it from the definition of the acts which
constitute practice of medicine;

 Related to constitutional guarantee to


religious freedom (freedom to believe and
freedom to act in accordance with one’s
belief);

 Acted in pursuance of his religious belief and


with the tenets of his church he professes, not
deemed to be a practice of medicine but part of his
religious freedom.

40. Vicarious liability of a physician


refers to his responsibility for:

a. injury to his patient resulting from failure of


the patient to follow instructions

1
AY 2020 -
Legal Medicine
Prelim Exam

c. injury to his patient because of d.All of the choices


misunderstanding between him and his patient
Rationale:-
d.injury to his patient because of a negligent
act committed by his nurse Practice of medicine is is a privilege or franchise
granted by State to any person to perform medical acts
Rationale:-DOCTRINE OF VICARIOUS LIABILITY upon compliance with law, that is, the Medical
or Doctrine of Imputed Negligence/Command Act of 1959 as amended which has been
Responsibility. -Vicarious liability means the promulgated by the State in the exercise of
responsibility of a person, who is not negligent, for the police power to protect its citizenry from
wrongful conduct or negligence of another Rules unqualified practitioners of medicine.
applied in determining the vicarious liability for the
negligent acts of the resident physicians, nurses and
others employees. P.S Written or Statutory Law (Lex Scripta)-
composed of laws which are produced by the country’s
legislations and which are defined, codified and
41. What could be the purpose for the Office of incorporated by the law-making body. Ex. Philippine
the Prosecutor in conducting an investigation? Laws

a.Probable cause

b.Improbable cause 43. Which of the following acts of diagnoses


and treatment is considered practice of
c.Walk for a cause
Medicine?
d.All of the above
a.Use of electric machine by barbers in giving treatment
Rationale:- Probable cause- sufficient reason based to one who sought to beautiful his body
upon known facts to believe a crime has been
b.A nurse that takes the blood pressure reading and gives
committed or that certain property is connected with a
advice and prescribes
crime.
Probable cause must exist for a law enforcement treatment
officer to make an arrest without a warrant, search
without a warrant, or seize property in the belief the c.The application of medicated
items were evidence of a crime. massage d.Use of an electric vibrator.

Rationale:- constitute practice of medicine:


42. What is the statutory basis for the regulation of
1. One who takes BP reading;
the practice of medicine? a.Medical Act of 1959
2. Application of medicated massage;
b.Power of the State
3. Hospital;
c.1987 Philippine Constitution

1
AY 2020 -
Legal Medicine
Prelim Exam

4. Nurse anesthesist

1
AY 2020 -
Legal Medicine
Prelim Exam

b.recovery of patient (same in above scenario)

44. .Which of the following does not affect termination


of physician-patient relationship?

a.withdrawal of physician without patient


consent.

b.recovery of

patient c.death of

patient d.death of

physician

Rationale:- TERMINATION It is the time when the


duties and obligations by a physician to his patient
ceases. The following are some ways of termination of
the relationship:

1. Recovery of the patient or when the physician


considers that his medical services will no longer
be beneficial to the patient;

2. Withdrawal of the physician provided:

a)with consent of the patient, and b) patient is given


ample time and notice;

3. Discharge of the physician by the

patient; 4.Death of either party

5. Incapacity of the physician

6. Fulfillment of the obligation.

45. Which of the following DOES NOT affect


a termination of physician-patient relationship?

a.Abandonment by the physician

2
AY 2020 -
Legal Medicine
Prelim Exam

c.patient withdraws from the contractual obliged to attent to his patient in the latter’s
relationship with his doctor home even if

d.death of physician

Rationale:- TERMINATION It is the time when


the duties and obligations by a physician to his
patient ceases. The following are some ways of
termination of the relationship:

1. Recovery of the patient or when the


physician considers that his medical services
will no longer be beneficial to the patient;

2. Withdrawal of the physician provided:

a)with consent of the patient, and b) patient


is given ample time and notice

3. Discharge of the physician by the

patient; 4.Death of either party

5. Incapacity of the physician

6. Fulfillment of the obligation.

46. Which of the following statement is not valid?

a. A medical practitioner has the right to


limit his practice to certain days of a week
and

certain hours of the day

b. Medical practitioner may refuse to make those


calls and concentrate his practice only in his
clinic or hospital

C. except in cases of emergency, a physician is


not obliged to serve every patient who solicits
his services

D. once a physician-patient relationship


has been established. A physician is

2
AY 2020 -
Legal Medicine
Prelim Exam

the patient has moved a town 20 kilometers and not duly-admitted to perform medical acts in
from the city where the physician practices compliance with law.
Rationale:- RATIONALE:- RIGHT TO LIMIT HIS
MEDICAL PRACTICE

 field of specialty

 private clinic or hospital

 within a political/geographical boundary

 certain days of the week/hours of the day

 certain class of people

 with due regard to dictate of conscience

 retirement

 imposed by the public, religion, professional ethics,


medical society, law, contract

47. Which of the following statements are true


regarding illegal practice of medicine?

a.The patient expressly asks the resident to do so in the


absence of attending physician

b.A person who had treated and prescribed for


certain from whom he received monetary
compensation without having previously
obtained the proper certificate of registration
from the Board of Medical Examiners

c.A faith healer

d.All of the

choices

Rationale:- ILLEGAL PRACTICE OF MEDICINE

 Practice of medicine by any person not qualified

2
AY 2020 -
Legal Medicine
Prelim Exam

good faith d.best judgement

48.Which of the following statements is/are Rationale:- DUTIES and OBLIGATIONS Imposed on
true of faith healing in relation to the practice the Physician in the Physician-Patient Relationship
of medicine:

a.There is nothing in the medical Act of 1959


exempting faith healing from the definition of
the acts which constitute practice of medicine

b.If a person acted pursuance of his religious


belief and the act is in accordance with the tenets
of his church, it is deemed to be part of his
religious freedom

c.Both

d.Neither

Rationale:- Faith Healing

 There is nothing in the Medical Act of 1959


exempting it from the definition of the acts which
constitute practice of medicine;

 Related to constitutional guarantee to


religious freedom (freedom to believe and
freedom to act in accordance with one’s
belief);

 Acted in pursuance of his religious belief and


with the tenets of his church he professes, not
deemed to be a practice of medicine but part of his
religious freedom.

49.Which one of the following is not an


obligation imposed on the physician-
patient relationship?

a.knowledge and skill of an average physician

b.use of knowledge and skill with


extraordinary care and diligence this
one.

c.duty to observe utmost


2
AY 2020 -
Legal Medicine
Prelim Exam

1. He should posses the knowledge and skill of which Practice Their Respective Professions in the Philippines
an average physician is concerned; General Proviso:
practitioner vs Specialist  good standing prior to their departure and in
their adopted country;
2. He should use such knowledge and skill with
ordinary care and diligence;  Have registered with PRC and paid
their professional fee;
–“locality rule” – the standard of care is measured by the
degree of care in the locality  Pay the corresponding income tax;
–“similar locality rule” – diligence is determined when
the other physicians in the locality or similar locality
could

have acted the same way

–“national standard of care” - the diligence is


determined on what is applicable on a national standard
basis

3. He is obliged to exercise the best judgment;

4. He has the duty to observe utmost good faith.

50.Which statement is not valid: A balikbayan


physician formerly a citizen of the Philippines and had
previously passed the medical licensure examination in
the Philippines. May practice his profession this
country provide he:

a.pay corresponding income tax due in all income


realized in practicing in the Philippines

b.Must become a Filipino citizen once more c.Is


a professional practitioner of good standing prior to
departure from the Philippines and in his adopted
country

d.Shall register with PRC and pay the proper


professional license fee TM

Rationale:- “Balikbayan” Physicians pursuant to PD


541, Allowing Former Filipino Professionals to
2
AY 2020 -
LEGAL MEDICINE
1ST SEMESTER PRESEM EXAM

1. The relevant Philippine law on anti-child abuse is 4. If the hymenal laceration involves more than the base of
a. R.A. 7610 the hymen, the degree of laceration is:
b. PD 604 a. Deep
c. R.A. 9262 b. Compound
d. PD 541 c. Complete
d. Superficial
Rationale: RA 7610- Special Protection of Children
Against Abuse, Exploitation and Discrimination Act Rationale:
Superfici - laceration does not go beyond 1/2
PD 604- YOUTH DEVELOPMENT, Dee - goes beyond 1/2 but does not reach base
PHYSICAL FITNESS AND AMATEUR SPORTS Complete- reaches base
DEVELOPMENT Compound or complicated - lacerates adjacent
structures (cervix, vagina canal, uterus, rectum...)
RA 9262- VIOLENCE AGAINST WOMEN
AND THEIR CHILDREN
5. Those guilty of adultery who could be prosecuted
PD 541- ACT TO PROVIDE LIFE PENSIONS a. Woman alone
FOR UNIFORMED OFFICERS b. Man alone
c. Married woman and the paramount who knew
her to be married
2. The most prominent sign of death is d. Whole family
a. brain death
b. progressive fall of the body temperature
c. cessation of heart action and circulation 6. All suspected cases of child abuse must be
d. cessation of respiration reported n what period of time from
knowledge?
a. 12 hours
3. It is found in the most dependent portion of the b. 24 hours
body involving the s is c. 48 hours
dull red purplish, uperficial layer of the ski n, color d. one month
uniform nor elevated which
appear after death.
a. postmortem lividity Rationale: The following are mandated to make a
b. postmortem suggilation report, either orally or in writing, to DSWD/LSWDO
c. livor mortis within forty-eight (48) hours, the examination and/or
d. all of the above treatment of a child who appears to have suffered from
e. none of the above abuse:

7. Carnal relation with a female under 12 years old is


called:
a. child rape
b. statutory rape
c. simple seduction
d. qualified seduction

Rationale: When the woman is under 12 years of age


-called statutory rape
-regardless of whether or not force or intimidation

1
AY 2020 - 2021
is applied or the child is not deprived of her reasons or 12. Lewd design is
otherwise unconscious a. A requisite in consented abduction
-even if child consented or even if child a Prostitute b. Requisite in acts of lasciviousness
c. Desire to have sex
d. Cheating misrepresentative
8. What is the penalty for indirect contempt
Rationale: Lewd design is the intent of the abductor to
fine not exceeding thirty thousand pesos or have sexual intercourse with the woman abducted.
imprisonment not exceeding six (6) months, or both

13. Virginity is a requirement in this crime:


9. The element of deceit is required in case of: A. simple seduction
A. forcible obduction B. qualified seduction
B.consented obduction C. forcible abduction
C. simple seduction D. white slave trade
D. qualified seduction
Rationale:
Rationale: Ordinary Qualified Seduction
SIMPLE SEDUCTION: a. offended party must be a virgin
b. offended party must be over twelve years and
Elements: under eighteen years of age
1. The offended party is over 12 but less than 18 c. there must be sexual intercourse between the
years of offender and the offended party; and
age; d. the sexual act was done through abuse of authority
2. The offended party must be single or a widow of or confidence
good
reputation;
3. There must be sexual intercourse done by the 14. The penalty for a convicted concubine -
offender
with her; and Five years imprisonment
4. The sexual act must be committed by means of
deceit
Rationale: Concubinage is punishable by imprisonment
ranging from 6 months and 1 day to 4 years and 2
10. Under the law, a child is defined as one who is months (5 years). On the other hand, the mistress is
below merely imposed a punishment of destierro.
a. 21 years old
b. 16 years old 15. Homosexuality is a ground for
c. 13 years old a. legal separation
d. 18 years old b. annulment
Rationale: RA 7610 b “Children” refers to person c. divorce
s of age or those over but d. any of the above
elow eighteen (18) year
are unable to fully take care of themselves or protect
themselves from abuse Rationale: Homosexuality
oncealmentis ofa homosexualit
ground for legal
separation. C g
round for annulmen y is a
g round for t. Homosexuality is not a
11. Its importance approximate time of death

Rigor mortis 16. Official autopsy is also known as

Medicolegal autopsy

2
AY 2020 - 2021
17. The principle of Stare Decisis states that D. cadaveric rigidity
a. The Justices of the Supreme Court can only be
removed by impeachment Rationale: The wounding weapon is firmly grasped
b. Only the Supreme Court can revise, by the hand of the victim (cadaveric spasm)
modify or affirm its own decision
c. Only the President of the Philippines can
22. The most important finding in a genital exam is in
review the decisions of the Supreme Court
the
d. The Three branches of government are all co-
a. Hymen
equal branches
b. Vaginal canal
c. Vestibular mucosa
Rationale: A principle that, when the court has once laid d. Labia majora and minora
down a principle of law or interpretation as applied to a
certain state of facts, it will adhere to and apply to all
Rationale: PARTS OF THE FEMALE BODY TO BE
future cases where the facts are substantially the same.
CONSIDERED IN THE DETERMINATION OF THE
CONDITION OF VIRGINITY.
18. Intentional and deliberate killing of a brother, 1. Breast
sister, uncle or aunt. 2. Vaginal Canal
a. Infanticide 3. Labia Majora and Labia Minora
b. Murder 4. Fourchette (present a V-shape appearance as the
c. Homicide two labia minora unite posteriorly.)
d. Parricide 5. Hymen- determines Physical virginity
Rationale:
Infanticide- The killing of a child less than 3 days 23. P ost mortem examination includes the following
old.
except
Murder - homicide with intent (pre-planned) a. may involve incision of skin
Homicide- act of o ne human killing another b. objective is to determine the cause of death
Parricid - the ki lling of a parent or other near c. external examination of the body
relative. d. aspiration of fluid by way of a syringe

Rationale: Postmortem examination refers to an external


19. What manner of death need not be autopsied? examination of a dead body
A. suicides witho
incision
B. accidental death
C. death unattended by physician
24. The medical witness who r
efuses to answer
D. death from Chronic illness questions propounded to him while testifying be may
cited for
20. Cause of death could still not be determined
despite an autopsy done a. Dishonarable conduct
b. Perjury
a.negative post-mortem c. Indirect contempt
b.negligent autopsy d. Direct contempt
c.negative autopsy
d.all of the above Direct Contempt Punished Summarily – “A
person guilty of misbehavior in the presence of or so
near a court as to obstruct or interrupt the proceedings
21. The presence of weapon which is highly by before the same, including disrespect toward the court,
graspe the hand of a victim of a shooting offensive personalities toward others, refusal to be sworn
i ncide is an example of or to answer as witness, or to subscribe an affidavit or
A. Rigor mortis deposition when lawfully required to do so, may be
B. death stiffening summarily adjudged in contempt by such court
C. cadaveric spasm

3
AY 2020 - 2021
25. One with an IQ of below 20 is considered -Fingerprints are not changeable
a. idiot
30. Cases when trauma or disease k that
b. average ill
c. moron there is no opportunity for sequel or complications to
d. imbecile develop is known as:
a. complicated cause of death
Rationale: An idiot is exempted from criminal liability. b. secondary cause of death
An idiot has an IQ of 0 – 20 c. immediate cause of death
d. proximate cause of death
26. The penalty for a convicted concubine is:
a. reprimand or censure Rationale: Primary or immediate cause of death- This
b. five years imprisonment applies to cases when trauma or disease kill quickly that
c. Destierro there is no opportunity for sequelae or complications to
d. fine of five thousand pesos develop.
31. Deceit is a requisite in what crime
Rationale: In concubinage the penalty for the man is
lower by one degree (prison correctional minimum & a. Qualified seduction
medium) while the concubine is given a separate penalty b. Forcible seduction
which is "destierro" or distance prohibition; & no c. Simple seduction
imprisonment d. Consented seduction
Rationale: Simple seduction – seduction of a woman
27. Insanity in criminal law is
who is single or a widow of good reputation,
a. mitigating circumstance o ver 12 but under 19 years old, committed by
b. aggravating circumstance means of deceit
c. exempting circumstance
32. A doctor has the duty to report all types of child
d. qualifying circumstance
abuse to
Rationale: As an Exempting Circumstance: a. Military
Art. 12, Revised Penal Code — Circumstances which b. City Health Office
exempt from criminal liability — The following are c. DSWD units
exempt from criminal liability: An imbecile or an d. PRCS
insane person, unless the latter has acted during a lucid
Rationale: A ll hospitals, clinics and other institution s
interval.
as well as p rivate s providing treatment -
28. As a witness in court, a physician may refuse to shall, within f orty-eight from knowledge of the
answer a question propounded to him if his answer case, report in writing to the city or provincial fiscal or
to the L
a. it will be self-incriminatory
b. will antagonize the judge f
c. will blacken the reputation of a superior ocal Council for the Protection o
d. will not be relevant to the case at issue Childre or to the nearest unit of the D
epartment
f Social Welfare (Ministry of Social Service and
Development), any case of a maltreated or abused child,
or exploitation of an employed child contrary
29. The o to the provisions of labor laws.
dds in two person s having the same exact
fingerprint s is
33. When a person has sexual desire with the
a. 1 to 60 billion of universally used
b. 1 to 64 billion notes identi
c. 1 to 640 billion ficati
d. 1 to 6 trillion on. It
is
Rationale: Fingerprinting is the most valuable method

4
AY 2020 - 2021
dead: a. gmalionism
b. P Narcissism
c. y Necrophilia
d. Partialism
because: Rationale: Necrophilia- A sexual perversion
characterized by erotic desire or actual sexual
-There are no two identical fingerprints – 1:64B intercourse with a corpse.

5
AY 2020 - 2021
37. The penalty or punishment for indirect contempt is:
34. An IQ of 95 is considered to be a. A fine of up to thirty thousand pesos and a
a. superior prison term of up to three months
b. above average b. A fine of up to two hundred pesos and a
c. average prison term of up to 10 days
d. below average c. A fine of up to three thousand pesos and a
prison term of up to 10 months
Rationale: d. A fine of up to thirty thousand pesos and
I .Q. a prison term of up to six months
Above 140- "Near" genius or genius
1 20- - Very superior intelligence
1 10- - Superior intelligence Rationale: Punishment for indirect contempt. – If the
90-110- Normal or average intelligence respondent is adjudged guilty of indirect contempt
80-90- Dullness, rarely classified as feeble-minded 70- committed against a Regional Trial Court or a court of
80 Borderline deficiency, sometimes classified as equivalent or higher rank, he may be punished by a fine
dullness, often as feeble-minded not exceeding thirty thousand pesos or imprisonment not
Below 70- Definitely feeble-minded exceeding six (6) months, or both.

35. The presence of a weapon which is tightly


grasped by the hand of the victim of shooting incident 38. The medical witness who refuses to attend the
is an example of h earing of the case may be cited for
a. Death stiffening a. perjury
b. Rigor mortis b. dishonorable conduct
c. Cadaveric spasm c. indirect contempt
d. Cadaveric rigidity d. direct contempt
Rationale: A physician who is subpoenaed by a trial
Rationale: Cadaveric Spasm – instantaneous rigidity of court but failed to appear without justifiable reason is
the muscles that occurs at the moment of death due to guilty of indirect contempt which is punishable by fine
extreme tension, exhaustion or injury to the nervous and imprisonment of not more than one month
system or injury to the chest. In cases of cadaveric
spasm, a weapon may be held in the hand before death 39. This term describes a woman who does not
and can only be removed know anything about the sex act and with an i ntact
with difficulty. hymen
a. Virgo-intacta
36. The following authorized to perform autopsies, b. Prostitute
a.EXCEPThealth officers c. Morally virgin
b. medical officers of law enforcement d. Demi-virginity
agencies
c. the medicolegal officer of the Rationale:
Commission on Human Rights Demi-virgin – condition of a woman who permits any
d. members of the medical staff of accredited form of sexual liberties as long as they abstain from
hospitals rupturing the hymen by sexual act. Virgo intacta – this
term refers to women who had isolated sexual
Rationale: Persons authorized to perform intercourses or even habitually provided she had not
autopsies: born a child
Morally virgin – state of not knowing the nature of
1. H ealth s sexual life and not having experienced sexual
2. Medical officers of l aw enforcement agencies relation; applies to children below the age of
3. Members of the medical staff of accredited puberty and whose sex organs and secondary sex
hospitals characters are not yet developed.
Prostitute – are women, who for money or profit,
habitually indulge in sexual intercourse oe lascivious
conduct. Its types include being either a call girl,
hustler, door knocker, factory girl.
6
AY 2020 - 2021
Provisions of the revised penal code include: 43. The most important finding in a genital
corruption of minors and white slave trade. examination is in the
a. Hymen
40. This is a medical evidence known or addressed to the b. Vestibular mucosa
senses of the court not limited to the sense of vision, but c. breasts
it is intended to that of hearing, taste, smell and touch d. Labia majora and minora
a. experimental evidence
Rationale: The hymen is used to determine whether
b. corpus delicti evidence there was forcibly especially in rape cases
c. documentary evidencechil and in cases of child abuse.
d. autoptic or real evidence
44. Reckless Imprudence resulting in homicide
Rationale: Autoptic or Real Evidence: under Art 365 of the Revised Penal code is punishable
This is an evidence made known or addressed to with imprisonment of up to
the senses of the court. It is not limited to that which is a. Five years and two months
known through the sense of vision but is extended to b. Four years and two months
what the sense of hearing, taste, smell and touch is c. Six years and one day to twelve years
perceived. d. Reclusion Perpetua
Rationale: Reckless imprudence resulting to physical
41. Forcing a man to sodomize the sex offender is injuries (Article 365 in Revised Penal Code)
considered
a. seduction - careless, reckless, inexcusable, not justified
b. no crime committed? - prison term: 4 yrs & 2 months
c. acts of lasciviousness- - you can apply for probation
d. rape through sexual assault
45. Intentional and deliberate killing of a brother,
Rationale: sister uncle or aunt
Sodomy a. Infanticide
- anal sex b. Murder
- Old law -acts of lasciviousnessless than 6 yrs c. Homicide
imprisonment d. Parricide
- New law- considered as rape Rationale:
Infanticide- is the killing of a child less than three days
42. Sexual intercourse is not necessary in this crime old.
a. Seduction Parricidal – killing of one’s relative
b. Concubinage Murder- with treachery, taking advantage of superior
c. Adultery strength with the aid of armed men, or employing means
d. Abduction to weaken the defense or of means or persons to insure
or afford immunity.
Rationale: Abduction – is the carrying away of a woman In consideration of a price, reward or promise
by the abductor with a lewd design. Lewd design is the
intent of the abductor to have sexual intercourse with the
woman abducted. The intent to seduce is sufficient, s 46. A grandmother was insisting that her helper be
n exual intercourse is not. But charged with infanticide for the death of her 5-day
ecessary
the forofthe
presence conviction
unchaste of desin
or lewd the abductor
is necessary. . The Assistant City Prosecutor
old
Concubinage – any husband who shall keep a mistress in disagreed and said that an infant is defined in infanticide
the conjugal dwelling, or shall have as
a. A newborn upto 28 days old
b. A child less than 3 days old
c. A baby upto 2 years old
s exu intercourse under scandalous d. A child less than 2 months old
circumstances, with a w oman who is not his wif e, or Rationale: Infanticide is the killing of a child less
shall cohabit with her in any other place than three days old.

7
AY 2020 - 2021
47. That kind of evidence necessary for conviction b. acts of lasciviousness
which proves the fact in dispute without the aid of c. adultery
any inference or and which d. concubinage
correspond to the precise or actual point at issue
a. preponderance of evidence Rationale: Acts Considered Lascivious:
b. circumstantial evidence 1. Embracing, kissing and h
c. direct evidence olding the woman's
breast
d. testimonial evidence the man's
2. Placing of genital organprivate organ over a girl's
Rationale:
Direct evidence- That which proves the fact in dispute
without the aid of any inference or 50. Its medico-legal- importance is to approximate the
presumption. Corresponds to the precise or actual time of death
Circumstantial
point at evidence- The proof of fact/s from which, a. muscular contraction
taken either singly or collectively, the existence of a b. cadaveric spasm
particular fact in dispute may be inferred as a necessary c. instantaneous rigidity
or probable consequence. Sufficient to produce d. rigor mortis
conviction when:
-There is more than one circumstance Rationale: The following are used to determine the t
-The facts from which the inferences are derived are pallor mortis, algor mortis, livor
proven ime of
mortis, rigor mortis, and decomposition. Cadaveric
-When the combination of all the circumstances is such spasm is used to determine the nature of death.
as to produce a conviction beyond reasonable doubt
51. Finger printing is considered to be the most
valuable method of identification and is universally used
because of the following factors except
48. Reasons for the in admissibility to the cour t of a. Odds in having the same fingerprint with
the result of Lie Detector examinatio n are the someone else is 1 is to 100 billion
following, excep t b. fingerprints are not changeable
a. the test has not gained a degree of development c. finger may be wounded or burned but the
beyond experimental stage whole pattern will still re-appear
b. there is no way to assure that a qualified examiner d. fingerprints are formed in the fetus in the
administered the test fourth month of pregnancy
c. polygraph techniques are still in the experimental Rationale:
stage Fingerprinting is the most valuable method of
d. the test cannot be relied upon because the identification. It is u niversally d because:
margin of error is at least 40%
-There are no two identical fingerprints – 1:64B
Rationale: Reasons for the inadmissibility to court of -Fingerprints are not changeable
the result of Polygraph Examination:
Still experimental and is not yet standardized
52. Nurse Antonina decided that her 55 y/o patient had
The trier of fact is apt to give almost exclusive
suffered enough from the pain of bronchogenic
weight to the polygraph expert’s opinion.
carcinoma. She decided to end his suffering by giving
There is no way to assure that a qualified examiner
him 1000 mg. of oxycodone. As a result, the patient dies.
administered the test.
She is most probably guilty of
Since the polygraph involves a certain unconscious
a. Homicide
quality of the examinee, he may unwittingly waive. The
b. Infanticide
test cannot be relied upon because of its 25% e
c. Murder
49.rr Mashing of the breasts of a married man who is d. Parricide
Rationale: The following would fall under the criteria
having an a of murder. The doctor played an active participation
married is ffair with a who knew him to be in the death of a perso e n and with ex plicit to
a. qualified seduction nd his life.

8
AY 2020 - 2021
53. Fractured skull recovered during exhumation is an
example of
a. Real evidence
b. Documentary evidence
c. Experimental evidence
d. Circumstantial evidence
Rationale:
Autoptic or Real Evidence:
Made known or addressed to the senses of the court
h the
sense of vision but is e
NOT limited to that which xtendis known throug of
d to what the sense
hearing, taste, smell, and touch is perceived

54. It is an expressed admission c of having


a. Confusion
b. ommitted an act but not as to the guilt
Confession
c. Interrogation
d. Admission

Rationale: Confession – expressed


acknowledgement by the accused in a criminal
case of the truth of his guilt as to the crime charged,
or of some essentials thereof. nds of de Admission – a
statement of fact by the accused which do not directly
involve an acknowledgement
of guilt of the accused.

55. The following are kinds of death EXCEPT


a. state of suspended animation
b. regulated death
c. cellular or molecular death
d. somatic or clinical death

Rationale:
Kinds of Death:
Somatic death or clinical death – state of the body
in which there is c omplete, t and
continuous cessation of the vital functions of the
brain, heart and lungs which maintain life and health
Molecular or cellular death – after cessation of the
v ital of the body there is still animal life
among individual cells. This is evidenced by the
presence of excitability of muscles and ciliary
movements and other function of individual cells.
State of suspended animation or “apparent death” -
This condition is n t ot really death but merely a
cranscient loss of consciousness or temporary
essation of the vital function s of the body on
account of disease, external stimulus or other forms of
influence.

9
AY 2020 - 2021

You might also like